L'Astronomie dans l'apprentissage des Mathématiques

Auteurs: Alain Vienne, Stéphane Erard, Jérôme Thiébaut, Arnaud Beck, Stéfan Renner, Marc Fouchard, Florent Deleflie

L'Astronomie dans l'apprentissage des Mathématiques

Introduction au projet

Sujets :

Ce site, "L'Astronomie dans l'apprentissage des Mathématiques" est une émanation de "Astrophysique Sur Mesure". Il utilise l'Astronomie comme source d'exemple dans l'apprentissage des Mathématiques. Il est structuré en applications comprenant chacune un ou plusieurs exercices avec les corrigés.

La coordination pédagogique de ce projet est assurée par Alain VIENNE (Université Lille1). Initié par Françoise Roques (Observatoire de Paris), le groupe est constitué de: Arnaud Beck (2008-2009, Observatoire de Paris), Florent Deleflie (2010- , IMCCE), Stéphane Erard (2008- , Observatoire de Paris), Marc Fouchard (2008- , Université Lille1), Stéfan Renner (2008- , Université Lille1), Jérôme Thiébaut (2008-2010, Observatoire de Paris) et Alain Vienne (2008- , Université Lille1) . La réalisation technique est de Damien Guillaume (cellule TICE de l'UFE de l'Observatoire de Paris).

Ce site a été produit avec l'Unité Formation-Enseignement de l'Observatoire de Paris dans le cadre d’UNISCIEL et dans le cadre du dispositif CNL (Contenus Numériques en Ligne) - Université Lille1.

Contact : images/mail_alain.png


Introduction

introductionIntroduction

La philosophie de ce présent projet est d'utiliser l'Astronomie comme source d'exemples dans l'apprentissage des Mathématiques. On espère donc que ce sera un un moyen de renforcer l'attractivité des mathématiques pour les jeunes.

Le plan suit le programme de Mathématiques de Licence afin d'être plus accessible aux enseignants. Le site est structuré en applications comprenant chacune un ou plusieurs exercices avec les corrigés (ou des éléments de correction avec réponse).

Notons que ce site est toujours en cours d'enrichissement. Son objectif est de couvrir tout le parcours de Mathématiques et de Physique des étudiants de L1, L2 et L3.

Ce site a été produit avec l'Unité Formation-Enseignement de l'Observatoire de Paris dans le cadre d’UNISCIEL et dans le cadre du dispositif CNL (Contenus Numériques en Ligne) - Université Lille1.

Contact : images/mail_alain.png

logo_obs_ufe.pngjpg_Lille1-d3120.jpgjpg_Unisciel_logo-cf902.jpg

Les outils mathématiques en astronomie


Introduction

On trouvera dans cette section quelques rappels essentiels de Physique:

On pourra reconstruire et caractériser l'orbite de la planète de la planète Mars, en utilisant les données d'observation historiques de Tycho Brahé, ou s'intéresser au phénomène de précession du périastre.

Ces deux applications permettent de se familiariser avec l'importance des approximations en astronomie ou en physique en général.

A l'aide de développements, on pourra s'intéresser à la description épicyclique du mouvement keplerien ou à la notion de sphère d'influence.


Importance des approximations

Auteur: S. Renner

L'orbite de la planète Mars

Auteur: S. Renner

Date de création: 17 février 2009

L'objectif de cet exercice est d'établir les caractéristiques de l'orbite de la planète Mars, en utilisant les données d'observation de Tycho Brahé, celles-là mêmes qui furent utilisées par Kepler.

On suppose que les mouvements des planètes s'effectuent dans le même plan, qui est celui de l'écliptique. La position des planètes est alors repérée par une coordonnée, la longitude écliptique. On choisit pour origine des longitudes la direction du point vernal (noté \gamma). La tableau ci-dessous rassemble les données concernant 5 couples d'observations de Mars, effectuées par Tycho Brahé. On y indique la longitude écliptique géocentrique du Soleil, notée l_S (c'est-à-dire l'angle, mesurée depuis la Terre, entre la direction du Soleil et celle du point vernal), et la longitude écliptique géocentrique de Mars, notée l_M (angle vu de la Terre entre la direction de Mars et celle du point vernal).

DATE {\bf l_S} {\bf l_M}
1a 17/02/1585 339°23' 135°12'
1b 05/01/1587 295°21' 182°08'
2a 19/09/1591 185°47' 284°18'
2b 06/08/1593 143°26' 346°56'
3a 07/12/1593 265°53' 3°04'
3b 25/10/1595 221°42' 49°42'
4a 28/03/1587 16°50' 168°12'
4b 12/02/1589 333°42' 218°48'
5a 10/03/1585 359°41' 131°48'
5b 26/01/1587 316°06' 184°42'

Ex: L'orbite de la planète Mars

Auteur: S. Renner

exerciceOrbite de Mars

Difficulté :    Temps : 2h30

Question 1)

L'observation de Mars en opposition a fourni la période qui sépare ces oppositions, ou période synodique notée T_{syn}. Cette période est égale à 780 jours.

En déduire la valeur de la période sidérale de mars, que l'on notera T_M.

Question 2)

Calculer la durée qui sépare les 2 dates successives de chaque couple d'observations. Quelle conclusion peut-on en tirer?

Question 3)

Trouver la relation entre la longitude écliptique héliocentrique de la Terre L_T (angle entre la direction de la Terre et celle du point vernal mesurée depuis le Soleil) et la longitude géocentrique du Soleil l_S. Calculer L_T pour chacune des dates du tableau.

Question 4)

Représenter sur une feuille de papier millimétré l'orbite de la Terre par un cercle de centre S (Soleil) et de rayon égal à 5 cm. Choisir la direction du point vernal selon une des lignes du papier.

Pour chacun des 5 couples d'observations, construire:

  • la première position Ti de la Terre sur son orbite et la direction Tix de Mars telle qu'on l'observe depuis la Terre
  • la seconde position T'i de la Terre sur son orbite et la direction T'ix' de Mars

Montrer que le point Mi représentant Mars est à l'intersection des deux demi-droites Tix et T'ix'.

On déterminera ainsi les 5 positions de M1, M2, M3, M4 et M5 de Mars.

Question 5)

Vérifier que ces 5 points ne sont pas sur un cercle centré sur le Soleil.

Dans ce qui suit, on admet que l'orbite de Mars est une ellipse de faible excentricité, dont la forme ne diffère pas significativement de celle d'un cercle, mais dont le centre n'est pas le Soleil.

Pour déterminer le rayon de ce cercle, on partira d'une première approximation qui est la moyenne des 5 rayons SMi. On tracera le cercle ayant ce rayon sur une feuille de papier calque et on cherchera, par tâtonnement, à le faire passer au mieux parmi les 5 points Mi. Si le rayon du cercle est trop petit, la majorité des points se trouveront toujours à l'extérieur du cercle; inversement, s'il est trop grand, la majorité des points se trouveront toujours à l'intérieur. On modifiera donc le rayon de ce cercle, millimètre par millimètre, pour qu'il passe au mieux parmi les 5 points. Soit alors a la valeur du rayon de ce cercle.

Autre possibilité: connaissant la période sidérale de Mars, calculer son demi-grand axe en UA (unité astronomique, égale à la distance moyenne Terre-Soleil, soit 149 597 871 km). Tracer le cercle correspondant à cette orbite et estimer une erreur en mm sur les positions de Mars.

Question 6)

Mesurer la distance du centre C du cercle ainsi déterminé au point S (Soleil). En déduire l'excentricité de l'orbite e=CS/a.

Calculer la valeur du petit axe b de l'ellipse, et discuter la validité de l'approximation faite ici, qui a conduit à assimiler l'ellipse à un cercle de rayon a.

Question 7)

Sachant que l'excentricité de la Terre est 0.016, l'approximation de l'orbite de la Terre par un cercle centré sur le Soleil est-elle justifiée?

Question 8)

Quelle est la distance minimale de Mars à la Terre lors d'une opposition? A quelle date de l'année ces oppositions favorables se produisent-elles? Quelle est la distance maximale de Mars à la Terre lors d'une opposition? A quelle date de l'année ces oppositions défavorables se produisent-elles?

Question 9)

Afin de vérifier la loi des aires, mesurer l'aire balayée par le rayon-vecteur Soleil-Mars entre le 6 août et le 7 décembre 1593 et celle balayée entre le 26 janvier et le 28 mars 1587. En déduire la valeur moyenne de l'aire balayée par jour dans chacun des deux cas. On pourra, pour cela, compter les carreaux du papier.


Précession du périastre

Auteur: S. Renner

Date de création: 06 janvier 2011

L'objectif de cet exercice est de quantifier la précession du périastre d'un satellite en orbite autour d'un corps central légèrement aplati aux pôles.

Il est conseillé de s'intéresser au préalable à la résolution du problème à 2 corps.


Ex: Précession du périastre

Auteur: S. Renner

exercicePrécession du périastre

Difficulté :    Temps : 1h30

On considère le mouvement d'un satellite de masse $m$ par rapport à un corps central de masse  $M$ et de rayon $R$. On note $a$ le demi-grand axe du satellite, r sa distance au corps central, et $e$ son excentricité supposée faible (e<<1).

Sous l'effet de sa rotation, le corps central est légèrement aplati aux pôles. En conséquence, le potentiel gravitationnel pour un satellite évoluant dans le plan équatorial est donné par : $$\displaystyle V(r)= - \frac{G M}{r} \Big{(} 1 + \alpha \frac{R^2}{r^2}  \Big{)},$$\alpha est une constante <<1 qui caractérise l'applatissement. On peut montrer qu'alors l'équation du mouvement du satellite s'écrit: $$\displaystyle \frac{d^2}{d\theta^2} \Big{(} \frac{1}{r} \Big{)} + \frac{1}{r} = \frac{GM}{C^2} + \frac{3 \alpha GM R^2}{C^2 r^2} \equiv K_1 + \frac{K_2}{r^2},$$\theta est la position angulaire du satellite sur sa trajectoire (anomalie vraie), et C=r^2 \dot{\theta} le moment cinétique.

Question 1)

Comparer K_1 et \frac{K_2}{r^2}, et en déduire que la trajectoire du satellite est peu modifiée sous l'effet de l'applatissement du corps central, par rapport au problème à 2 corps classique.

Question 2)

Montrer que la solution de l'équation du mouvement peut s'écrire: $$\displaystyle r = \frac{q}{1+e \cos (1+ \epsilon) \theta},$$ avec \epsilon <<1, et exprimer q et \epsilon en fonction de $a$, $e$, $R$ et \alpha.

Question 3)

Quelle est alors l'allure de la trajectoire? Exprimer l'avance \Delta \theta du périastre de la trajectoire, c'est-à-dire l'angle dont les axes de l'ellipse ont tourné après une révolution du satellite.

Question 4)

Déterminer:

a) l'avance du périgée d'un satellite artificiel d'altitude h=1000 km autour de la Terre ($R= 6378$ km, \alpha=542 \times 10^{-6}).

b) l'avance du périastre du satellite Pan (a=133583 km et période orbitale  T=0.575 jours) autour de Saturne (R=60268 km, \alpha=8149 \times 10^{-6}).

c) l'avance du périhélie de Mercure (a=0.387 UA \simeq 58 \times 10^{6} km, T \simeq 88 jours) autour du Soleil (R=696000 km, \alpha = 10^{-5}). La précession observée du périhélie de Mercure s'élève en fait à 43''/siècle. Commenter.


Dimensions et unités

Auteur: Stéphane Erard

Dimensions et unités physiques

Les formules mathématiques utilisées en Astronomie définissent des relations entre grandeurs physiques. Ces grandeurs ont une dimension physique, et sont mesurées dans une certaine unité.

La dimension est inhérente à une grandeur physique, sa valeur est fonction de l'unité utilisée. On distingue sept types de grandeurs physiques, ou dimensions, indépendantes. Toute quantité physique peut s'exprimer comme combinaison de ces grandeurs de base.

Le système officiel en vigueur est le SI (Système International d'unités) ou MKSA, qui définit les unités de mesure des sept grandeurs indépendantes (voir par exemple la définition de la seconde). Différents systèmes d'unités ont été utilisés au cours de l'histoire, et d'autres systèmes sont en usage dans des domaines particuliers. En Astronomie, on utilise couramment des unités en rapport avec les phénomènes étudiés, par exemple l'unité astronomique, l'année-lumière, le parsec, ou le décalage vers le rouge pour les distances.

Table 1
Grandeur de base Dimension Unité S. I. Symbole S. I.
Longueur L mètre m
Masse M kilogramme kg
Temps T seconde s
Intensité de courant I Ampère A
Température Θ Kelvin K
Quantité de matière N mole mol
Intensité lumineuse J candela cd

Deux autres grandeurs sont utilisées en complément de celles-ci. Elles sont dépourvues de dimension physique (elles peuvent être comprises comme des rapports de longueurs ou de surfaces), mais peuvent s'exprimer dans différentes échelles. En pratique, on préfère l'échelle qui n'introduit pas de coefficient dans les fonctions trigonométriques (en radians, par opposition aux degrés pour les angles plats).

Table 2
Grandeur dérivée Dimension Unité S. I. Symbole S. I.
Angle plan 1 radian rad
Angle solide 1 stéradian sr

Ecrire une équation aux dimensions consiste à remplacer dans une formule les grandeurs par leurs dimensions et à négliger les coefficients de proportionnalité.

exempleExemple

La définition de la vitesse donne la dimension physique de cette grandeur :

v= \frac{dx}{dt}

L'équation aux dimensions est :

[V] = LT^{-1}

et la vitesse se mesure en m/s dans le Système International.


Equation aux dimensions

On peut toujours multiplier ou diviser des grandeurs quelconques entre elles, mais on ne peut additionner que des grandeurs physiques de même dimension — l'inverse reviendrait littéralement à additionner les torchons et les serviettes.

Les équations ou formules doivent donc être homogènes : chaque membre (et chaque terme) d'une équation doit avoir la même dimension physique. La vérification de l'homogénéité d'une formule ou d'un résultat de calcul doit être un réflexe en physique : c'est un moyen efficace pour éliminer les erreurs de calcul, et éviter les non-sens.

exempleExemple

Le principe fondamental de la dynamique donne la dimension physique de la force :

f = m \frac{d^2r}{dt^2}

L'équation aux dimensions est :

[F]= MLT^{-2}

et l'unité SI de la force est le kg\;m\;s^{-2} (couramment appelée Newton).

Les constantes qui apparaissent dans les lois physiques ont également une dimension. On peut dériver celle-ci en posant l'équation aux dimensions. La valeur numérique dépend encore une fois du système d'unités utilisé.

exempleExemple

L'attraction universelle (loi de Newton) s'écrit :

F= \frac{Gmm'}{r^2}

où G est la constante de gravitation. La dimension de G est donc

[G]= [F]L^2M^{-2}=L^3T^{-2}M^{-1}

La valeur numérique doit se mesurer expérimentalement, et dépend du système d'unités adopté.

Les fonctions mathématiques n'acceptent que des arguments sans dimension, ou dédimensionalisés. En pratique, ce sont des nombres purs ou des rapports de quantités de même grandeur.

exempleExemple

L'équation de Boltzmann donne la population d'un niveau d'énergie atomique ou moléculaire en fonction de la température :

N_i \propto e^{- \frac{E_i}{kT}}

La quantité kT est donc homogène à une énergie.

On peut en déduire la dimension physique de la constante de Boltzmann k :

Les équations aux dimensions permettent également de dériver les ordres de grandeur de phénomènes physiques, éventuellement en utilisant des modèles dérivés d'hypothèses simples.

exempleExemple

L'équation barométrique dérive d'un modèle basique d'atmosphère isotherme. Elle donne la pression P en fonction de l'altitude z sous ces hypothèses très simplifiées :

P(z) = P_0\; e^{-\frac{Mgz}{RT}}

P_0 est la pression au sol, M la masse molaire moyenne, R la constante des gaz parfaits, g l'accélération de la pesanteur, et T la température (supposée constante) de l'atmosphère.

La quantité h = RT/Mg est donc homogène à une distance — c'est l'altitude à laquelle la pression est réduite d'un facteur 2,7 dans ce modèle. Elle est appelée échelle de hauteur, et donne une estimation de l'épaisseur de la basse atmosphère des planètes.

On peut l'évaluer dans la troposphère terrestre :

M \sim 28 \ 10^{-3} kg \,  mol^{-1} (masse molaire de l'azote, principal constituant)

T ~ 280 K (température au sol)

g \simeq 10\; ms^{-2}

R \simeq 8\; JK^{-1}mol^{-1}

Ce qui donne pour l'échelle de hauteur h ~ 8 km.

De façon similaire, chaque grandeur possède une dimension tensorielle : scalaire, vecteur, ou tenseur d'ordre supérieur. La dimension tensorielle se préserve de la même façon que la dimension physique (chaque terme d'une équation doit avoir la même dimension tensorielle).

exempleExemple

Le principe fondamental de la dynamique peut s'écrire de manière vectorielle, et donne la direction de la force :

\vec{f} = m \frac{d^2\vec{r}}{dt^2}

On peut aussi l'écrire de manière scalaire en utilisant les normes :

f = m \frac{d^2r}{dt^2}

Les tenseurs d'ordre 2 sont utilisés pour décrire des quantités qui en chaque endroit dépendent aussi de la direction. Des exemples de tenseurs d'ordre 2 sont donnés par le tenseur métrique de la relativité générale, ou par les tenseurs de contrainte et de déformation en mécanique des milieux continus.


En savoir plus: Analyse dimensionnelle

L'analyse des dimensions d'un problème complexe permet de prédire la forme d'une loi physique, dans le cas fréquent où elle s'exprime comme produit des grandeurs qui interviennent.

Théorème de Vaschy-Buckingham

Si une loi physique s'écrit comme une relation entre n grandeurs indépendantes ayant k dimensions physiques indépendantes : \Psi(a_1;a_2;\dots;a_n) = 0

on peut l'exprimer comme une relation entre (n-k) nombres sans dimensions : \Phi(C_1;C_2;\dots;C_{n-k}) = 0

ceux-ci étant des produits de puissances des grandeurs de départ : C_i = \prod a_j^{\alpha_j}

En particulier, si (n-k) = 2 on peut toujours écrire C_1 = f(C_2)


Ex: analyse dimensionnelle

exerciceEx: analyse dimensionnelle

Difficulté :    Temps : 20 min

Question 1)

On étudie les oscillations d'un pendule à l'aide d'une simple analyse dimensionnelle. Enumérer les paramètres physiques qui interviennent dans ce problème.

Question 2)

Combien de grandeurs et de dimensions indépendantes interviennent dans le problème ? Combien de nombres sans dimension peut-on construire avec celles-ci ?

Question 3)

Dériver ces nombres sans dimension.

Question 4)

Ecrire une relation décrivant le problème. Commenter.


Obtention pragmatique de développements

Auteurs: Alain Vienne, S. Renner

Séries du problème des 2-corps

Auteur: Alain Vienne

Souvent, lorsque l'on considère le problème des 2 corps et quand l'excentricité e de l'orbite est petite, on a besoin de l'approximation suivante:

V=M+2e \sin M + \frac{5}{4}e^2 \sin 2M  +O (e^3)

\frac{r}{a}=(1+\frac{e^2}{2})-e\cos M - \frac{e^2}{2} \cos 2M  +O (e^3)

r et V sont les coordonnées polaires du corps, V étant compté à partir du péricentre (anomalie vraie). M est le temps ou plus précisément c'est l'anomalie moyenne M=\frac{2\pi}{T} (t-t_0) avec T la période, t le temps et t_0 l'instant de passage au péricentre. a et e sont respectivement le demi-grand axe et l'excentricité de l'orbite.

Dans l'exercice qui est proposé ci-après, on utilisera en plus des anomalies moyennes et vraies, l'anomalie excentrique. L'animation qui est donnée ici visualise leur évolution dans le cas d'une excentricité de 0,7.

Evolution des 3 anomalies (respectivement vraie, excentrique et moyenne)
figures/kepler.gif
Crédit : Astrophysique sur Mesure / Bessou

Si l'existence des développements ci-dessus est admise, l'astronome s'autrorise alors une démarche pragmatique pour les obtenir. Sa démarche n'a pas la rigueur du mathématicien. Dans le cas présenté ici, elle n'a pas non plus une grande efficacité si on souhaite "pousser" le développement plus loin en ordre.Elle a le seul avantage de pouvoir "se tirer d'affaire" dans le cas qui le préoccupe.

Dans l'exercice qui est proposé, on utilisera au besoin le développement de Taylor, l'intégration d'un développement, la substitution de développements.

complementExercices reliés

Pour la recherche de la solution du problème des 2-corps, on peut voir cet exercice. Il y aussi celui qui utilse le théorème de Lagrange. D'autres exercices sur le problème de 2 corps existent sur ce site. On en trouvera, entre autres, sur l'équation de Kepler et son inversion, sur les solutions géométriques du problème de 2 corps, sur le problème de 2 corps perturbé et sur l'excentricité limite dans les développements du problème de 2 corps.


Ex : Séries du problème des 2-corps

Auteur: Alain Vienne

exercice Séries du problème des 2-corps

Difficulté : ☆☆   Temps : 1h

introductionIntroduction

Quand on intègre le problème des 2-corps, la loi des aires permet d'écrire:

M=(1-e^2)^{3/2} \int_0^V \frac{dv}{(1+e \cos v)^2}

Question 1)

On utilisant cette relation et en négligeant les termes d'ordre supérieur ou égal à 3 en excentricité, montrer que l'on a:

M=V-2e \sin V +\frac{3}{4}e^2 \sin 2V + O(e^3)

Question 2)

On sait que \frac{r}{a} = 1 - e \cos E et M=E-e\sin E (E est l'anomalie excentrique). En déduire le développement de \frac{r}{a} en puissance de e et en fonction de M (limité à l'ordre 2)

Question 3)

A partir du développement obtenu à la première question, déduire celui qui donne V en fonction de M (limité à l'ordre 2 en e).

remarqueRemarque

Dans cette dernière question, le calcul est fait "en crabe", il faut donc veiller à la discussion sur l'ordre en excentricité. Plus généralement, tous ces calculs supposent l'existence des développements recherchés. Cette supposition et l'unicité ont permis d'éviter de se soucier des conditions d'application des théorèmes utilisés.


Séries du problème des 2-corps (Lagrange)

Auteur: S. Renner

Date de création: 10 avril 2013

Dans le problème des 2 corps, lorsque l'excentricité e de l'orbite est petite, on peut écrire :

V=M+2e \sin M + \frac{5}{4}e^2 \sin 2M  +O (e^3),

V est l'angle entre la direction du péricentre et la position du corps sur son orbite (anomalie vraie), M est le temps ou plus précisément l'anomalie moyenne M=\frac{2\pi}{T} (t-t_0), avec T la période, t le temps et t_0 l'instant de passage au péricentre.

L'animation donnée ci-après montre l'évolution des anomalies vraie et moyenne (et excentrique) dans le cas d'une excentricité e=0.7.

Evolution des 3 anomalies (respectivement vraie, excentrique et moyenne)
figures/kepler.gif
Crédit : Astrophysique sur Mesure / Bessou

Dans l'exercice proposé, on établit le développement ci-dessus à l'aide du théorème d'inversion de Lagrange.


Ex : Séries du problème des 2-corps (Lagrange)

Auteur: S. Renner

exercice Séries du problème des 2-corps (Lagrange)

Difficulté : ☆☆   Temps : 1h

introductionIntroduction

Quand on intègre le problème des 2-corps, la loi des aires permet d'écrire:

M=(1-e^2)^{3/2} \int_0^V \frac{dv}{(1+e \cos v)^2}

Question 1)

On utilisant cette relation et en négligeant les termes d'ordre supérieur ou égal à 3 en excentricité, montrer que l'on a:

M=V-2e \sin V +\frac{3}{4}e^2 \sin 2V + O(e^3)

Question 2)

Utiliser le théorème d'inversion de Lagrange pour montrer que V=M+2e \sin M + \frac{5}{4}e^2 \sin 2M  +O (e^3).


Description épicyclique du mouvement keplerien

Auteur: S. Renner

Date de création: 3 février 2010

Souvent en dynamique du système solaire, l'excentricité est très faible, et il est donc utile de considérer des approximations au premier ordre en excentricité, en particulier pour des systèmes vus dans des repères tournants. Cette approche est par exemple intéressante pour décrire la dynamique des anneaux planétaires, ou les effets de l'aplatissement d'une planète sur les orbites de satellites.

On propose ici un exercice qui porte sur la description du mouvement keplerien par des épicycles : le mouvement elliptique d'une particule P autour d'un foyer F est vu dans un repère centré sur un point G (le centre guide) en orbite circulaire uniforme autour de F (de rayon a égal au demi-grand axe de la particule, et de vitesse angulaire égale au moyen mouvement n=2 \pi/T, où T est la période orbitale de P).

L'exercice est largement inspiré du livre Solar System Dynamics (C.D. Murray & S.F. Dermott, 1999).


Ex: Description épicyclique du mouvement keplerien

Auteur: S. Renner

exerciceDescription épicyclique du mouvement keplerien

Difficulté : ☆☆   Temps : 2h

introductionIntroduction

Dans le problème à deux corps, on considère le mouvement d'une particule P sur une ellipse de foyer F et de demi-grand axe a. On note n le moyen mouvement de la particule, et f son anomalie vraie.

Soit G un point fictif tournant autour du foyer F sur une orbite circulaire de rayon a égal au demi-grand axe de P, avec une vitesse angulaire égale au moyen mouvement n de la particule. On note M l'angle entre la ligne FG et la direction du péricentre de la particule (c'est donc l'anomalie moyenne de P et on a n = dM/dt).

remarqueRemarque

Les questions 5 à 8 correspondent exactement à la représentation de Ptolémée du mouvement du Soleil autour de la Terre : un mouvement circulaire de rayon R=a avec la Terre au foyer F, et uniforme par rapport à F' (appelé l'équant). Le modèle de Ptolémée était donc du premier ordre en excentricité, et le triomphe de Kepler fut d'élaborer une théorie à l'ordre 2.

Question 1)

Ecrire les coordonnées x, y de P dans le repère orthonormé ($G$,{\bf \hat{x}},{\bf \hat{y}}) tel que {\bf \hat{x}}={\bf FG}/a.

Question 2)

A partir de la loi des aires, montrer que: \displaystyle dM= \frac{(1-e^2)^{3/2}}{(1+e \cos f)^2} df.

Question 3)

Ainsi en intégrant on obtient: \displaystyle M= (1-e^2)^{3/2} \int_0^{f} \frac{du}{(1+e \cos u)^2} . Montrer à l'aide de cette relation que \displaystyle f-M \simeq 2 e \sin M + O(e^2).

Question 4)

En déduire que par rapport à G, la particule P suit une orbite elliptique de demi-grand axe 2ae et de demi-petit axe ae. Dans quel sens s'effectue ce mouvement et avec quelle période?

Question 5)

Soit R la distance entre P et le centre O de l'ellipse. Montrer que \displaystyle R \simeq a (1 - \frac{1}{2}  e^2 \sin ^2 M).

Question 6)

Montrer qu'au premier ordre en excentricité, la trajectoire de P est alors un cercle centré sur O, et que l'angle \widehat{POF} est confondu avec l'anomalie excentrique E.

Question 7)

On note g l'angle \widehat{FF'P}, où F' est le second foyer (vide) de l'ellipse. Ecrire \cos g et en déduire que cos g \simeq \cos M + O(e^2). On rappelle que FP+F'P=2a, et on utilisera le développement: r/a \simeq 1 - e \cos M + (e^2/2)(1 \cos 2 M) (voir cet exercice).

Question 8)

Ainsi, le mouvement de la particule P, vu du foyer F', est uniforme avec une vitesse angulaire égale au moyen mouvement n. Que peut-on dire des droites F'P et FG?


Sphère d'influence

Auteur: S. Renner et A. Vienne

Date de création: 3 mars 2010

Nous allons définir la notion de sphère d'influence en s'intéressant au problème à trois corps Soleil + Jupiter + satellite, de masses respectives M>>m>>m' (qui constitue ce que l'on appelle un problème de Kepler perturbé).


Ex: Sphère d'influence

Auteurs: S. Renner, A. Vienne

exerciceSphère d'influence

Difficulté : ☆☆   Temps : 3h

introductionIntroduction

On considère le système gravitationnel formé du Soleil $S$ de masse M_\odot \equiv 1, de Jupiter J de masse $m$ petite devant 1 (m=1/1047) et d'un troisième corps A de masse m_{A} petite devant $m$ (par exemple un astéroïde ou une sonde spatiale). On note K la constante de gravitation universelle, \overrightarrow{r}=\overrightarrow{SJ}, \overrightarrow{p}=\overrightarrow{SA}, et \overrightarrow{\Delta}=\overrightarrow{p}-\overrightarrow{r}.

Question 1)

Ecrire les équations du mouvement héliocentrique de J (c'est-à-dire de \overrightarrow{r}).

Question 2)

Quelle est la nature du mouvement de J si la quantité m_{A} est négligeable?

Question 3)

On suppose désormais que le mouvement de J est circulaire (c'est-à-dire que m_A est négligeable et que r est constant). Montrer que l'équation du mouvement héliocentrique de A (c'est-à-dire de \overrightarrow{p}) est \frac{d^{2}\overrightarrow{p}}{dt^{2}}=\overrightarrow{A_{0}} + \overrightarrow{A_{1}}  \textrm{, avec }  \overrightarrow{A_{0}}=-K\frac{\overrightarrow{p}}{p^{3}} \textrm{ et }  \overrightarrow{A_{1}}=-Km \Big{(} \frac{\overrightarrow{\Delta}}{\Delta^{3}}+ \frac{\overrightarrow{r}}{r^3} \Big{)}.

Question 4)

Montrer que l'équation du mouvement jovicentique de A (c'est-à-dire de \overrightarrow{\Delta}) est \frac{d^{2}\overrightarrow{\Delta}}{dt^{2}}= \overrightarrow{B_{0}}+ \overrightarrow{B_{1}} \textrm{, avec } \overrightarrow{B_{0}}=-Km\frac{\overrightarrow{\Delta}} {\Delta^{3}} \textrm{ et } \overrightarrow{B_{1}}=-K \Big{(} \frac{\overrightarrow{p}}{p^{3}}-\frac{\overrightarrow{r}}{r^3} \Big{)}.

Question 5)

Dans la suite, on cherche à étudier la surface $(S)$ définie par \frac{\left\Vert \overrightarrow{A_{1}}\right\Vert } {\left\Vert \overrightarrow{A_{0}}\right\Vert } =\frac{\left\Vert \overrightarrow{B_{1}}\right\Vert } {\left\Vert \overrightarrow{B_{0}}\right\Vert }. On va voir notamment qu'elle a presque la forme d'une sphère, appelée sphère d'influence de Jupiter. Justifier cette appellation en expliquant ce qui se passe pour le mouvement de A lorsque celui-ci est respectivement très à l'intérieur ou très à l'extérieur de cette sphère.

Question 6)

On pose \displaystyle u=\frac{\Delta}{r} et \varphi=\textrm{angle}(\overrightarrow{JS},\overrightarrow{JA}). Montrer que l'on a A_{1}=\frac{Km}{\Delta^{2}}\sqrt{1-2u^{2}\cos\varphi+u^{4}}.

Question 7)

Montrer que l'on a B_{1}=\frac{K}{r^{2}}\sqrt{1+\Big{(}\frac{r}{p}\Big{)}^{4}-2\Big{(} \frac{r}{p}\Big{)}^{3} \frac{\overrightarrow{p}.\overrightarrow{r}}{r^{2}}} .

Question 8)

En exprimant \displaystyle \Big{(} \frac{p}{r} \Big{)}^{2} et \displaystyle  \frac{\overrightarrow{p}.\overrightarrow{r}}{r^{2}} en fonction de \varphi et u, montrer que B_{1}=\frac{K}{r^{2}}\frac{\sqrt{1+(1-2u\cos\varphi+u^{2})^{2} -2(1-u\cos\varphi)\sqrt{1-2u\cos\varphi+u^{2}}}}{1-2u\cos\varphi+u^{2}}.

Question 9)

En déduire que l'équation de la surface (S) peut s'écrire m^{2}=f(u,\varphi), et donner l'expression de f(u,\varphi).

Question 10)

Puisque m est petit devant 1, on peut considérer que, sur la surface (S), u est également petit. Montrer que le développement de f(u,\varphi) suivant les puissances de u, limité à son terme de plus bas degré vaut \displaystyle u^{5}\sqrt{1+3\cos^2\varphi}. Justifier alors le mot sphère dans l'expression sphère d'influence qui désigne la surface (S).

Question 11)

Calculer le rayon de la sphère d'influence de Jupiter sachant que $r=5.2$ UA.


Les réels et les suites


Introduction

introductionIntroduction

On trouvera dans cette partie les exercices suivants :


Suites réelles

Auteur: Marc Fouchard

Equation de Kepler elliptique

Marc Fouchard

Le but de cet exercice est de résoudre l'équation de Kepler par la méthode de Newton.

L'équation de Kepler est:

E-e\,\sin E -M =0

E s'appelle l'anomalie excentrique, M l'anomalie moyenne et e l'excentricité. Dans le cas présent on a 0\le e < 1, M\in [0;2\pi] et E\in[0;2\pi]. Dans l'exercice on va se limiter à l'intervalle [0;\pi]. On peut facilement en déduire la résolution de l'équation dans l'intervalle [\pi;2\pi] par symétrie. La figure ci-dessus montre le lien entre ces anomalies.

Lien entre les 3 anomalies
kepler.gif
Crédit : Astrophysique sur Mesure / Bessou Fouchard

On peut voir l'exercice suivant pour voir des méthodes plus complexes de résolution de l'équation de Kepler. Le cas hyperbolique fait l'objet de cet exercice.


Ex: equation de Kepler elliptique

Auteur: Marc Fouchard

exerciceéquation de Kepler elliptique

Difficulté :    Temps : 30 mn

Question 1)

Soit f la fonction définie par f(x)=x-e\sin x -M, avec M \in [0;2\pi] une constante. Montrer que f est continue, dérivable deux fois et que f'(x)>0 et pour x\in [0;\pi] , f''(x)\ge 0.

Question 2)

Soit r le nombre réel tel que f(r)=0 (on peut montrer rapidement que r existe d'après la continuité de f et le théorème de la valeur intermédiaire). Montrer que pour x\in]r;\pi], f est strictement positive.

Question 3)

Montrer que la courbe représentative de f est au dessus de sa tangente sur [r;\pi].

Question 4)

En déduire que la suite \left\lbrace u_n \right\rbrace définie par:

u_{n+1}=u_n-\frac{f(u_n)}{f'(u_n)},

avec \pi\ge u_0 \ge r, est décroissante et minorée par r.

Question 5)

En déduire que la suite \left\lbrace u_n \right\rbrace converge et que sa limite est r.


Fonction d'une variable réelle : généralité et continuité


Introduction

On trouvera dans ce chapitre les exercices suivants :


Continuité

Auteur: Marc Fouchard

Loi de Wien

Auteur : Marc Fouchard

La loi de Planck montre que pour un corps noir, l'énergie émise par rayonnement suivant un longueur d'onde, ne dépend que de la température de surface du corps noir. Cette loi est donnée par la relation suivante :

E(\lambda)=\frac{2 h c^2}{\lambda ^5}\cdot \frac{1}{\exp \left( \frac{h c}{k \lambda T} \right) -1}c correspond à la vitesse de la lumière dans le vide, hest la constante de Planck, kla constante de Boltzmann, \lambdala longueur d'onde à laquelle le rayonnement est émis et Tla température de surface du corps noir.

La figure ci dessous montre le comportement de E(\lambda) pour différentes températures de surface du corps noir. On peut remarquer que le maximum de la courbe se déplace sur la gauche lorsque la température augmente. Autrement dit, la longueur d'onde \lambda_{\rm max} pour laquelle le rayonnement émis est maximal diminue lorsque la température de surface augmente.

Loi de Planck application.png

Le but de cet exercice est de trouver la relation exacte entre \lambda_{\rm max} et T.

remarqueRemarque

Cette exercice repose sur la détermation du maximum d'une fonction sur un intervalle donné. L'exercice utilise aussi le théorème du point fixe dans \mathbf R, mais ce théorème peut être admis ici.


Ex : loi de Wien

Auteur: Marc Fouchard

exerciceLoi de Wien

Difficulté : ☆☆   Temps : 1h

Question 1)

Sachant que h, c et ksont des constantes strictement positives et que la température Tétant mesurée en degré Kelvin est aussi strictement positive, montrer que E(\lambda) est de classe {\mathcal C}^{\infty} et est toujours strictement positive lorsque \lambda \in \left\rbrack 0 , + \infty \right \lbrack.

Question 2)

Montrer que les limites de E(\lambda) quand \lambdatend vers 0 et vers + \inftysont toutes les deux égales à zéro.

Question 3)

En déduire qu'il doit exister un maximum pour E(\lambda) sur \left\rbrack 0 ; +\infty \right\lbrack.

Question 4)

En effectuant le changement de variable u=\frac{h c}{k \lambda T}, montrer qu'étudier le signe de \frac{{\rm d} E(\lambda)}{{\rm d} \lambda}revient à étudier celui de \frac{{\rm d} E(u)}{{\rm d} u}.

Question 5)

En déduire une condition sur u, de la forme f(u)=u, pour que \frac{{\rm d} E(u)}{{\rm d} u} s'annule. On note u_S la solution de cette équation lorsqu'elle existe.

Question 6)

On peut monter par le théorème du point fixe dans {\mathbf R} que fadmet un point fixe et que la suite définie par u_{n+1} = f(u_n)converge vers ce point fixe (voir Loi de Wien et théorème du point fixe). En prenant u_0=5 trouver une valeur \tilde{u}_S qui soit une valeur approchée de u_S à 10^{-7} prêt.

Question 7)

En déduire la relation \lambda_{\rm max}\cdot T =Ac=299\,792\,458~ {\rm m}\cdot{\rm s}^{-1}, h = 6,626\,17\times 10^{-34}~{\rm J}\cdot{\rm s} etk = 1,380\,66 \times 10^{-23}~{\rm J}\cdot{\rm K}^{-1}. Cette relation correspond à la loi de Wien pour les corps noirs. Justifier l'utilisation de \tilde{u}_S dans le calcul de la constante A.


Fonction d'une variable réelle : dérivabilité


Introduction

On trouvera dans cette partie les exercices suivants :


Définition

Auteurs: Marc Fouchard, S. Renner, Stéphane Erard

Rétrogradation de Mars

Auteur : Marc Fouchard

Le mouvement de Mars vu depuis la Terre montre des périodes pendant lesquelles Mars se déplace dans le sens inverse au Soleil par rapport au fond d'étoiles fixes. L'exercice présenté ici consiste à étudier cette phase de rétrogradation de Mars.

L'animation ci-dessous montre à gauche le mouvement de la Terre et de Mars autour du Soleil et à droite les mêmes mouvements vus depuis la Terre.

Retrogradation de Mars
retrogradation_de_Mars.gif
Mouvement de la Terre et de Mars autour du Soleil.
Crédit : Astrophysique sur Mesure / Bessou Fouchard

On remarque que vu de la Terre le mouvement de Mars se fait dans le sens inverse (sens retrograde) à celui du Soleil (sens prograde). Le but de cet exercice est d'étudier cette phase de rétrogradation.


Ex : rétrogradation de Mars

Auteur: Marc Fouchard

exerciceRétrogradation de Mars

Difficulté : ☆☆   Temps : 1h

On suppose que la Terre T et Mars M se déplacent uniformément sur des cercles centrés sur le Soleil S. Soit r_{\rm T} et r_{\rm M} les rayons respectives des orbites de la Terre et de Mars, et \omega_{\rm T}, \omega_M leur vitesses angulaires respectives. On suppose que les plans de l'orbite de la Terre et de Mars sont confondus. Ainsi, dans un repère fixe centré sur le Soleil on note (x_{\rm T},y_{\rm T}) et (x_{\rm M}, y_{\rm M}) les coordonnées respectives de la Terre et de Mars. On suppose qu'initialement le Soleil, la Terre et Mars sont alignés dans cet ordre sur l'axe des abscisses du côté des abscisses positives.

Question 1)

Exprimer les coordonnées de la Terre et de Mars en fonction du rayon de leur orbite, de leur vitesse angulaire et du temps t. Soit (D\cos \lambda , D\sin \lambda) les coordonnées du vecteur \overrightarrow{TM}. En déduire une expression de D et de \lambda en fonction de r_{\rm T}, r_{\rm M}, \omega_{\rm M}, \omega_{\rm T} et t.

Question 2)

Calculer la dérivée de \lambda par rapport au temps, puis déterminer son signe pour \omega_{\rm M}t-\omega_{\rm T}t=0^\circ et 180^\circ. On utilisera la propriété \omega_T^2 r_T^3=\omega_M^2 r_M^3 qui dérive de la troisième loi de Kepler. Conclure.

Question 3)

Cacluler la valeur de \omega_{\rm M}t-\omega_{\rm T}t lorsque \frac{{\rm d} \lambda}{{\rm d} t} s'annule. Ces positions correspondent aux stations de Mars. On notera dans la suite t_s un instant conrrespondant à une station.

Question 4)

Calculer les deux instants correspondant aux stations en fonction de \alpha=\sqrt{\frac{r_M}{r_T}}. En déduire la durée de la rétrogradation.


Orbites perturbées du problème à 2 corps

Auteur: S. Renner

Date de création: 04 avril 2011

L'objectif de cet exercice est de déterminer quels types de forces perturbatrices peuvent modifier le demi grand-axe ou l'excentricité d'une orbite.

Il est nécessaire de s'intéresser au préalable à la résolution du problème à 2 corps.


Ex: Orbites perturbées du problème à 2 corps

Auteur: S. Renner

exerciceOrbites perturbées du problème à 2 corps

Difficulté :    Temps : 1h

Un corps en orbite elliptique autour du Soleil (de rayon vecteur r) est soumis à une force perturbatrice de la forme d {\bf F} = F_r {\bf u_r} +  F_\theta {\bf u_\theta} + F_z {\bf u_z}, où F_r, F_\theta, F_z sont respectivement les composantes (constantes) radiale, tangentielle et normale de la force, et {\bf u_r} = {\bf r}/r, {\bf u_\theta}, {\bf u_z} des vecteurs orthonormés unitaires. Cette force est suffisamment faible pour que la trajectoire de l'objet reste keplerienne.

Question 1)

On peut écrire la variation d'énergie {\dot E}={\dot \bf r}.d{\bf F} due à la force d {\bf F}. Sachant de plus que  {\dot E}= \frac{\mu}{2a^2} {\dot a} avec \mu = G (M+m), montrer quels types de forces vont modifier le demi-grand axe a en calculant da/dt .

Question 2)

En écrivant la variation du moment cinétique {\dot h} due à d {\bf F}, montrer quels types de forces modifient l'excentricité en calculant de/dt .


Loi de Planck en fréquence

Auteur: Stéphane Erard

Date de création: 07 mars 2013

La loi de Planck donnant le spectre du corps noir est souvent donnée en fonction de la longueur d'onde. L'objectif de cet exercice est de dériver cette loi en fonction de la fréquence du rayonnement. Cette expression est plus naturellement utilisée dans certains domaines, en particulier aux basses énergies (domaine radio) et aux basses températures (dans le milieu interstellaire par exemple).


Ex: loi de Planck en fréquence

Auteur: Stéphane Erard

exerciceLoi de Planck en fréquence

Difficulté :    Temps : 30 min

On connaît la luminance du corps noir en fonction de la longueur d'onde, donnée par la loi de Planck (voir par exemple l'exercice sur la loi de Wien) : B_{\lambda} = \frac{2hc^2}{\lambda^5 (e^{hc/kT\lambda} -1) }

c est la vitesse de la lumière dans le vide, h la constante de Planck, k la constante de Boltzmann, \lambda la longueur d'onde et T la température du corps noir.

Cette expression est une luminance directionnelle, donnée habituellement en W\,m^{-2}\,sr^{-1}\,\mu m^{-1}.

Question 1)

Donner l'expression de cette luminance en fonction de la fréquence du rayonnement.

Question 2)

Comparer les graphiques de ces deux expressions en échelle linéaire.


Théorème des accroissements finis

Auteurs: Alain Vienne, Marc Fouchard

Les racines du polynôme de la méthode de Laplace

Auteur: Alain Vienne

Lors de la découverte d'un nouvel objet dans le système solaire, on souhaite rapidement connaitre sa trajectoire. Celle-ci est généralement héliocentrique et, dans un premier temps, on la suppose képlérienne. Or les observations terrestres donnent uniquement la direction de l'astre mais pas sa distance. La méthode de Laplace propose un moyen qui, à partir de 3 observations de direction faites à des dates assez rapprochées, donne les vecteurs position et la vitesse de l'astre. Le détail de la méthode peut être trouvé dans le cours suivant: Dynamique du système solaire. On peut y voir notamment que la méthode conduit à chercher les racines d'un polynôme de degré 8. Il y est affirmé qu'il y a 4 racines réelles (1 négative, 3 positives) et 4 complexes non réelles. C'est cette affirmation qui est étudiée dans l'exercice qui suit.

Le polynôme est de la forme:

f(r)=r^8 + a_6 r^6 + a_3 r^3 + a_0

On sait que a_0 < 0 et qu'il existe au moins 2 solutions distinctes strictement positives. L'une des deux est r=SS est la distance Terre-Soleil. On peut aller voir l'exercice qui vérifie cette racine ici. La deuxième solution distincte de S et strictement positive suppose que les 3 observations ont été bien faites et correspondent physiquement à un même objet du système solaire. Elle n'est pas garantie mathématiquement mais s'appuie sur l'argument que cette solution "doit exister".

Voici à titre d'exemple le graphe du polynôme dans le cas de 3 observations de Jupiter à son opposition (courbe "complète" et un agrandissement):

lagrange_typ.pnglagrange_typ_agr.png
Le polynome de la méthode de Laplace dans le cas de 3 observations de Jupiter (courbe "complète" et un agrandissement). L'axe horizontal est gradué en ua. On note que ce polynôme n'est pas très bien conditionné car la vue d'ensemble ne donne pas une idée des racines ni même du nombre de ces racines. La deuxième figure est agrandissement sur la partie utile. On voit la racine r=1 ua (S) et les 2 autres racines dont celle qui nous intéresse à 5 ua.
Crédit : Astrophysique sur Mesure / Vienne

Ex: Les racines du polynôme de la méthode de Laplace

Auteur: Alain Vienne

exerciceLes racines du polynôme de la méthode de Laplace

Difficulté : ☆☆   Temps : 1h

Soit un polynôme à coefficients réels de la forme:

f(r)=r^8 + a_6 r^6 + a_3 r^3 + a_0

On sait que a_0 < 0 et qu'il existe au moins 2 solutions distinctes strictement positives.

Question 1)

Calculer f'(r) et étudier le polynôme g(r)=\frac{f'(r)}{r^2} dans le cas où a_6 \ge 0

Question 2)

En déduire que a_6 est strictement négatif

Question 3)

Montrer que g s'annule en un point \beta positif

Question 4)

g'(r) peut donc s'écrire g'(r)=40 r^2 (r-\gamma ) (r+\gamma ) (avec \gamma positif). Monter que g(\gamma ) \le 0.

Question 5)

Monter que g(- \gamma ) > 0.

Question 6)

Montrer que g s'annule en \beta_1, \beta_2 et \beta_3 tels que \beta_1<-\gamma < \beta_2 < + \gamma < \beta_3

Question 7)

Etudier les 2 cas \beta_1 < \beta_2 < 0 < \beta_3 et \beta_1 < 0 <  \beta_2  < \beta_3. Monter que le premier cas est impossible et que le deuxième cas conduit à une ou trois racines positives

Question 8)

Conclure.


Les racines des polynômes de Legendre

Auteur: Alain Vienne

En Mécanique Céleste, on est souvent conduit à utiliser les polynômes de Legendre que l'on note ici P_n .

C'est le cas, par exemple, dans le développement du potentiel terrestre. Si on suppose que la Terre est un sphéroïde, le potentiel peut s'écrire:

U(r,\varphi)=\frac{KM_T}{r} \ [1 - \sum_{m=1}^{\infty} J_{2m} (\frac{a_e}{r})^{2m}  P_{2m}(\sin\varphi) \ ]

K est la constante de gravitation de la Terre, M_T la masse totale de la Terre, a_e son rayon équatorial et J_{2m} des coefficients numériques. r et \varphi sont le rayon et la latitude du point pour lequel on évalue le potentiel U .

Un autre exemple d'utilisation est de considérer 2 corps M et M' décrivant autour d'un centre P des orbites proches d'un mouvement elliptique. Pour décrire les perturbations (gravitationnelles) entre M et M', on doit écrire l'inverse de la dsitance entre M et M', 1/\Delta, en fonction de leurs éléments d'orbite. On montre facilement que:

\frac{1}{\Delta} = \frac{1}{r'} (1-2\rho \cos S + \rho^2)^{-1/2}

Avec r=PM, r'=PM', \rho = r/r' et S l'angle entre M et M' vu de P.

Cette dernière expression est développée en puissance de \rho grâce aux polynômes de Legendre:

(1-2\rho \cos S + \rho^2)^{-1/2} =  \sum_{n=0}^{\infty} \rho^n  P_n(\cos S)

Ce développement est rapidement convergent si \rho est petit. C'est le cas si, par exemple, M est la Terre, P le Soleil et M' un satellite artificiel.

Plus de détails de ces développement peuvent être vus dans le cours de Mécanique Céleste de Luc Duriez.

Les polynômes de Legendre ont de nombreuses propriétés. Celle que nous allons utiliser dans l'exercice qui suit est la formule de Rodrigues:

P_m(x)=\frac{1}{2^m \  m!} \  \frac{d^m}{dx^m} (x^2-1)^m

Cette formule va nous permettre de montrer que l'équation P_m(x)=0 a toutes ses racines dans [-1,+1] et en a m distinctes.


Ex: Les racines des polynômes de Legendre

Auteur: Alain Vienne

exerciceLes racines des polynômes de Legendre

Difficulté : ☆☆   Temps : 1h (pour une rédaction correcte)

Les polynômes de Legendre, bien connus en Mécanique Céleste, peuvent se déterminer par la formule de Rodrigues:

P_m(x)=\frac{1}{2^m \  m!} \  \frac{d^m}{dx^m} (x^2-1)^m

Question 1)

Montrer que l'équation P_m(x)=0 a toutes ses racines dans [-1,+1] et en a m distinctes.


Projection de Mollweide

Auteur: Marc Fouchard

La projection de Mollweide est la projection d'une sphère sur un plan qui conserve les aires au sacrifice de la conservation des distances et des formes. La projection d'une sphère rempli une ellipse dont le petit axe est le double du grand axe.

L'avantage d'une telle projection en astronomie est qu'elle permet d'avoir une idée globale de la répartition d'une certaine quantité sur la sphère céleste par unité de surface (ou stéradian). Par exemple, l'image suivante montre la répartition du rayonnement du fond cosmologique sur une projection de Mollweide de la sphère céleste. Comme ce rayonnement est mesuré par unité de surface (ou par stéradian), la conservation des aires est ici fondamentale pour bien visualiser les données.

Le rayonnement du fond cosmologique par WMAP
figures/ilc_9yr_moll1024.png
Répartition du rayonnement du fond cosmologique observé par le satellite WMAP (l'échelle de couleur est entre +/- 200 micro Kelvin par rapport à une valeur moyenne) sur une projection de Mollweide de la sphère céleste. Le plan de référence est celui de la Voie Lactée dont le rayonnement a été soustrait.
Crédit : NASA/WMAP

Sur une sphère, on défini un système de coordonnées en choisissant un plan de référence (par exemple l'équateur), à partir duquel seront mesurées les latitudes, notées \phi, un méridien de référence (par exemple le méridien de Greenwich) à partir duquel sont mesurées les longitudes, notées \lambda. Pour chaque angle un sens positif est défini (par exemple vers le nord pour les latitudes et vers l'ouest pour les longitudes).

Les coordonnées (x,y) par la projection de Mollweide d'un point de coordonnées (\lambda,\phi) de la sphère céleste sont définies par:

\begin{array}{lcr} x&=&\frac{2\sqrt{2}}{\pi}\lambda \cos \theta, \\ y&=&\sqrt{2}\sin \theta, \end{array}

où la longitude \lambda est mesurée entre -\pi et \pi et \theta est un angle auxiliaire défini par :

2\theta + \sin (2\theta) = \pi \sin  \phi. (*)

L'équation (*) ne peut être résolue analytiquement. Le but de cet exercice est de trouver une méthode permettant de déterminer \theta afin de pouvoir calculer x et y.


Ex : projection de Mollweide

Auteur: Marc Fouchard

exerciceprojection de Mollweide

Difficulté : ☆☆   Temps : 1h30

Question 1)

Nous allons commencer par étudier la fonction:

f (\theta)=\arcsin \left( \frac{2\theta+\sin(2\theta)}{\pi}\right),

sur l'intervalle I=[-\pi/2,\pi/2].

Montrer que f est définie et continue sur I et qu'elle est impaire.

Question 2)

Montrer que f est dérivable sur ]-\pi/2, \pi/2[, puis en prolongeant par continuité sur I.

Question 3)

En déduire que f est strictement croissante de I dans lui-même et donc qu'il existe une fonction réciproque, notée g. Déterminer les propriétés principales de g et en particulier que g([0,\pi/2])=[0,\pi/2]

Question 4)

On remarque que pour \phi donné, g(\phi) est la solution de l'équation (*).

En déduire l'image des points, ou des ensembles de points suivants, par la projection de Mollweide :

  1. points de latitude nulle.
  2. pôles.
  3. deux méridiens de longitude \lambda = \pm \pi/2 .
  4. deux méridiens de longitude \lambda = \pm \pi .
  5. parallèle de latitude \phi.

Question 5)

La fonction g n'est pas définie analytiquement. On va montrer ici qu'on peut estimer g(\phi) par la méthode de Newton-Raphson. Soit h_\phi la fonction définie sur [-\pi/2,\pi/2] par :

h_\phi(\theta)=2\theta+\sin(2\theta)-\pi\sin \phi.

Résoudre l'équation (*) revient donc à résoudre h_\phi(\theta)=0.

Montrer que si h_\phi(\theta)=0 alors h_{-\phi}(-\theta)=0 et que si \phi\in[0,\pi/2] alors la solution de h_\phi(\theta)=0 est dans [0,\pi/2]. De même montrer que h_0(0)=0 et h_{\pi/2}(\pi/2)=0.

Question 6)

Ainsi on peut se limiter à résoudre h_\phi(\theta)=0 pour \phi\in]0,\pi/2[. On sait déjà que la solution se trouve dans ]0,\pi/2[ d'après la question 3. Montrer que, pour \phi\in]0,\pi/2[, h est définie continue dérivable et strictement croissante sur [0,\pi/2[ et que sa dérivée est définie continue, strictement positive et strictement décroissante sur [0,\pi/2[.

Question 7)

Soit \theta_S la solution de h_\phi(\theta)=0. Soit \theta_0 \in [0,\theta_S[. On note S(\theta_S,0) et A_0(\theta_0,h(\theta_0)) les points de la courbe représentative \mathcal{H} de h dans un repère orthonormé. On note B_1(\theta_1,0), le point d'intersection de la tangente en A_0 à \mathcal{H} avec l'axe des abscisses. Montrer que \theta_1=\theta_0-\frac{h_\phi(\theta_0)}{h_\phi'(\theta_0)} et que \theta_0 < \theta_1 < \theta_S.

Question 8)

Montrer que la suite définie par \theta_0=0 et \theta_{n+1}=\theta_n-\frac{h_\phi(\theta_n)}{h_\phi'(\theta_n)} converge vers \theta_S.

ensavoirplusConvergence de la méthode de Newton-Raphson

La convergence de cette suite dépend fortement du choix de \theta_0. La figure ci-dessous montre le nombre d'itérations nécessaires pour attendre un précision relative de l'ordre de 10^{-12} sur \theta_S en fonction de \theta_0 et \phi. On voit que pour certains choix la convergence est très mauvaise voire impossible. Par contre avec \theta_0=0, la méthode converge pour toute valeur de \phi même si ce n'est pas le choix optimal.

Convergence de la méthode de Newton-Raphson pour la projection de Mollweide
figures/test_conv.png
Nombre d'itérations nécessaires pour attendre un précision relative de l'ordre de 10^{-12} sur \theta_S en fonction de \theta_0 et \phi.
Crédit : Astrophysique sur Mesure / Marc Fouchard.


Extrema

Auteurs: Marc Fouchard, Jérôme Thiébaut

Loi de Wien

Auteur : Marc Fouchard

La loi de Planck indique que pour un corps noir, l'énergie émise par rayonnement à une longueur d'onde donnée, ne dépend que de la température de surface du corps noir. Cette loi est donnée par la relation suivante :

E(\lambda)=\frac{2 h c^2}{\lambda ^5}\cdot \frac{1}{\exp \left( \frac{h c}{k \lambda T} \right) -1}

c correspond à la vitesse de la lumière dans le vide, h est la constante de Planck, k la constante de Boltzmann, \lambda la longueur d'onde à laquelle le rayonnement est émis et T la température de surface du corps noir.

La figure ci dessous montre le comportement de E(\lambda) pour différentes températures de surface du corps noir. On peut remarquer que le maximum de la courbe se déplace sur la gauche lorsque la température augmente. Autrement dit, la longueur d'onde pour laquelle le rayonnement émis est maximal diminue lorsque la température de surface augmente.

application.png

Le but de cet exercice est de trouver la relation exacte entre \lambda_{\rm max} et T.

remarqueRemarque

Cette exercice repose sur la détermation du maximum d'une fonction sur un intervalle donné. Il utilise aussi le théorème du point fixe dans \mathbf R, mais ce théorème peut être admis ici.


Ex : loi de Wien

Auteur: Marc Fouchard

exerciceLoi de Wien

Difficulté : ☆☆   Temps : 2h

Question 1)

Sachant que h, c et k sont des constantes strictement positives et que la température T étant mesurée en Kelvin est aussi strictement positive, montrer que E(\lambda) est de classe \mathcal{C}^{\infty} sur ]0,+\infty[ et est toujours strictement positive sur cet intervalle.

Question 2)

Montrer que les limites de E(\lambda) quand \lambda tend vers 0 et vers + \infty sont toutes les deux égales à zéro. Ce résultat peut être admis ici.

Question 3)

En déduire qu'il doit exister un maximum pour E(\lambda) sur \left\rbrack 0 ; +\infty \right \lbrack.

Question 4)

En effectuant le changement de variable u=\frac{hc}{k T \lambda }, montrer qu'étudier le signe de \frac{{\rm d} E(\lambda)}{{\rm d} \lambda} \frac{{\rm d} E(\lambda)}{{\rm d} \lambda}revient à étudier celui de \frac{{\rm d} E(u)}{{\rm d} u}.

Question 5)

En déduire une condition sur u, de la forme f(u)=u, pour que \frac{{\rm d} E(u)}{{\rm d} u} s'annule. On note u_S la solution de cette équation lorsqu'elle existe.

Question 6)

On peut monter par le théorème du point fixe dans {\mathbf R} que fadmet un point fixe et que la suite définie par u_{n+1} = f(u_n) converge vers ce point fixe (voir Loi de Wien et théorème du point fixe). En prenant u_0=5 trouver une valeur \tilde{u}_S qui soit une valeur approchée de u_S à 10^{-7} prêt.

Question 7)

En déduire la relation \lambda_{\rm max}\cdot T =Ac=299\,792\,458~ {\rm m}\cdot{\rm s}^{-1}, h = 6,626\,17\times 10^{-34}~{\rm J}\cdot{\rm s} etk = 1,380\,66 \times 10^{-23}~{\rm J}\cdot{\rm K}^{-1}. Cette relation correspond à la loi du déplacement de Wien pour les corps noirs. Justifier l'utilisation de \tilde{u}_S dans le calcul de la constante A.


Etoile à neutron

Auteur : Jérôme Thiébaut

Une étoile à neutron constitue l'étape ultime d'évolution des étoiles de masses inférieures à trois masses solaires. Ayant brulé tout son carburant, l'étoile devient une supernova, elle éjecte ses couches extérieures et son coeur s'éffondre sur lui même. Les électrons et les protons fusionnent ensemble et se transforment en neutrons. La densité devient alors comparable à celle de la matière nucléaire et la température est de l'ordre de 10^8K. Le but de cet exercice est de déterminer grâce à un modèle simple le rayon d'équilibre de ces étoiles.


Ex: Etoile à neutron

Auteur: Jérôme Thiébaut

exerciceEtoile à neutron

Difficulté :    Temps : 45 min

On assimile l'étoile à neutron à un gaz parfait de neutrons contenu dans une sphère. La densité d'états (ou fonction de distribution) de l'impulsion rho(p) est la suivante: rho*(p) dp = frac(V;pi^2*planck^3)*p^2*dp, où V est le volume et planckla constante de Planck réduite.

Question 1)

On définit la densité de particule n: n=frac(1;V)*intégrale(rho(p);p;0;p_f)p_fest l'impulsion de Fermi, c'est à dire l'impulsion maximale. Exprimer n en fonction de p_f.

Question 2)

Exprimer l'impulsion de Fermi en fonction de la masse du neutron, m, celle de l'étoile, M, et du rayon de l'étoile, R.

Question 3)

On définit la densité d'énergie, epsilon: epsilon=frac(1;V)*intégrale(frac(p^2;2*m)*rho(p);p;0;p_f). calculer epsilon en fonction de p_f.

Question 4)

L'énergie du gaz E vallant frac(4;3)*pi*R^3*epsilon, l'exprimer en fonction de l'impulsion de Fermi puis en fonction des caractéristiques de l'étoile et de la masse du neutron.

Question 5)

L'énergie gravitationnelle de l'étoile est E_g=-frac(3*G*M^2;5*R) , où G est la constante de gravitation. L'équilibre est atteint lorsque l'énergie totale (celle du gaz plus celle gravitationnelle) de l'étoile est minimum. Calculer le rayon R_étoile qui minimise l'énergie en fonction de la masse de l'étoile et des constantes m, G et planck. Calculer ce rayon pour le soleil.


Sphère photonique

Auteur: Jérôme Thiébaut

En relativité générale la gravitation n'est pas une force mais une déformation de l'espace temps due aux corps qu'il contient. Par conséquent, la métrique, c'est à dire la manière de mesurer les distances, s'en trouve transformée par rapport aux distances euclidiennes usuelles. La métrique de Schwarzschild est une métrique applicable à un corps massif central statique ou en rotation lente. Cette métrique s'applique à l'extérieur du corps en question et n'est plus valable en son sein. Elle permet de plus, à grande distance ou dans le cas de potentiel faible, de retrouver la gravitation newtonnienne. Le but de cet exercice est de montrer que pour des trous noirs, il existe une orbite circulaire en deça de laquelle il est impossible d'orbiter, et que cette orbite correspond à la trajectoire de photons.


Ex: Sphère photonique

Auteur: Jérôme Thiébaut

exerciceSphère photonique

Difficulté :    Temps : 20mn

La métrique de Schwarzschild est une métrique applicable à un corps massif central statique ou en rotation lente. Elle s'écrit: ds^2=-psi*c^2*dt^2+dr^2/psi+r^2*d*theta^2+r^2*sin(theta)^2*d*phi^2, où t est le temps,c la vitesse de la lumière, r theta phisont les coordonnées sphériques et psi est défini comme psi=1-r_s/r . Le rayon de Schwarzschild r_s est directement relié à la masse du corps central par r_s=2GM/c^2, où G est la constante de gravitation.

Question 1)

On se place dans le plan theta=pi/2. Que vaut ds^2dans le cas d'une orbite circulaire ?

Question 2)

On pose d*phi=omega*dt, où omega est la fréquence angulaire du mouvement vu par un observateur lointain. Sachant que pour qu'un mouvement soit physiquement réalisable il faut que ds^2<0 (ceci vient uniquement d'un choix spécifique de métrique); déterminer la condition sur omega.

Question 3)

Montrer que l'orbite finale (correspondant à la fréquence limite calculée précédemment) correspond à la trajectoire de photons pour lesquels ds^2=0. Que vaut son rayon ?


Intégrale de Riemann


Introduction

On trouvera dans cette partie les exercices suivants :


Longueur d'une courbe, aire d'une surface

Auteurs: Jérôme Thiébaut, Marc Fouchard, Alain Vienne

Redshift

Auteur : Jérôme Thiébaut

En coordonnées cartésiennes, un élément de longueur ds se calcule selon le théorème de Pythagore: ds^2=dx^2+dy^2+dz^2 .

Ceci donne en coordonnées sphériques: ds^2=dr^2+r^2*d*theta^2+r^2*sin(theta)^2*d*phi^2.

On voit que l'expression de ds^2 dépend de la métrique utilisée, c'est à dire de la manière de décrire l'espace. En cosmologie, dans le cadre de la relativité générale, on calcule de même les éléments de longueur en fonction de la métrique de l'espace temps soit:

ds^2=-c^2*dt^2+a(t)^2*(dr^2/(1-Kr^2)+r^2*d*theta^2+r^2*sin(theta)^2*d*phi^2),

ou a(t)est le facteur d'échelle qui décrit l'expansion de l'univers, t le temps, r , theta et phi les coordonnées comobiles (c'est à dire fixes par rapport à l'expansion de l'univers) , c la vitesse de la lumière et K la courbure de l'univers.

Pour un photon, la trajectoire est telle que ds^2=0.

On se propose dans cet exercice d'étudier la trajectoire d'un photon radial afin de relier le redshift (ou décallage spectral), z, au facteur d'échelle, a.


Ex: Redshift

Auteur: Jérôme Thiébaut

exerciceRedshift

Difficulté : ☆☆   Temps : 20mn

On considère un photon radial émis à une distance r_1 au temps t_1 par une galaxie lointaine. Ce photon nous est reçu au temps t_0 en r=0.

Sa trajectoire est décrite par la métrique: ds^2=-c^2*dt^2+a(t)^2*(dr^2/(1-Kr^2)+r^2*d*theta^2+r^2*sin(theta)^2*d*phi^2)=0.

Question 1)

Simplifier la métrique compte tenu de la nature du photon et exprimer l'égalité sous forme intégrale.

Question 2)

Un deuxième photon est émis à t_1 + delta*t_1 et est reçu en t_0 + delta*t_0. Quelle est la nouvelle égalité sous forme intégrale ?

Question 3)

Sachant que a(t) est considéré comme constant pendant un temps delta*t faible et que la distance comobile (r_1) est constante par définition, montrer que a(t_1)/a(t_0)=delta*t_1/(delta*t_°).

Question 4)

Le redshift z est défini comme suit: z=(lambda_0 -lambda_1)/lambda_1 , où lambda_0 est la longueur d'onde du photon reçu et lambda_1 celle du photon émis.

Sachant qu'à un delta*t correspond une longueur d'onde lambda, et que par convention, a(t_0)=1, relier la quantité 1+z au facteur d'échelle.


Calotte sphérique

Auteur : Marc Fouchard

Etant donné qu'un observateur sur Terre se trouve à une distance finie de la Lune, lorsqu'il regarde la Lune il ne perçoit qu'une calotte et non un hemisphère. L'objectif de cet exercice est de trouver la surface de la partie de la Lune observable depuis la Terre et de comparer cette surface à celle de l'hémisphère.

On note O l'observateur, L le centre de la Lune et \mathcal{C}_{\rm L} l'intersection de la sphère correspondant à la Lune avec un plan contenant la droite (OL). Ainsi \mathcal{C}_{\rm L} est un cercle de centre L et de rayon R, où R est le rayon de la Lune. On note d la distance OL. Les tangentes à \mathcal{C}_{\rm L} passant par O coupent \mathcal{C}_{\rm L} en Z et Z'. Soit N le point de {\mathcal{C}_{\rm L} tel que (LN) est perpendiculaire à (OL) avec N du même coté que Z de la droite (OL). On note H le projeté orthogonal de Z sur (OL), \alpha l'angle \widehat{ZOL} et r la distance HZ.

La calotte visible depuis O correspond donc à la partie de la surface de la Lune tournée vers O et de frontière le cercle de centre H et de rayon r.


Ex : Calotte sphérique

Auteur: Marc Fouchard

exerciceCalotte sphérique

Difficulté :    Temps : 30 mn

Question 1)

Faire la figure qui correspond à l'énoncé.

Question 2)

Soit \delta=\frac{R}{d}, calculer r en fonction de R et \delta.

Question 3)

Calculer la surface \mathcal{A}_{\rm C} de la calotte visible depuis O en fonction de la surface d'un hémisphère de la Lune \mathcal{A}_{\rm L} et de \delta.

Question 4)

Montrer que dans le cas d'un astronaute autour de la Terre la portion de surface \mathcal{A}_{\rm C} de la Terre visible par l'astronaute s'écrit sous la forme de la surface d'un cercle dont on déterminera le rayon en fonction du rayon de la Terre R_\oplus et de l'altitude h de l'astronaute. On supposera donc que h \ll R_\oplus.


Potentiel gravitationnel de la Terre

Auteur : Alain Vienne

Le potentiel gravitaionnel de la Terre est souvent modélisé par:

U(r,- ,\varphi ) = {\frac{KM}{r}}\,\ \left( 1 - J_2 \left({\frac{a_e}{r}}\right)^2  P_2(\sin \varphi) \right)

avec  P_2(s) = \frac{3}{2} s^2 -\frac{1}{2}

C'est le potentiel évalué en un point P de coordonnées sphériques (r,\lambda,\varphi) (dont le plan horizontal est la plan de l'équateur).

M est la masse totale de la Terre et a_e son rayon équatorial (K la constante de gravitation universelle). J_2 est un coefficient qui caractérise l'aplatissement de la Terre suivant l'axe des pôles. Sa valeur (sans unité) est de 0,00108.

Dans l'exercice qui suit, nous allons évaluer le potentiel d'un anneau massif et homogène. Nous verrons que l'expression obtenue aura exactement la même forme que celle ci-dessus.

Un exemple d'application concerne la prise en compte de la gravitation des anneaux de Saturne: il suffit de réévaluer le cooefficient d'aplatissement J_2 de Saturne.

Voici un autre exercice sur le potentiel gravitationnel terrestre.


Ex: Potentiel gravitationnel de la Terre

Auteur: Alain Vienne

exercicePotentiel gravitationnel de la Terre

Difficulté :    Temps : 1h

Question 1)

Soit un anneau de centre O et de rayon a. On repère un point P de coordonnées sphériques (r,\lambda,\varphi) (dont le plan horizontal est le plan de l'anneau).

Soit A un point de l'anneau. Il fait un angle \alpha avec le premier axe (même origine que l'angle \lambda).

Calculer la distance \Delta de A à P.

Question 2)

Calculer \frac{1}{\Delta} en se limitant aux termes de degré 2 au plus en (\frac{a}{r}).

Question 3)

Pour avoir le potentiel total de l'anneau, il faut sommer cette expression pour A variant le long de l'anneau. C'est-à-dire, il faut intégrer cette expression par rapport à \alpha qui varie de 0 à 2 \pi.

A partir de l'expression précédente, calculer U = \int_{An} \frac{K \  dm}{\Delta} .

Question 4)

En comparant cette expression avec celle utilisant le coefficient J_2, donner le rayon de l'anneau correspondant au potentiel terrestre. On donne a_e = 6400 km.


Théorèmes

Auteur: Jérôme Thiébaut, Stéphane Erard

Supernova 1A

Auteur: Jérôme Thiébaut

Les supernovae de type 1A correspondent à l'explosion d'une étoile de type naine blanche suite à l'accrétion de matière arrachée à une étoile géante proche. Ces phénomènes extrêmement lumineux sont visibles de très loin ce qui permet leurs détections. La courbe de luminosité d'une SN1A est caractéristique et permet de déterminer sa magnitude absolue. Le but de cet exercice est de montrer comment grâce à des mesures de luminosité, on peut déterminer le redshift de l'étoile, et donc de la galaxie hôte. Le redshift étant une mesure de la distance, les SN1A servent de balises pour mesurer les distances dans l'univers.

Courbe de lumière d'une supernova.
supernovacourbe.jpg
Courbe de lumière et spectre au maximum d'intensité d'une supernova.
Crédit : Supernova Cosmology Project, Berkeley University

Ex: Supernova 1A

Auteur: Jérôme Thiébaut

exerciceSupernova 1A

Difficulté :    Temps : 30 min

On observe une supernova de type 1A dans une galaxie et on mesure sa magnitude apparente m=24.75. Grâce à sa courbe de lumière, on détermine sa magnitude absolue M=-19.20.

Question 1)

On exprime la distance luminosité, D_L, comme suit: m=M+5*log(D_L)+25, où D_L est exprimée en mégaparsec (Mpc) (Le parsec étant une unité de distance correspondant à 3.10^16m).

Calculer sa valeur.

Question 2)

La distance de la galaxie est par définition: r=intégrale(frac(c;a(t));t;t_e;t_r), où c est la vitesse de la lumière, t le temps, t_e le temps d'émission de la lumière par la galaxie, t_r celui de réception par l'observateur et a(t) le facteur d'échelle décrivant l'expansion de l'univers.

Sachant que le facteur d'échelle est relié au redshift, z, par la relation suivante, a=frac(1;1+z) et que par définition la constante de Hubble vaut H=frac(dtemps(a;1);a); exprimer la distance r sous forme d'une intégrale selon z (on posera qu'à t_e, z=z_G et par définition à t_r , z=0).

Question 3)

Calculer r, puis sachant que la distance luminosité et la distance r sont reliées par la relation D_L=(1+z_G)*r (dans le modèle cosmologique standart LambdaCDM), déterminer l'équation du second degré à laquelle obéit z_G.

Question 4)

Résoudre cette équation et déterminer la valeur du redshift z_G (positive par définition). On donne c=3.10^5*km*s^(-1) et H=70 km*s^(-1)*Mpc^(-1).


Bruit de numérisation

Auteur: Stéphane Erard

Les instruments modernes utilisent des détecteurs numériques tels que des CCD, c'est-à-dire qu'ils fournissent en sortie un signal numérisé sur un nombre fini de valeurs. Cette étape produit une erreur d'arrondi appelée "bruit de quantification" ou "bruit de numérisation" qui peut dans certains cas limiter la précision de la mesure. On étudie ici la statistique de ce bruit.


Ex: bruit de numérisation

Auteur: Stéphane Erard

exerciceBruit de numérisation

Difficulté :    Temps : 30 min

L'exercice consiste à estimer l'erreur due à la numérisation (ou quantification) d'un signal continu.

Question 1)

On mesure un signal lumineux avec une caméra CCD. Tracer l'allure de la fonction de réponse du convertisseur analogique/numérique (CAN). Si le convertisseur fonctionne sur 12 bits, combien de valeurs sont disponibles en sortie ?

Le convertisseur est réglé pour couvrir la dynamique de la caméra jusqu'à un niveau analogique S_{max}. Quel est le pas de numérisation du signal ?

Question 2)

Estimer l'erreur quadratique moyenne due à la numérisation.

Question 3)

Calculer le rapport signal sur bruit correspondant. Comment peut-on améliorer celui-ci, et jusqu'à quel point ?

Question 4)

Comparer aux autres sources de bruit.


Intégrales Généralisées

Auteurs: Marc Fouchard, Stéphane Erard, S. Renner

Loi de Stefan

Auteur : Marc Fouchard

La loi de Planck montre que pour un corps noir, l'énergie émise par rayonnement suivant une longueur d'onde, ne dépend que de la température de surface du corps noir. Cette loi est donnée par la relation suivante :

E(\lambda)=\frac{2 h c^2}{\lambda ^5}\cdot \frac{1}{\exp \left( \frac{h c}{k \lambda T} \right) -1}

c correspond à la vitesse de la lumière dans le vide, hest la constante de Planck, kla constante de Boltzmann, \lambdala longueur d'onde à laquelle le rayonnement est émis et Tla température de surface du corps noir.

La figure ci dessous montre le comportement de E(\lambda) pour différente température de surface du corps noir. Sachant que l'énergie totale E_{\rm tot} émise par le corps noir par seconde et par unité de surface correspond à l'aire comprise en l'axe des abcisses et la courbe, on remarque que E_{\rm tot} augmente avec la température de surface du corps noir (il ne faut pas cocher la case "normaliser").

Le but de cet exercice est d'établir la relation exacte entre E_{\rm tot} et la température de surface T du corps noir.

Loi de Planck application.png

remarqueRemarque

On pourra aussi voir cet exercice en lien avec la loi de Planck pour les corps noirs.


Ex : loi de Stefan

Auteur: Marc Fouchard

exerciceloi de Stefan

Difficulté : ☆☆   Temps : 1h

Question 1)

Sachant que h, c et ksont des constantes strictement positives et que la température Tétant mesurée en degré Kelvin est aussi strictement positive, montrer que E est de classe {\mathcal C}^{\infty} sur ]0,+\infty[ et est toujours strictement positive sur cet intervalle.

Question 2)

L'énergie totale E_{\rm tot} est donnée par :

E_{\rm tot}=\int_0^{+\infty} \frac{2 h c^2}{\lambda ^5}\cdot \frac{1}{\exp \left( \frac{h c}{k \lambda T} \right) -1}{\rm d} \lambda.

Montrer, en effectuant un changement de variable approprié, que E_{\rm tot} peut s'écrire sous la forme:

E_{\rm tot}=A \int_0^{+ \infty} \frac{u^3}{{\rm e}^u-1}{\rm d} u \,T^4

C est une constante que l'on déterminera.

Question 3)

Montrer que l'intégrale

I=\int_0^{+\infty}\frac{u^3}{{\rm e}^u-1}{\rm d} u

est convergente.

remarqueRemarque

Cette intégrale est une fonction zeta de Reimann. On peut montrer que I=\pi^4/15.

Question 4)

En déduire la loi de Stefan:

E_{\rm tot}=\sigma \, T^4

\sigma est une constante que l'on déterminera.


Distribution des vitesses de Maxwell

Auteur: Stéphane Erard

On considère un gaz en équilibre, pour lequel on veut connaître les vitesses des molécules. La théorie cinétique des gaz ne donne qu'une valeur moyenne (la vitesse quadratique moyenne) :

v_q= \sqrt{\overline{v^2}} = \sqrt{\frac{3kT}{m}}

où m est la masse des molécules, T est la température, k la constante de Boltzman.

On cherche ici la distribution de vitesse, c'est-à-dire la probabilité d'avoir une vitessse comprise entre v et v+dv. Le calcul qui suit est classique (non quantique) et reproduit l'étude de Maxwell au XIXe siècle.


Ex: Distribution des vitesses de Maxwell

Auteur: Stéphane Erard

exerciceIntégrales gaussiennes

Difficulté : ☆☆   Temps : 60 min

Le calcul des propriétés de la loi normale suppose l'intégration de la fonction gaussienne, et des intégrales similaires apparaissent dans le calcul suivant.

Le moment d'ordre n de la loi normale réduite (de moyenne nulle) est :

M_n = C \int_{-\infty}^{\infty}x^n e^{-ax^2} dx

où a > 0 et n ≥ 0, C étant une constante de normalisation. On s'intéresse ici à :

I_n = \int_{0}^{\infty}x^n e^{-ax^2} dx

Question 1)

Trouver une relation de récurrence entre les intégrales I_n.

Question 2)

Calculer I_1. Que représente cette quantité ?

Question 3)

Calculer l'intégrale de Gauss I = 2 \times I_0 = \int_{-\infty}^{\infty}e^{-ax^2} \ dx

Question 4)

En déduire les moments de la loi normale centrée.

Auteur: Stéphane Erard

exerciceDistribution des vitesses

Difficulté : ☆☆   Temps : 60 min

La probabilité pour qu'une molécule ait une vitesse comprise entre \vec{v} et d\vec{v} est notée G(\vec{v}). Cette probabilité ne dépend pas de la position ni du temps, car le gaz est en équilibre. Si on fait l'hypothèse que la vitesse est isotrope (qui est vérifiée au sommet d'une atmosphère planétaire , ou dans un nuage de gaz interstellaire), G ne dépend que du module de la vitesse.

Question 1)

Ecrire les conditions d'indépendance entre les composantes du vecteur vitesse, et d'isotropie.

Question 2)

En déduire la forme de G.

Question 3)

Identifier deux conditions qui permettent de calculer les coefficients ci-dessus.

Question 4)

Calculer les intégrales I_2 et I_4 de l'exercice précédent.

Question 5)

En dériver l'expression de G(v^2) à l'aide des intégrales gaussiennes.

Question 6)

En dériver l'expression de F(v), densité de probabilité pour le module de la vitesse.

Question 7)

Tracer cette fonction, expliquer sa forme.

Question 8)

Comment peut-on utiliser cette fonction pour expliquer l'évolution des atmosphères planétaires ?


Temps de vie d'une étoile

Auteur : S. Renner

On estime ici la durée de vie d'une étoile de type solaire, en supposant tout d'abord que la seule source d'énergie est la gravitation, puis en considérant le cas réel des réactions de fusion thermonucléaire de l'hydrogène en hélium. La première hypothèse (dissipation de l'énergie gravitationnelle) est une idée qui apparaît avec les travaux de Kelvin au XIXe siècle.


Ex: Temps de vie d'une étoile

Auteur: S. Renner

exerciceTemps de vie d'une étoile

Difficulté :    Temps : 1h

On assimile l'étoile à une sphère homogène de masse M et de rayon R.

Question 1)

Montrer que son énergie de liaison gravitationnelle est E_G = - \frac{3}{5} \frac{GM^2}{R}.

Question 2)

En déduire le temps de vie \tau du Soleil sur ses seules ressources gravitationnelles. On rappelle que la luminosité (puissance totale rayonnée) du Soleil est L_\odot = 4 \times 10^{26} W, sa masse M_\odot = 2 \times 10^{30} kg et son rayon R_\odot = 7 \times 10^8 m.

Question 3)

Même question en considérant le cas réel des réactions de fusion nucléaire de l'hydrogène en hélium au coeur du Soleil. On suppose que 10% de la masse M_\odot est convertie en hélium et que la luminosité L_\odot reste constante. Le rendement de la réaction hydrogène -> hélium est de 0.7%, et on rappelle la relation d'équivalence masse-énergie E = m c^2.

Question 4)

Le Soleil brille depuis 4.5 milliards d'années. Combien a t-il perdu en masse ?


Développements d'une fonction d'une variable réelle


Introduction

On trouvera dans ce chapitre les exercices suivants :


Développements limités

Auteurs: Marc Fouchard, Jérôme Thiébaut, S. Renner, Stéphane Erard

Lentille gravitationnelle

Auteur : Jérôme Thiébaut

L'idée de base de la relativité générale est que la matière, par sa masse, courbe l'espace. Ainsi, une planète orbitant autour d'une étoile n'est pas soumise à une force de gravitation mais circule librement sur un espace courbé par l'étoile. Il s'ensuit que la lumière, bien que dépourvue de masse, est également déviée par la présence d'un corps massif. Si un corps massif se situe entre une galaxie lointaine et un observateur, celui ci va donc dévier la lumière de la galaxie et déformer son image. C'est ce qu'on appelle une lentille gravitationnelle. Dans le cas où les trois objets sont parfaitement alignés, l'image de la galaxie se déforme pour former un anneau autour de la lentille appelé anneau d'Einstein. Le but de cet exercice est de déterminer le diamètre angulaire de cet anneau en fonction des caractéristiques du système (masse et distances).

Lentille Gravitationnelle
gravlens.jpg
Lentille gravitationnelle : la déviation de la lumière par un fort potentiel gravitationnel (l'amas de galaxies 0024+1654) conduit à de multiples images d'un objet situé derrière le centre de masse du déflecteur.
Crédit : HST

Ex : Lentille gravitationnelle

Auteur: Jérôme Thiébaut

exerciceLentille gravitationnelle

Difficulté :    Temps : 30 min

La figure montre le schéma du principe d'une lentille gravitationnelle. La source, S, est déviée par la lentille, L, et son image, I, est donc vue par l'observateur, O, selon un angle theta_I au lieu de theta_S.

Principe d'une lentille gravitationnelle
lg.png
Crédit : Jérôme Thiébaut
Question 1)

Exprimer les distances AS et AI en fonction des angles theta_S et theta_I et de la distance OS.

Question 2)

Les calculs relativistes montrent que la distance SI vaut alpha*LSoù l'angle de déviation, alpha, vaut 4GM/c^2*theta_I*OL, avec G la constante de gravitation, M la masse de la lentille et c la vitesse de la lumière. Dans l'approximation des petits angles, déterminer l'équation du second degré à laquelle obéit theta_I en fonction de theta_S, et des grandeurs caractéristiques du système, LS, OS, OL et M.

Question 3)

Dans le cas où la source, la lentille et l'observateur sont parfaitement alignés, déterminer le rayon angulaire (rayon d'Einstein) sous lequel sera vu la source toujours en fonction des grandeurs caractéristiques du système.


Repérage des astres

Auteur : S. Renner

Date de création: 30 novembre 2009

On propose ici un exercice en lien avec le passage des coordonnées locales (ou horizontales) aux coordonnées horaires (angle horaire et déclinaison).

Il est donc préférable de se familiariser avec les systèmes de coordonnées utilisés en astronomie pour le repérage des étoiles dans le ciel, ainsi qu'avec les formules de base de la trigonométrie sphérique.


Ex: Repérage des astres

Auteur: S. Renner

exerciceRepérage des astres

Difficulté : ☆☆   Temps : 1h

Dans le triangle sphérique PZM (voir la fiche de résolution du triangle sphérique), on obtient des relations qui permettent de passer des coordonnées horizontales aux coordonnées horaires, et inversement.

Question 1)

La distance zénithale z dépendant seulement de la variable H (angle horaire), démontrer que {dz \over dH} = \cos \phi \sin a , puis {da \over dH} = {\cos \delta \cos S \over \sin z}.

Question 2)

En déduire qu'au voisinage du passage au méridien d'un astre, sa distance zénithale z varie comme le carré de l'angle horaire (en se limitant au 2ème ordre).


Equation de Kepler

Auteur : Marc Fouchard

Dans le problème de deux corps (voir cet exercice), on sait que le déplacement d'un corps par rapport à l'autre se fait sur une conique dont le deuxième corps occupe l'un des foyers (voir aussi cet . Une fois la conique fixée il ne reste alors qu'à positionner le corps sur son orbite. Pour cela on utilise une quantité qu'on appelle anomalie. On définie trois types d'anomalie: l'anomalie moyenne, l'anomalie vraie et l'anomalie excentrique.

Les trois anomalies
kepler.gif
Crédit : Astrophysique sur Mesure / Bessou Fouchard

L'animation ci-dessus montre le lien entre les 3 anomalies. Comme on peut le voir, l'anomalie moyenne correspond en fait à un temps. Il n'existe pas de relation géométrique entre l'anomalie moyenne et les autres anomalies. En revanche, il existe une relation (voir cet exercice), appelée équation de Kepler, qui relie l'anomalie moyenne à l'anomalie excentrique. Cette relation est:

M=E-e\sin E,

M est l'anomalie moyenne, E l'anomalie excentrique et e l'excentricité de la trajectoire.

On voit que connaissant E il est facile d'avoir M, mais en revanche connaissant M il n'est pas possible d'avoir E sous forme analytique. L'objet de cette exercice est justement de déterminer un algorithme puissant d'inversion de cette équation.


Ex: Equation de Kepler

Auteur: Marc Fouchard

exerciceEquation de Kepler

Difficulté : ☆☆   Temps : 1h30

Question 1)

On souhaite résoudre une équation du type f(x)=0. Soit a une solution à l'équation. On supposera dans la suite que, sur un intervalle I contenant a, f' ne s'annulle jamais et que f est de classe \mathcal{C}^{\infty} sur cet intervalle. Soit x_n, la suite définie par :

x_{n+1}=x_n+\delta_n,

avec \delta_n, la solution de f(x_n)+\delta_n f'(x_n)+\cdots+\frac{\delta^n}{k!}f^{(k)}(x_n)=0 (k étant fixé).

On souhaite montrer que si la suite x_n converge vers a, alors elle converge au moins avec l'ordre k+1.

Calculer f(x_{n+1}) en fonction de \delta_n et un nombre \zeta_1 \in \lbrack x_n,x_{n+1}\rbrack.

Question 2)

Soit \epsilon_{n+1} l'erreur de x_{n+1} définie par \epsilon_{n+1}=a-x_{n+1}. Cacluler \epsilon_{n+1} en fonction de \delta_n, \zeta_1 et un nombre \zeta_2 \in \lbrack x_{n+1},a\rbrack.

Question 3)

Montrer que:

\frac{\epsilon_n}{\delta_n}=\frac{f'(x_n)}{f'(\zeta_3)}+\frac{\delta_n}{2!}\frac{f''(x_n)}{f'(\zeta_3)}+\cdots+\frac{\delta_n^{k-1}}{k!}\frac{f^{(k)}(x_n)}{f'(\zeta_3)}, avec \zeta_3 un nombre entre a et x_n.

Question 4)

En déduire que \lim_{n\to +\infty}\frac{\epsilon_n}{\delta_n}=1.

Question 5)

En déduire que \lim_{n\to+\inty}\frac{\epsilon_{n+1}}{\epsilon_n^{k+1}}=-\frac{1}{(k+1)!}\frac{f^{(k+1)}(a)}{f'(a)}, et que la suite x_n converge vers a au moins à l'ordre k+1.

Question 6)

Il nous faut maintenant calculer la solution \delta_n de l'équation qui le défini. Avant tout, jusitfier pourquoi il suffit d'avoir une solution approxée à \delta_n^{k+1} près.

Question 7)

Montrer que pour k=3, on a:

\delta_n=-\frac{f(x_n)}{f'(x_n)+\frac{1}{2}\delta_n f''(x_n)+\frac{1}{6}\delta_n^2f'''(x_n)}

Question 8)

Montrer qu'en utilisant la séquance suivante:

\delta_{n1}=-\frac{f(x_n)}{f'(x_n)},

\delta_{n2}=-\frac{f(x_n)}{f'(x_n)+\frac{1}{2}\delta_{n1}f''(x_n)},

\delta_{n3}=-\frac{f(x_n)}{f'(x_n)+\frac{1}{2}\delta_{n2}f''(x_n)+\frac{1}{6}\delta_{n2}f'''(x_n)},

on obtient une approximation de \delta_n à \delta_n^4 près. On pourra procéder par étape en montrant que \delta_n-\delta_{n1}=\mathcal{O}(\delta_n^2), puis que \delta_n-\delta_{n2}=\mathcal{O}(\delta_n^3) et enfin que \delta_n-\delta_{n3}=\mathcal{O}(\delta_n^4).

Question 9)

Montrer que l'on peut appliquer l'algorithme précédent pour inverser l'équation de Kepler sur l'intervalle [0,2\pi]. Aplliquer l'algorithme à quelque exemple et remarquer qu'on obtient une erreur inférieure à 10^{-12} en moins de 4 itérations.


Effet de marée et limite de Roche

Auteur : S. Renner

Date de création: 31 janvier 2011

On cherche à déterminer la distance, que l'on notera d_R, en dessous de laquelle un satellite commence à se disloquer sous l'action des forces de marée causées par la planète autour de laquelle il orbite.

Cette distance théorique s'appelle la limite de Roche. Elle tire son nom de l'astronome français qui l'a formulée en 1850.


Ex: Effet de marée et limite de Roche

Auteur: S. Renner

exerciceEffet de marée et limite de Roche

Difficulté : ☆☆   Temps : 1h

On se place dans un référentiel galiléen centré sur la planète, supposée sphérique, homogène, de masse M, rayon R, et masse volumique \rho_P.

On suppose que le satellite est constitué de deux sphères homogènes identiques (de masse m, de rayon r et de masse volumique \rho), et qu'il est en orbite circulaire de rayon d autour de la planète.

roche_limite.jpeg
Bilan des forces agissant sur une masse m dans le repère lié à la planète (supposé galiléen) : {\bf F_{P1}} est la force d'attraction gravitationnelle de la planète, {\bf F_m} celle de la deuxième sphère de masse m, {\bf C_1} la force de contact entre les deux sphères.
Crédit : Astrophysique sur Mesure / Renner
Question 1)

En appliquant le principe fondamental de la dynamique (PFD) au système des deux masses m, montrer qu'au premier ordre en r/d la vitesse angulaire de la comète est \displaystyle \omega = \sqrt{\frac{G M}{d^3}}.

Question 2)

Appliquer le PFD à l'une des masses m, et trouver un critère de fragmentation du satellite (contact rompu entre les deux sphères) au premier ordre en r/d.

Question 3)

En déduire l'expression de la limite de Roche d_R en fonction de M, m, r, puis en fonction de \rho_P, \rho, R.

Question 4)

Retrouver l'expression de la limite de Roche en écrivant que la différence de force entre les deux masses m due à l'attraction gravitationnelle de la planète est supérieure à la force de gravitation mutuelle entre les deux sphères.


Générateur de coefficients de Taylor

Auteur : Marc Fouchard

Date de création: 8 Mai 2013

En astronomie on a souvent besoin de reproduire le mouvement des objets en effectuant des intégraitons numériques. Quelque soit la méthode, elle repose toujours sur des approximations et elle nécessite un certain temps de calcul. Ainsi l'objectif d'une méthode est de trouver le meilleur compromis entre temps de calcul et précision.

Une méthode très performante est la méthode dite de Taylor.

Soit l'équation différentielle suivante :

\mathbf{y}' = f(t,\mathbf{y}), \quad \mathbf{y}(t_0)=\mathbf{y}_0.

Soit h, un pas d'intégration. Le but est de trouver \mathbf{y}(t_0+h) avec la meilleure précision possible.

La méthode de Taylor repose sur le développement de \mathbf{y} en t_0 :

\mathbf{y}(t_0+h)=\mathbf{y}(t_0) + \mathbf{y}'(t_0) h + \frac{1}{2!}\mathbf{y}''(t_0) h ^2 + \cdots + \frac{1}{k!}\mathbf{y}^{(k)}(t_0) h^k + \cdots

Le problème principal est donc d'estimer les coefficients du développement de Taylor de la solution.


Ex: générateur de coefficients de Taylor

Auteur: Marc Fouchard

exerciceGénérateur de coefficients de Taylor

Difficulté : ☆☆   Temps : 30mn

Question 1)

Soit f, une fonction analytique. On note (f)_i, le ième coefficient de Taylor de la fonction f en t_0 défini par :

(f)_i = \frac{f^{(i)}(t_0)}{i!}.

Soit p et q, deux fonctions analytiques, déterminer (p \pm q )_i et (pq)_i

Question 2)

Déterminer \left(\frac{p}{q}\right)_i.

Question 3)

Montrer que :

(p^a)_i=\frac{1}{p}\sum_{r=0}^{i-1} \left ( a -\frac{r(a+1)}{i}\right) (p)_{i-r}(p^a)_r


Limites de la loi de Planck

Auteur : Stéphane Erard

Date de création: 30 Mai 2013

La loi de Planck donne l'expression exacte du spectre du corps noir. Historiquement elle a été dérivée après des approximations valables aux grandes et courtes longueurs d'onde, qui sont toujours utilisées dans certaines situations. L'exercice propose de retrouver ces approximations à partir de la loi complète.


Ex: Limites de la loi de Planck

Auteur: Stéphane Erard

exerciceLimites de la loi de Plack

Difficulté :    Temps : 30 min

On connaît la luminance du corps noir en fonction de la longueur d'onde, donnée par la loi de Planck (voir par exemple l'exercice sur la loi de Wien) : B_{\lambda} = \frac{2hc^2}{\lambda^5 (e^{hc/kT\lambda} -1) }

c est la vitesse de la lumière dans le vide, h la constante de Planck, k la constante de Boltzmann, \lambda la longueur d'onde et T la température du corps noir.

Cette expression donne la luminance directionnelle, en W\,m^{-2}\,sr^{-1}\,\mu m^{-1}, du corps noir à toutes les longueurs d'onde.

Question 1)

Donner une expression de cette loi à courtes longueurs d'onde. Cette expression est connue sous le nom de loi ou distribution de Wien.

Question 2)

Donner une expression de cette loi à grandes longueurs d'onde. Cette expression est connue sous le nom de loi de Rayleigh-Jeans. Commentaire ?

Question 3)

Tracer les graphiques de ces deux expressions et de la loi de Planck en échelle log/log, comparer. Quel problème pose l'approximation de Rayleigh-Jeans ?


Fonctions de deux variables


Introduction

On trouvera dans ce chapitre les exercices suivants :


Dérivées partielles, gradient et plan tangent

Auteurs: Jérôme Thiébaut, Arnaud Beck

Perturbations linéaires

Auteur: Jérôme Thiébaut

Le CMB (Cosmic Microwave Background où fond diffus cosmologique) représente le champ de densité de l'univers lorsque celui ci avait environ 300 000 ans (dans le cadre de la théorie du Big Bang). L'un des enjeux de la cosmologie est, à partir de ce champ de densité, de comprendre son évolution et la formation des structures à différentes échelles (galaxies, amas, super amas, filaments). Dans un premier temps, on peut linéariser les équations régissant le mouvement d'un fluide afin de déterminer l'évolution linéaire du champ de densité. Le but de l'exercice est de déterminer l'équation différentielle à laquelle obéit le champ de densité dans ce régime.

Le fond diffus cosmologique vu par COBE et WMAP.
COBE_WMAP_sm.jpg
Cartes du ciel obtenues par les satellites COBE (en haut) et WMAP (en bas); on remarque la résolution spatiale bien meilleure de WMAP.
Crédit : COBE/WMAP

Ex: Perturbations linéaires

Auteur: Jérôme Thiébaut

exercicePerturbations linéaires

Difficulté : ☆☆   Temps : 1 h

Les équations fondamentales gouvernant le mouvement d'un fluide sont:

D/Dt=drond/drondt+pscalaire(v;nabla)est la dérivée convective (variation totale dûe à la variation temporelle, frac(drond;drondt), et spatiale, pscalaire(v;nabla), du fluide) , v la vitesse du fluide, P sa pression, rho sa densité, G la constante de gravitation et Phi le potentiel gravitationnel. On souhaite tout d'abord linéariser les équations autour d'un fond homogène, rho=rho_0+delta*rho, v=v_0+delta*v, P=P_0+delta*P et Phi=Phi_0+delta*Phi.

Question 1)

Réecrire les trois équations linéarisées au premier ordre.

Question 2)

Soustraire à ces équations les équations à l'ordre 0 (c.à.d. les équations non perturbées où les delta*x sont nuls) et les exprimer en fonction du contraste de densité lambda=delta*rho/rho_0

Question 3)

La dérivée convective des quantités perturbées est donc d/dt=drond/drondt+pscalaire(v_0;nabla). Ces équations sont écrites en coordonnées eulériennes x(t) (système de coordonnées fixes). On se propose maintenant de les écrire en coordonnées comobiles, r(t), système de coordonnées qui évolue et se dilate avec l'expansion de l'universafin de s'en affranchir. Le passage de l'une à l'autre se fait par: x(t)=a(t)*r(t) et delta*v(t)=a(t)*u(t)a(t) est le facteur d'échelle caractérisant l'expansion et u(t)=dr/dt est la vitesse particulière du fluide. On peut montrer que pscalaire(delta*v;nabla)*v_0=(dtemps(a;1)/a)*delta*v . Réecrire les équations d'Euler et de conservation en fonction de a et u(t).

Question 4)

La vitesse du son c_S est par définition subsup(c;S;2)=drond*p/drond*rho . Dériver l'équation de conservation afin de déterminer l'équation différentielle du deuxième ordre régissant l'évolution de lambda .


Potentiel gravitationnel terrestre

Auteur: Arnaud Beck

A l'ordre zéro, le potentiel gravitationnel de la Terre est supposé être celui d'une masse ponctuelle située au centre de la terre. Cette hypothèse revient en fait à supposer que la symétrie sphérique de la Terre est parfaite et donc, que son potentiel gravitationnel U ne dépend que de la distance au centre r. Or, des mesures suffisamment précises ont montré que des effets "non sphériques" étaient détectables. Cet exercice propose d'étudier la fonction du potentiel terrestre corrigé au premier ordre. Cette correction est celle qui est due à l'aplatissement de la Terre aux pôles et rend le potentiel dépendant de la latitude.

Voici un autre exercice sur le potentiel gravitationnel terrestre.


Ex: Potentiel gravitationnel terrestre

Auteur: Arnaud Beck

exercicePotentiel gravitationnel terrestre

Difficulté :    Temps : 1h

introductionIntroduction

Dans le repère du centre de gravité de la Terre et en supposant la Terre à symétrie de révolution autour de son axe, on peut montrer qu'une bonne approximation du potentiel gravitationnel est donné par

U(r,\theta)=-\frac{\mathcal{G}m_T}{r}+\frac{1}{2}\frac{\mathcal{G}m_T}{r}\left(\frac{r_e}{r}\right)^2J_2(3\sin (\theta)^2-1)

r_e est le rayon équatorial de la Terre, m_T sa masse, \mathcal{G} la constante de gravitation, J_2 un coefficient de correction sans dimension, \theta la latitude et r la distance au centre.

Question 1)

Sachant que le champ de gravitation est donné par le gradient du potentiel U, donner les composantes radiale et tangentielle de ce champ.

Question 2)

En symétrie parfaitement sphérique, le champ gravitationnel de la Terre est non nul en tout point de l'espace. Il est intéressant de noter que ce n'est pas le cas si l'on prend en compte l'aplatissement de la Terre. Déterminer les points en lesquels le champ gravitationnel s'annule. D'un de point de vue mathématique, que sont ces points pour la fonction U ?

Question 3)

Voici une représentation de U en niveau de gris. Le cercle blanc centrale est une zone où le potentiel diverge et n'est pas évalué. Il ne faut donc pas en tenir compte. En faisant des coupes sur cette image, déterminer la variation de U dans les directions radiale et tangentielle au niveau des points critiques. En déduire si ces points sont des minima, des maxima ou des points selles.

application.png


Equations différentielles linéaires


Introduction

On trouvera dans cette partie les exercices suivants :


Du premier ordre

Auteurs: Stéphane Erard, Marc Fouchard

Transfert de rayonnement

Le transfert de rayonnement décrit l'interaction du rayonnement électromagnétique et de la matière. Cette discipline permet notamment d'analyser la propagation de la lumière à travers un milieu gazeux et joue donc un rôle fondamental dans l'analyse des spectres stellaires et des atmosphères planétaires.

L'équation de transfert fait le bilan énergétique relatif au transport de photons dans un milieu. Comme toujours, on écrit que la quantité à laquelle on s'intéresse varie proportionnellement à sa valeur sur un intervalle suffisamment petit pour que le coefficient soit constant : \frac{dI_{\nu}}{d\tau_{\nu}}  = -I_{\nu} + S_{\nu}

I_{\nu} est l'intensité lumineuse à la fréquence \nu, \tau_{\nu} est la profondeur optique du milieu, S_{\nu} est la fonction source, égale au rapport du coefficient d'émission au coefficient d'absorption du milieu traversé.

En intégrant cette équation le long du trajet du faisceau lumineux, on a :

I_{\nu}}(s)  = I_{\nu}(0) e^{-\tau_{\nu}} + \int_{0}^{\tau_{\nu}} S_{\nu}(t_{\nu}) e^{-t_{\nu}} dt_{\nu}

t_{\nu}} = \int_{s'}^{s} \kappa_{\nu}(z) dz est l'épaisseur optique entre les points s' et s, et \kappa_{\nu}(z) est le coefficient d'absorption du milieu en z.

On veut résoudre cette équation pour connaître l'intensité en fonction des propriétés du milieu. Les exercices suivants étudient des situations particulières qu'on rencontre fréquemment.


Ex: Transfert de rayonnement

Auteur: Stéphane Erard

exerciceAtmosphère planétaire en visible

Difficulté :    Temps : 15 min

Question 1)

On se place dans l'approximation "plan-parallèle" où on néglige localement la courbure de la planète. Déterminer l'expression de l'intensité lumineuse reçue à la surface de la Terre ou de Mars, dans le cas où le Soleil est au zénith.

figure 1
transfert_rayonnement_exo_1.png
On s'intéresse au flux descendant, le Soleil étant à la verticale.
Crédit : Astrophysique sur Mesure / Bessou & Erard

Question 2)

Cas où le Soleil est vu sous un certain angle (la profondeur optique est toujours mesurée à la verticale).

figure 2
transfert_rayonnement_exo_2.png
Le Soleil est cette fois vu sous un certain angle.
Crédit : Astrophysique sur Mesure / Bessou & Erard

Auteur: Stéphane Erard

exerciceFace nuit de Vénus

Difficulté : ☆☆   Temps : 30 min

Question 1)

Vénus a une température de surface très élevée, de l'ordre de 740 K, qui ne varie quasiment pas au cours de la journée. La température diminue avec l'altitude, pour s'établir à environ 230 K au sommet des nuages côté nuit.

Quelle est l'allure du spectre infrarouge de la face nuit de Vénus ?

figure 3
transfert_rayonnement_exo_3.png
On s'intéresse cette fois au flux montant, qu'on mesure au sommet de l'atmosphère.
Crédit : Astrophysique sur Mesure / Bessou & Erard

Question 2)

A quoi est due la température de surface ?

Question 3)

Il existe néanmoins d'étroites régions spectrales entre 1 et 2,5 \mu m où l'atmosphère n'est pas entièrement opaque. Ecrire le flux émergent dans ces régions spectrales où l'atmosphère est semi-transparente.

Question 4)

La figure 4 donne un spectre observé de la face nuit de Vénus. Interpréter le flux spectral mesuré à la lumière des questions précédentes.

figure 4
Spectrenuit.png
Spectre infrarouge de Vénus mesuré sur la face nuit (instrument Virtis / Venus-Express). La courbe rouge est un corps noir à 233 K.
Crédit : ESA / LESIA


Equation de Kepler

Auteur: Marc fouchard

On reprend les résultats obtenus dans l'exercice sur la résolution du problème de 2 corps. Le but est d'obtenir à partir de ces résultats l'équation de Kepler. Cette équation est fondamentale en mécanique céleste puisque c'est elle qui fait le lien entre le temps et la position de l'objet sur son orbite (voir la figure).

Les trois anomalies
kepler.gif
Crédit : Astrophysique sur Mesure / Bessou Fouchard

Ex: equation de Kepler

exerciceéquation de Kepler

Difficulté : ☆☆☆   Temps : 1h

Question 1)

On considère le mouvement d'un corps ponctuel P de masse négligeable soumise à l'attraction universelle d'un corps de masse m situé au centre O du repère de référence. La force s'appliquant à P est donnée par : \mathbf{F}=-\frac{\mu m}{r^3}\mathbf{OP}, où la notation en gras dénote des vecteurs, \mu est la constante universelle de la gravitation, et r=||\mathbf{OP}||. Les coordonnées de P dans le repère tournant étant {r \choose \theta}, on a vu dans l'exercice du problème de 2 corps que la solution des équations du mouvement est :

r=\frac{p}{1+e\cos (\theta -\omega)},

p=\frac{h^2}{\mu}=a(1-e^2), e est l'excentricité de la trajectoire, a est le demi-grand axe et \omega est l'argument du péricentre

On a vu aussi que la norme du moment angulaire h=r^2\dot{\theta}=\sqrt{\mu a (1-e^2)} est une constante du mouvement, ainsi que l'intégrale de l'énergie donnée par C=\frac{1}{2}v^2-\frac{\mu}{r}, où v est la norme de la vitesse de P.

Dans le repère tournant on a vu que v^2=\dot{r}^2+r^2\dot{\theta}^2.

Entre un instant t et l'instant t+\delta t, le point P{r \choose \theta} s'est déplacé au point P' {r+\delta r \choose \theta+\delta \theta }. En déduire, l'aire élémentaire \delta A balayée par le rayon vecteur \mathbf{OP} pendant l'intervalle de temps \delta t. On ne retiendra que les quantités d'ordre 1.

Question 2)

En déduire que le mouvement moyen n=\frac{2\pi}{T} , où T est la période du mouvement est tel que \mu=n^2 a^3.

Cette relation correspond à la troisième loi de Kepler.

Question 3)

Après avoir vérifier que h=na^2\sqrt{1-e^2}, montrer que:

r\dot{f}=\frac{na}{\sqrt{1-e^2}}(1+e\cos f)=\frac{na^2\sqrt{1-e^2}}{r}

et que:

\dot{r}=\frac{na}{\sqrt{1-e^2}}e\sin f

f=\theta-\omega. f s'appelle l'anomalie vraie et correspond à l'angle entre le péricentre et P vu depuis le foyer O.

Question 4)

Après avoir montrer que v^2=\mu\left(\frac{2}{r}-\frac{1}{a}\right), en déduire que l'intégrale de l'énergie C=-\frac{\mu}{2a}.

Question 5)

Soit \mathcal{C} le cercle de centre le centre C de l'ellipse correspondant à la trajectoire de P et de diamètre le grand axe de l'ellipse, c'est à dire 2a. La projection de P sur le cercle \mathcal{C} parallèlement au petit axe de l'ellipse est noté P'. On appelle anomalie excentrique l'angle E entre O et P' vu depuis C. Sachant que l'équation de l'ellipse correspondant à la trajectoire de P dans le repère centré sur C et d'axe des abscisses le grand axe dirigé vers le foyer O, et d'axe des ordonnées la direction orthogonale dans le sens direct, est:

\left(\frac{\bar{x}}{a}\right)^2+\left(\frac{\bar{y}}{b}\right)^2=1,

en déduire l'expression de \bar{x} et \bar{y} en fonction de E, puis l'expression de r en fonction de E.

Question 6)

Montrer, en utilisant l'expression de v^2 que r vérifie l'équation différentielle suivante:

\frac{\mathrm d r}{\mathrm d t}=\frac{na}{r}\sqrt{a^2e^2-(r-a)^2}

Question 7)

En déduire que l'anomalie excentrique E vérifie l'équation différentielle:

\frac{\mathrm d E}{\mathrm dt}=\frac{n}{1-e\cos E}.

Question 8)

En déduire l'équation de Kepler M=E-e\sin E, où l'anomalie moyenne M est définie par M=n(t-\tau)\tau correspond à l'instant de passage au péricentre, c'est à dire quand E=0.


Du second ordre

Auteurs: Arnaud Beck, Marc Fouchard, S. Renner, Florent Deleflie, Alain Vienne

Pendule et équation de Mathieu

Auteur: Alain Vienne

Beaucoup de modèles dynamiques, après maintes transformations (hypothèses simplificatrices, moyennisations, ...), ressemblent au modèle du pendule (masse à une distance constante d'un point fixe sous l'effet de la pesanteur). Ici nous allons nous intéresser à un type d'équation du pendule correspondant à l'équation de Mathieu:

\ddot{\theta} + \omega_0^2(1+\varepsilon \cos \omega t ) \theta = 0

Si \varepsilon est nul, c'est l'équation d'un pendule simple pour de petites oscillations. Dans ce cas, \omega_0^2 est inversement proportionnel à la longueur du pendule. On rappelle que la période T_0 est alors \frac{2\pi}{\omega_0}.

Ici \epsilon est un petit paramètre. On dit que le modèle du pendule simple est perturbé. L'équation de Mathieu est un cas particulier de l'équation

\ddot{\theta} +  w^2(t) \theta = 0

w(t) est une fonction périodique de période T qui est utilisée en Mécanique Céleste pour l'étude du mouvement de la Lune.

De manière plus ludique, ces équations peuvent modéliser le mouvement d'une balançoire dont le passager se lève et s'assied (périodiquement) afin de s'élancer. Le fait de se lever et de s'assoir régulièrement revient à déplacer le centre de gravité du passager et donc, revient à faire varier périodiquement la longueur du pendule (ici la balançoire).

L'exercice qui suit ne résoud pas l'équation différentielle. Il cherche simplement à savoir dans quelles conditions la solution est bornée ou non (problème de stabilité). Il est insipré du théorème de Gustave Floquet (1847-1920).


Ex: Pendule et équation de Mathieu

Auteur: Alain Vienne

exerciceExercice

Difficulté : ☆☆   Temps : 2h30

On considère l'équation différentielle suivante (équation de Mathieu):

\ddot{\theta} + \omega_0^2(1+\varepsilon \cos \omega t ) \theta = 0

\theta est une fonction du temps t. Cette équation dépend des paramètres \omega_0, \omega et \varepsilon.

On souhaite déterminer dans quelles conditions la solution est bornée ou non.

Question 1)

Ecrire l'équation de Mathieu comme une équation différentielle du premier ordre:

\dot{Z}(t) = B(t) Z(t)

B(t) est une matrice 2 \times 2 réelle. Donner la signification de la nouvelle inconnue Z (par rapport à \theta).

Question 2)

On se donne une condition initiale Z(0) = x (on a x \in \mathbb{R}^2). D'après le théorème de Cauchy l'équation différentielle a alors une solution unique que l'on note Z_x

Monter que l'application x \mapsto Z_x est linéaire.

Question 3)

Ainsi, à t donné, cette application va de  \mathbb{R}^2 dans  \mathbb{R}^2. Il existe donc une matrice A de \mathcal{M}_{2,2}(\mathbb{R}) telle que Z_x = A x ou encore Z_x(t) = A(t) x . Explciter la matrice A(t) seulement dans le cas \varepsilon = 0.

Question 4)

Pouvoir expliciter A, revient à obtenir toutes les solutions de l'équation différentielle. A est donc une inconnue. Montrer que A vérifie le système suivant:

\dot{A}(t) = B(t) A(t)

A(0) = I

I est la matrice identité.

Question 5)

Montrer que pour tout système de la forme

\dot{A}(t) = B(t) A(t)

, on a le résultat suivant:

| A(t) | = e^{ + \int_0^t \textrm{trace} (B(u)) \mathrm{d}u}

|A(t)| est le déterminant (d'ordre 2) de A(t) et \textrm{trace} (B(u)) = \sum_{i=1}^n b_{i,i} (somme des éléments diagonaux).

Question 6)

Reprendre la question précédente, pour une système (n\times n). C'est-à-dire pour A et B étant des matrices (n\times n).

Question 7)

Montrer que:

A(t+T) = A(t) \times A(T)

T = \frac{2 \pi}{\omega}

Question 8)

La matrice A(T) s'appelle la matrice de Floquet.

Montrer que les valeurs propres \lambda_1 et \lambda_2 de la matrice de Floquet vérifient:

\lambda_1 \lambda_2 = 1

Question 9)

Ainsi, si ces valeurs propres sont réelles, l'une d'elle en valeur absolue est supérieure à 1. Donc avec A(t+T) = A(t) \times A(T) et une condition initale prise dans la direction du vecteur propre associé à cette valeur propre, la solution tendera vers l'infini.

Montrer que sinon (valeurs propres complexes), les solutions sont bornées.

Question 10)

Dans notre cas (équation de Mathieu), \varepsilon est petit. On peut alors montrer que la somme des valeurs propres de A(T) est la trace de la matrice  \left( \begin{array}{cc} \cos \omega_0 T & \frac{1}{\omega_0}\sin \omega_0 T       \\ -\omega_0 \sin \omega_0 T  & \cos \omega_0 T \end{array} \right) (ie: correspondant à celle trouvée dans le cas de perturbation nulle). Donc

\lambda_1 + \lambda_2 = 2 \cos \omega_0 T

Montre que des solutions non bornées sont possibles que si la période T du forçage est telle que

\omega_0 T = k \pi

k est un entier relatif.


Longueur de Debye

Auteur: Arnaud Beck

Un plasma est une collection de particules chargées. Pour simplifier, considérons qu'il n'est composé que d'électrons de charge -e et d'ions de charge +Ze.

Un ion, considéré comme ponctuel, lorsqu'il est dans le vide crée autour de lui un potentiel \phi=\frac{Ze}{4\pi\epsilon_{\rm 0}r}r est la distance à l'ion. Ce potentiel est appelé le potentiel Coulombien.

Dans un plasma, il en va différemment. En effet, il va attirer autour de lui des charges de signe opposé (les électrons) qui vont écranter son potentiel. La sphère d'électrons qui se forme autour de l'ion est appelée la sphère de Debye et son rayon est appelé la longueur de Debye. C'est un paramètre fondamental en physique des plasmas.

remarqueRemarque

Dans cet exercice, on propose de retrouver la valeur de ce rayon et la forme du potentiel à l'intérieur de la sphère de Debye.


Ex: Détermination du potentiel dans une sphère de Debye

Auteur: Arnaud Beck

exerciceDétermination du potentiel dans une sphère de Debye

Question 1)

On considère un ion en r=0 et soit n_{\rm 0} la densité ionique moyenne dans le plasma. Si le plasma est suffisamment chaud, on peut montrer que la densité électronique est égal à

n_{\rm e}(r)=n_{\rm 0}Z\left(1+\frac{e\phi(r)}{k_{\rm B}T}\right)

\phi(r) est le potentiel en r, T la température du plasma et k_{\rm B} la constante de Boltzmann.

Par ailleurs, l'équation de Poisson relie la densité de charge \rho et le potentiel \phi de la manière suivante:

\Delta(\phi)=-\frac{\rho}{\epsilon_{\rm 0}}

1) Écrire l'équation différentielle vérifiée par le potentiel \phi (r) sous la forme a(r)\phi ''+b(r)\phi ' +c(r)\phi=0

2) Trouver un changement de variable \psi=f(\phi) tel que l'équation différentielle du second ordre vérifiée par \psi soit à coefficients constants.

3) Trouver la forme du potentiel \phi. Les conditions aux limites sont que le potentiel \phi doit tendre vers 0 lorsque r tend vers l'infini et il doit être équivalent au potentiel Coulombien lorsque r tend vers 0. En déduire la distance caractéristique d'écrantage de la charge centrale (longueur de Debye) dans ce cas.


Le problème de 2 corps

Auteur : Marc Fouchard

En mécanique céleste le premier problème à résoudre est le problème de deux corps. Ce problème consiste à trouver les trajectoires de deux corps s'attirant l'un l'autre suivant le principe universelle de la gravitation établi par Newton.

Si on considère deux corps ponctuels C_1 et C_2 de masses respectives m_1 et m_2, isolés de toute autre influence, alors l'équation du mouvement de C_2 par rapport à C_1 est:

\frac{{\rm d}^2{\mathbf r}}{{\rm d}t^2} + \frac{\mu {\mathbf r}}{r^3}=0

\mu = G (m_1 + m_2) avec G la constante universelle de la gravitation, et {\mathbf r}={\mathbf r_2} - {\mathbf r_1} avec {\mathbf r}_1 et {\mathbf r}_2 désignant les vecteurs positions des corps C_1 et C_2 dans un repère inertiel.

Le but de l'exercice est donc de résoudre cette équation.

complementExercices reliés

De nombreux exercices sur le problème de 2 corps existent sur ce site. On en trouvera, entre autres, sur l'équation de Kepler et son inversion, sur les solutions géométriques du problème de 2 corps, sur le problème de 2 corps perturbé et sur l'excentricité limite dans les développements du problème de 2 corps.


Ex : le problème de 2 corps

Auteur: Marc Fouchard

exercice Le problème de 2 corps

Difficulté : ☆☆   Temps : 2h

Question 1)

Montrer que {\mathbf h}={\mathbf r} \land \frac{{\rm d} {\mathbf r}}{{\rm d}t} est une constante du mouvement. Cette constante s'appelle l'intégrale du moment angulaire.

Question 2)

En utilisant les coordonnées polaires (r,\theta)r est la norme de {\mathbf r} et \theta est l'angle en radian entre une direction fixe et {\mathbf r} compté positivement dans le sens trigonométrique, montrer que la norme du moment angulaire s'écrit h=r^2\dot{\theta}, où le point (\dot{}) désigne la première dérivée par rapport au temps. Cette équation correspond à la deuxième loi de Kepler.

Question 3)

En multipliant scalairement l' équation du mouvement par \dot{\mathbf r} (qui n'est rien d'autre que le vecteur vitesse), montrer que C=\frac{1}{2}v^2 - \frac{ \mu}{r} est une constante du mouvement (v désignant la norme du vecteur vitesse). C s'appelle l'intégrale de l'énergie.

Question 4)

En utilisant les coordonnées polaires montrer que l' équation du mouvement revient à résoudre le système :

\ddot{r}-r\dot{\theta}^2=-\frac{\mu}{r^2}

\frac{1}{r}\frac{{\rm d}}{{\rm d}t} (r^2 \dot{\theta}) = 0

On remarquera que la deuxième équation correspond à l'intégrale du moment angulaire.

Question 5)

Soit u=\frac{1}{r} , exprimer \dot{r} et \ddot{r} en fonction de h, u, et les dérivées première et seconde de u par rapport à \theta que l'on notera u ' et u ''.

Question 6)

En faisant le changement de variable u=1/r dans l'équation différentielle du second ordre obtenue pour r, écrire une équation linéaire du second ordre pour u en considérant u comme une fonction de \theta.

Question 7)

Résoudre l'équation obtenue en donnant une solution sous la forme A+H \cos (B\, \theta-\omega)A et B sont des constantes que l'on déterminera et H et \omega des constantes d'intégrations.

Question 8)

Montrer que la solution génrérale de cette équation peut s'écrire: r=\frac{p}{1+e\cos(\theta-\omega)} avec p=h^2/\mu, e=H\,h^2/\mu et H et \omega sont deux constantes d'intégration.

remarqueRemarque

Pour 0< e < 1, on pourrait montrer que dans ce cas la solution correspond à une ellipse d'excentricité e et de demi-grand axe a=\frac{p}{2(1-e^2)} mais ceci fait l'objet d'un autre exercice.

On peut cependant remarquer que dans ce cas les valeurs minimale et maximale de r sont r_{\rm min}=p/(1+e) et r_{\rm max}=p/(1-e) et sont obtenues pour \theta-\omega=0 et \theta-\omega=\pi respectivement. Ces positions sont appelées péricentre et apocentre respectivement. Elles sont à l'oposées l'une de l'autre, \omega donnant la direction du pericentre et \omega+\pi celle de l'apocentre. La distance séparant ces deux positions est donc r_{\rm min}+r_{\rm max}=\frac{p}{1-e^2}=2\,a, où a est ce qu'on appelle le demi-grand axe.


Train gravitationnel

Auteur: S. Renner

Date de création: 14 décembre 2009

L'accélération de la pesanteur dépend de la distance au centre de la Terre. Dans l'exercice qui suit, on va utiliser cette propriété pour imaginer un moyen de transport très rapide: en perçant un tunnel rectiligne entre 2 points A et B quelconques de la surface terrestre, un train roulant sans frottement dans ce tunnel pourrait parcourir très rapidement la distance entre A et B. La durée du trajet, de 42 minutes environ, est même indépendante des points A et B choisis.


Ex: Train gravitationnel

Auteur: S. Renner

exerciceTrain gravitationnel

Difficulté :    Temps : 1h

On assimile la Terre à une sphère sans rotation de rayon R_T=6378 km et de masse volumique uniforme \rho = 5.5 \times 10^3 {\rm kg.m}^{-3}. Soit G=6.67 \times 10^{-11} S.I. la contante de gravitation universelle. On imagine un tunnel rectiligne entre 2 points A et B quelconques de la surface terrestre, et un train roulant sans frottement dans ce tunnel. Partant de A sous l'action de la pesanteur, le wagon va accélérer jusqu'au milieu du tunnel, puis décélérer une fois atteinte la distance de moindre approche du centre 0 de la Terre (voir figure). Le train atteindra-t-il le point B, et si oui, en combien de temps?

tunnel_2.jpg
Tunnel rectiligne entre 2 points A et B de la surface terrestre. Le train est repéré par la coordonnée x, sa distance au centre de la Terre est notée r.
Crédit : Astrophysique sur Mesure / Renner
Question 1)

Soit x la distance du train au milieu du tunnel. Exprimer x en fonction de la distance au centre de la Terre r et de l'angle \alpha.

Question 2)

Donner l'expression de la force gravitationnelle F agissant sur le train en fonction de la masse du train m, de la masse volumique de la Terre \rho et de la distance au centre de la Terre r.

Question 3)

En déduire l'équation du mouvement du train dans le tunnel.

Question 4)

Le train peut-il atteindre le point B, et si oui, en combien de temps?


Pendule de Foucault

Florent Deleflie & Alain Vienne

Date de création: 21décembre 2010

Le pendule de Foucault est une expérience conçue pour mettre en évidence la rotation de la Terre, depuis un site terrestre d'observation. Son principe est basé sur la force de Coriolis qui existe dans tout réferentiel non galiléen, comme le référentiel terrestre d'observation. La réalisation de l'expérience est facilitée si la longueur du pendule est grande, comme sous le dôme d'une cathédrale par exemple. La première démonstration publique a eu lieu en 1851, sous la voûte du Panthéon, à Paris.

L'animation ci-dessous tient compte de toutes les forces sans les approximations qui seront faites dans l'exercice suivant.

Pendule de Foucault application.png


Ex: Pendule de Foucault

Auteur: Auteurs : Alain Vienne, Florent Deleflie.

exercicePendule de Foucault

Difficulté : ☆☆   Temps : 1h

Le but de cet exercice est de reprendre la modélisation du pendule en tenant compte du fait que le repère terrestre n'est pas galiléen, mais est animé d'un mouvement de rotation de la Terre elle-même. La véritable motivation de l'exercice est la résolution de l'équation différentielle du mouvement. On considère un pendule constitué d'un fil PM accroché en P de longueur L et d'une boule Mde masse m. L'espace est rapporté au système d'axes (Oxyz) fixe dans le réferentiel lié à la Terre, l'axe (Oz) passant par P et le centre de la Terre (voir figures). On se place dans le cas de faibles oscillations. On rappelle que dans ce cas, le mouvement de la boule M de coordonnées (x,y,z) se fait dans le plan (Oxy) et que le module de la tension du fil \overrightarrow{T} est T=mgg est l'accélération de la pesanteur. On note \overrightarrow{\Omega} le vecteur rotation de la Terre et \lambda la latitude du lieu. Le bilan des forces doit faire intervenir la force d'inertie de Coriolis, dont l'expression est \overrightarrow{f}_{ic} = -2m \overrightarrow{\Omega} \wedge \overrightarrow{v}\overrightarrow{v} désigne la vitesse dans le repère (OXYZ). Par contre, il n'y a pas lieu de tenir compte de la force d'inertie d'entrainement, car celle-ci est déjà incluse dans la définiton de la pesanteur, i.e. dans l'expression de m \overrightarrow{g}. Une fois le bilan des forces effectué, on peut montrer que les composantes de la tension du fil dans (Oxyz) sont \frac{mg}{L}\Big(-x,-y,L\cos \theta\Big) et que les composantes de la force de Coriolis sont 2m\Omega\Big( \dot{y}\sin \lambda, -\dot{x}\sin \lambda, -\dot{y}\cos \lambda\Big). D'après l'hypothèse faite sur la petitesse des oscillations, il n'y a pas lieu de considérer l'équation obtenue par projection sur (Oz), celle-ci pouvant être considérée comme un terme correctif. On ne considère donc que les projections selon les deux autres directions, et en posant u=x+\i y\i = \sqrt{-1}, et en posant aussi \Omega_0 = \Omega \sin \lambda \mbox{ et } \omega_0=\sqrt{\frac{g}{L}} on peut montrer que l'équation du mouvement, complexe, se met sous la forme : \ddot{u}+2\i \Omega_0 \dot{u} + \omega_0^2u=0.

Schéma du pendule dans l'espace
figures-pendule-foucault/pendule-espace.pngfigures-pendule-foucault/repere_coupe.png
Le pendule vu dans le repère de l'observateur, et en coupe verticale.
Crédit : Astrophysique sur Mesure / Vienne
Auteur: A. Vienne, F. Deleflie

exercicePendule de Foucault

L'équation différentielle du mouvement qu'on se propose de résoudre est : \ddot{u}+2\i\Omega_0\dot{u}+\omega_0^2u=0

Question 1)

Ecrire le discriminant réduit de cette équation.

Question 2)

Déterminer les solutions de l'équation caractéristiques, et en déduire la forme générale de la solution de l'équation différentielle.

Question 3)

Particulariser la solution précédente en considérant qu'à l'instant initial, le mobile se trouve en (x_0,0,0) avec une vitesse initiale nulle.

Question 4)

Dans les conditions de l'expérience de Foucault faite au Panthéon en 1851, on a les valeurs suivantes: L=67m, g=9,81 m/s2, et \lambda = 49^\circ Nord. Justifier que \Omega_0 < \! \! \!< \omega_0.

Question 5)

Simplifier l'expression de la solution trouvée en négligeant  \Omega_0 devant  \omega_0. Interpréter.


Topologie et espace métrique


Introduction

On trouvera dans cette partie les exercices suivants :


Théorème du point fixe

Auteur: Marc Fouchard

Loi de Wien

Auteur: Marc Fouchard

La loi de Planck montre que pour un corps noir, l'énergie émise par rayonnement suivant une longueur d'onde, ne dépend que de la température de surface du corps noir. Cette loi est donnée par la relation suivante :

:E(\lambda)=\frac{2 h c^2}{\lambda ^5}\cdot \frac{1}{\exp \left( \frac{h c}{k \lambda T} \right) -1}

c correspond à la vitesse de la lumière dans le vide, hest la constante de Planck, kla constante de Boltzmann, \lambdala longueur d'onde à laquelle le rayonnement est émis et Tla température de surface du corps noir.

La figure ci dessous montre le comportement de E(\lambda) pour différente température de surface du corps noir. On peut remarquer que le maximum de la courbe se déplace sur la gauche lorsque la température augmente. Autrement dit, la longueur d'onde \lambda_{\rm max} pour laquelle le rayonnement émis est maximal diminue lorsque la température de surface augmente.

Loi de Planck application.png

Le but de cet exercice est de trouver la relation exacte entre \lambda_{\rm max} et T.

remarqueRemarque

Cette exercice repose sur la détermation du maximum d'une fonction sur un intervalle donné.


Ex : loi de Wien

Auteur: Marc Fouchard

exerciceloi de Wien

Difficulté : ☆☆   Temps : 1h

Question 1)

Sachant que h, c et ksont des constantes strictement positives et que la température Tétant mesurée en degré Kelvin est aussi strictement positive, montrer que E(\lambda) est de classe {\mathcal C}^{\infty} sur ]0,+\infty[ et est toujours strictement positive sur cet intervalle.

Question 2)

Montrer que les limites de E(\lambda) quand \lambdatend vers 0 et vers + \inftysont toutes les deux égales à zéro.

Question 3)

En déduire qu'il doit exister un maximum pour E(\lambda) sur \left\rbrack 0 ; +\infty \right\lbrack.

Question 4)

En effectuant le changement de variable u=\frac{h c}{k \lambda T}, montrer qu'étudier le signe de \frac{{\rm d} E(\lambda)}{{\rm d} \lambda}revient à étudier celui de \frac{{\rm d} E(u)}{{\rm d} u}.

Question 5)

En déduire une condition sur u, de la forme f(u)=u, pour que \frac{{\rm d} E(u)}{{\rm d} u} s'annule. On note u_S la solution de cette équation lorsqu'elle existe.

Question 6)

Montrer que fest une application contractante sur l'intervalle fermé \left\lbrack 4 ; 6 \right\rbrack.

Question 7)

En déduire l'existence d'un point fixe unique de fdans cet intervalle. Constuire une suite récurente convergent vers ce point fixe. En déduire une valeur \tilde{u}_S qui soit une valeur approchée de u_S à 10^{-6} prêt.

Question 8)

En déduire la relation \lambda_{\rm max}\cdot T =Ac=299\,792\,000~ {\rm m}\cdot{\rm s}^{-1}, h = 6,626\,17\times 10^{-34}~{\rm J}\cdot{\rm s} etk = 1,380\,66 \times 10^{-23}~{\rm J}\cdot{\rm K}^{-1}. Cette relation correspond à la loi de Wien pour les corps noirs. Justifier l'utilisation de \tilde{u}_S dans le calcul de la constante A.


Fonctions usuelles


Introduction

On trouvera dans cette section des exercices concernant :


Exponentielles et logarithmes

Auteurs: Alain Vienne, Stéphane Erard

La magnitude des étoiles

Auteur: Alain Vienne

Les anciens classaient les étoiles suivant leur "grandeur". Cette grandeur correspond à l'éclat tel que le percevaient les anciens: ils observaient des étoiles de "première grandeur", de "deuxième grandeur", etc ... On appelle aussi ces grandeurs magnitudes et on la note m: m=0 pour l'étoile la plus brillante du ciel (Véga de la constellation de la Lyre) et m=5 est souvent considéré comme la limite des étoiles visibles à l'oeil nu. Avec les plus grands télescopes actuels, on peut voir jusqu'à la magnitude 30. A l'inverse, le Soleil qui est très "éclatant" a une magnitude -27.

Voir aussi le cours AMC

En fait, la perception visuelle suit une échelle logarithmique par rapport au phénomène physique correspondant. Pour conserver la classification des anciens, la magnitude ou magnitude apparente (puisque que c'est la magnitude qui nous "apparaît" de l'endroit où on observe) est définie par:m=-2,5 \log \frac{e}{e_0}e est l'éclat de l'astre que l'on observe. e_0 est l'éclat de l'étoile Véga qui est ainsi prise en référence (pour assurer que sa magnitude apparente est 0). On rappelle que la notation \log désigne le logarithme en base 10.

Il est clair que l'éclat est d'autant plus important que l'observateur est proche de la source lumineuse. Plus précisément, on a e=\frac{P}{4\pi d^2}P est la puissance totale émise par l'astre et d est sa distance.

Pour caractériser la brillance intrinsèque d'un astre, on utilise la magnitude absolue, notée M. C'est la magnitude qu'aurait cet astre si il était observé à la distance de 10 parsecs. On a donc pour un même astre : m=-2,5 \log \frac{P}{d^2} + C et M=-2,5 \log \frac{P}{d_0^2} + C avec d_0=10pc

On rappelle que le parsec est la distance pour laquelle on voit le rayon de l'orbite de la Terre (1 UA) sous l'angle de 1" de degré. Ainsi 1pc=206265UA (de la même manière qu'il y a 206265 " dans un radian).


Ex: la magnitude des étoiles

Auteur: Alain Vienne

exerciceMagnitude du Soleil vu de alpha du Centaure

Difficulté :    Temps : 30mn

Question 1)

Vu de la Terre, le Soleil a une magnitude apparente égale à -27. Calculer la magnitude apparente qu'aurait le Soleil s'il était observé depuis l'étoile alpha du Centaure. La parallaxe de cette étoile est \varpi = 0'',76.

Auteur: Alain Vienne

exerciceMagnitudes absolues du Soleil et de Véga

Difficulté :    Temps : 20mn

Question 1)

Calculer la magnitude absolue du Soleil et celle de Véga (dont la parallaxe est \varpi = 0,''125)

Auteur: Alain Vienne

exerciceMagnitude de l'amas des pleïades

Difficulté :    Temps : 30mn

Question 1)

L'amas des pléiades contient 7 étoiles visibles à l'oeil nu:

Etoile magnitude
Alcyone 3,00
Atlas 3,80
Electra 3,80
Maia 4,00
Merope 4,30
Taggeta 4,40
Pleione 5,00

Calculer la magnitude globale de l'amas.

Auteur: Alain Vienne

exerciceVisibilités des satellites de mars

Difficulté :    Temps : 20mn

Question 1)

A partir de quelle distance à la planète Mars, un voyageur vers cette planète pourra-t-il voir à l'oeil nu les satellites de Mars? On donne les magnitudes de Phobos et Deimos vus de la Terre à l'opposition de Mars: m_P = 11,3 et m_D = 12,4. On supposera que l'orbite de Mars est un cercle de rayon 1,524 UA.

Auteur: Alain Vienne

exerciceMagnitude apparente d'une planète

Difficulté : ☆☆   Temps : 50mn

Question 1)

Lors de l'opposition, une planète extérieure est vue depuis la Terre avec la magnitude m_0. Exprimez la magnitude de cette planète lorsqu'elle est à la distance \Delta de la Terre et à la distance d du Soleil. On donnera cette expression en fonction de m_0, d et \Delta (on négligera l'effet de phase).

Question 2)

Application à Jupiter pour lequel m_0 = -2,5 et d=5,2UA, puis à Mars pour lequel m_0 = -2,0 et d=1,52UA: Calculer la magnitude de ces planètes lorsqu'elles sont en quadrature.

Auteur: Alain Vienne

exerciceMagnitude instrumentale

Difficulté : ☆☆   Temps : 60mn

Question 1)

On observe à l'oeil nu une étoile de magnitude apparente m. On l'observe ensuite au travers d'un instrument dont le diamètre d'ouverture est D avec une pupille de sortie dont le diamètre \delta est égal à celle de l'oeil.

Quelle est la magnitude instrumentale m_{inst} de cette étoile au travers de cet instrument.

Question 2)

Quelle est la magnitude limite observable avec cet instrument?

Question 3)

Faire l'application numérique avec les télescopes d'ouverture suivante: 5cm, 20cm, 1m, 8 m. On prendra \delta= 6mm


Désintégration radioactive

Auteur: Stéphane Erard

Les éléments chimiques les plus légers sont formés au début de l'univers, les plus lourds sont formés essentiellement dans les étoiles.

Tous ne sont pas stables. Un radionucléide est un noyau atomique instable qui se désintègre en une autre espèce. La probabilité de désintégration de chaque atome est constante au cours du temps, et les événements sont indépendants.


Ex: désintégration radioactive

Auteur: Stéphane Erard

exerciceDésintégration radioactive

Difficulté :    Temps : 15 min

Question 1)

On considère une seule espèce radioactive. Soit N(t) le nombre d'atomes à l'instant t, quel est le nombre de désintégrations dN pendant l'intervalle de temps dt ?

Question 2)

En déduire le nombre d'atomes présents à l'instant t.

Question 3)

Au bout de quel intervalle de temps t_{1/2} le nombre d'atomes radioactifs est-il réduit de moitié ?

Tracer la courbe d'évolution et sa tangente à l'instant initial. Reporter \lambda et t_{1/2}.

Question 4)

On définit l'activité A comme le nombre de désintégrations par seconde d'une espèce. C'est une grandeur observable, qui se mesure en Becquerels (Bq) dans le Système International. Exprimer celle-ci en fonction du temps.

Auteur: Stéphane Erard

exerciceDatation de météorites

Difficulté :    Temps : 30 min

On utilise la loi de décroissance radioactive pour dater un échantillon de météorite. Les âges étant élevés (de l'ordre de l'âge du Système solaire, ~5 milliards d'années) on utilise des isotopes à longue période.

Le rubidium 87 décroît par radioactivité \beta en strontium 87 avec une demi-vie de 49 milliards d'années, selon la réaction suivante :

 ^{87} Rb  \Rightarrow ^{87} Sr + e^- + \overline{\nu}

Un des neutrons se transforme en proton (radioactivité \beta). Le nombre de masse (87) est inchangé, le nombre de charges varie (de 37 à 38). La charge totale est conservée par l'émission d'un électron. La quatrième particule est un anti-neutrino symétrique de l'électron, dont la présence est requise par la conservation du moment cinétique.

Question 1)

Ecrire la quantité de ^{87} Sr à l'instant de la mesure t en fonction des quantités de ^{87} Sr initiale et de ^{87}Rb actuelle et initiale.

Question 2)

Récrire cette équation pour éliminer une des quantités inconnues.

En pratique, on mesure des rapports d'abondance ; en l'occurrence on rapporte toutes les abondances à celle du ^{86} Sr, isotope stable du strontium qui n'est pas un produit de désintégration (son abondance n'est donc pas fonction du temps). Faire apparaître ces rapports. Commentaires ?

Question 3)

On lève l'indétermination précédente en effectuant cette mesure sur différents minéraux présents dans la même météorite, et formés au même moment. Reporter les points de mesures attendus sur un graphique dérivé de la fonction précédente.

Question 4)

Les mesures des rapports isotopiques dans l'exemple sont les suivantes :

Rapports isotopiques
^{87}Rb/^{86}Sr ^{87}Sr/^{86}Sr
0.059 0.703
0.137 0.708
0.158 0.709
0.295 0.718
0.323 0.720
0.376 0.724
0.386 0.724

Trouver un ordre de grandeur de l'âge de la météorite à l'aide des chiffres fournis. Que mesure-t-on exactement avec cette méthode ?


Fonctions hyperboliques et inverses

Auteurs: Jérôme Thiébaut, Marc Fouchard

Univers à courbure négative

Auteur: Jérôme Thiébaut

Les équations d'Einstein de la relativité générale appliquées à l'univers que l'on suppose être un fluide homogène et isotrope, aboutissent à l'équation de Friedmann,

(dtemps(a;1)/a)^2=H_0^2*(Omega_m*(a_0/a)^3+Omega_r*(a_0/a)^4+Omega_Lambda+Omega_K*(a_0/a)^2),

décrivant l'évolution de l'univers en fonction de son contenu. Ce contenu est défini par les paramètres de densité de matière, Omega_m, de rayonnement, Omega_r, de constante cosmologique, Omega_Lambda et de courbure, Omega_K. H_0 est la constante de Hubble et a est le facteur d'échelle décrivant l'évolution de l'univers. La composition de l'univers évoluant avec le temps, les différents paramètres de densité ont des importances relatives différentes en fonction de l'ère cosmologique considérée. Ils sont tour à tour dominants (Omega_r puis Omega_m et Omega_K et enfin Omega_Lambda) ou négligeables. On se propose dans cet exercice d'étudier un modèle d'univers dominé par la matière avec une courbure négative et de vérifier si il peut coïncider avec les observations actuelles.


Ex: Univers à courbure négative

Auteur: Jérôme Thiébaut

exerciceUnivers à courbure négative

Difficulté : ☆☆   Temps : 30 mn

On considère un univers dominé par la matière non relativiste et avec une courbure négative. Dans ce cas, l'équation de Friedmann s'écrit:

(dtemps(a;1)/a)^2=H_0^2*Omega_m*(a_0/a)^3+H_0^2*Omega_K*(a_0/a)^2

H_0est la constante de Hubble, Omega_mle paramètre de densité et Omega_Kle paramètre de courbure. La solution sous une forme paramétrique est: a=A*(cosh(Theta)-1), t=B*(sinh(Theta)-Theta), où A et B sont des constantes.

Question 1)

Dériver a et t par rapport au temps et éliminer la dépendance en dtemps(Theta;1) de dtemps(a;1).

Question 2)

Calculer les constantes A et B comme fonction de la constante de Hubble et des paramètres de densité et de courbure.

Question 3)

Calculer le paramètre de décélération q défini comme: q=-a*dtemps(a;2)/dtemps(a;1)^2 . Les observations actuelles montrent que l'univers est dans une phase d'accélération. Ce type d'univers a t'il une phase accélérée ? Peut-il représenter notre univers ?


Equation de Kepler hyperbolique

Auteur : Marc Fouchard.

Le but de cet exercice est de résoudre l'équation de Kepler dans le cas hyperbolique. On a déjà vu ici comment résoudre l'équation de Kepler dans le cas elliptique. On va voir ci une méthode similaire pour une trajectoire hyperbolique. Dans ce cas l'équation de Kepler est :

M=e \sinh E-E ,

M est l'anomalie moyenne, e est l'excentricité (qui est >1 dans le cas hyperbolique) et E est l'anomalie excentrique. On peut voir ici une animation avec le lien entre les trois anomalies dans le cas hyperbolique. (f correspond à l'anomalie vraie)


Ex: équation de Kepler hyperbolique

Auteur: Marc Fouchard

exerciceéquation de Kepler hyperbolique

Difficulté : ☆☆   Temps : 1h

Question 1)

Soit f la fonction définie par f(x)=e \sinh x - x - M sur \mathbb{R} avec M une constante. Montrer que f est continue dérivable 2 fois, que f' est strictement supérieure à zéro et que f'' est supérieure à zéro pour x>0. On rappelle que dans le cas hyperbolique e>1.

Question 2)

Soit r, le nombre réel positif tel que f(r)=0. Montrer que pour x\ge r, f(x)>0.

Question 3)

Montrer que la courbe représentative de f est au-dessus de sa tangente sur [r,+\infty[.

Question 4)

En déduire que la suite \lbrace u_n \rbrace définie par:

u_{n+1}=u_n-\frac{f(u_n)}{f'(u_n)},

avec u_0\ge r, est décroissante et minorée par r.

Question 5)

En déduire que la suite \lbrace u_n \rbrace converge et que sa limite est r.

Cette propriété de la suite \lbrace u_n\rbrace est utilisée pour résoudre par itération et de manière approchée l'équation de Kepler.


Fonctions trigonométriques et inverses

Auteur: Marc Fouchard

Astrolabe

Auteur: Marc Fouchard

astrolabe
image/Astrolabe_dsc03864.jpg
Astrolabe du XVI siècle.
Crédit : David Monniaux, licence : CC BY-SA 3.0

L'astrolabe est un outil astronomique permettant de représenter la partie du ciel observée en fonction de la date et de l'heure pour un lieu donné. Il permet ensuite de faire différentes mesures comme la détermination des heures de lever et de coucher d'un astre. Les applications de l'astrolabe sont pourtant très nombreuses. Pour avoir plus de détails, on pourra aller voir l'astrolabe.

La construction d'un astrolabe repose sur la projection stéréographique. Le but de cet exercice est d'étudier les propriétés principales de cette projection et d'en déduire l'image de points et de cercles caractéristiques de la sphère céleste.

 

 

Ce sont ces constructions qui ont permis de construire l'animation suivante.

astrolabe application.png

prerequissphère céleste

Sur une sphère céleste un point est repéré par une longitude et une latitude. La latitude correspond à un angle entre [-\pi/2,\pi/2] donnant la hauteur au dessus d'un grand cercle de référence et en choisissant un côté positif (comme sur Terre, la latitude d'un lieu correspond à une hauteur au-dessus de l'équateur, comptée positivement dans l'hémisphère nord). Ce cercle de référence permet de définir l'axe des pôles et les pôles. La longitude correspond à l'angle, compris dans l'intervalle [0,2\pi], entre un méridien de référence et le méridien passant par le point considéré. Cet angle est mesuré sur le cercle de référence en choisissant un sens positif. Sur la Terre les longitudes sont mesurées à partir du méridien passant par Greenwich en prenant comme sens positif la direction de l'Ouest (ce qui correspond au sens des aiguilles d'une montre lorsqu'on regarde du pôle nord).

Dans notre cas, la sphère céleste \mathcal{S} est une sphère de rayon unité (arbitraire), centrée sur l'observateur. Sur cette sphère on projette l'équateur terrestre, ce qui nous donne un grand cercle , appelé équateur céleste, le pôle nord se projette au point P et le pôle sud au point P' , appelés respectivement pôle céleste nord et pôle céleste sud.

En astronomie différents ensembles de longitude et de latitude sont utilisés :

La figure ci-dessous montre la correspondance entre ces différents systèmes de coordonnées. On remarquera aussi sur la figure le lien entre la position du zénith et la latitude terrestre du lieu.

sphère céleste
image/sphere-celeste.png
Les coordonnées utilisées en astronomie. Le grand cercle vert correspond à l'horizon céleste et le rouge à l'équateur céleste. La hauteur du pôle céleste nord sur l'horizon correspond à la latitude du lieu. Le point \gamma correspond à la direction du point vernal.
Crédit : Astrophysique sur Mesure / Marc Fouchard

En un lieu de latitude \phi on a alors les équations suivantes permettant de passer d'un ensemble de coordonnées à l'autre:

\begin{array}{rcl}\sin h&=& \sin \phi \sin \delta + \cos \phi \cos \delta \cos H \\ \cos h \sin  A &=& \cos \delta \sin  H \\ \cos h \cos A &=& -\sin \delta \cos \phi + \cos \delta \sin \phi \cos H \\ && \\ \sin \delta &=& \sin h \sin \phi - \cos h \cos \phi \cos A \\ \cos \delta \sin H &=& \cos h \sin A \\ \cos \delta \cos H &=& \sin h \cos \phi + \cos h \sin \phi \cos A \end{array}

Le lien entre l'ascension droite et l'angle horaire se fait en utilisant l'angle horaire du point vernal appelé temps sidéral, que l'on notera \tau. On a la relation : \tau=\alpha+H.


Ex : astrolabe

Auteur: Marc Fouchard

exerciceAstrolabe

Difficulté : ☆☆   Temps : 2h

Question 1)

L'astrolabe est basée sur la projection stéréographique d'une sphère sur un plan. On considère ici la projection de pôle céleste sud P'. Un point E de la sphère aura pour image, le point E' intersection de la droite (P'E) avec le plan \mathcal{P} passant par le centre de la sphère O et perpendiculaire à la droite (PP') reliant les pôles. La figure suivante illustre cette projection.

projection
proj_stereo1.png
Projection stéréographique de pôle P'.
Crédit : Astrophysique sur Mesure / Marc Fouchard

Quelle est l'image du pôle P ? Quelles sont les points invariants par cette projection ?

Question 2)

On définie un repère sur le plan \mathcal{P}, centré en O, l'axe des abscisses est dirigé vers l'origine des angles horaires sur l'équateur céleste et l'axe des ordonnées fait un angle de \pi/2 dans le sens trigonométrique vu du pôle céleste Sud. On utilisera alors un système de coordonnées polaires (r,\theta) pour placer un point sur \mathcal{P}.

Soit E un point de \mathcal{S}, de coordonnées (\lambda,\delta)\lambda est l'angle horaire et \delta la déclinaison. Montrer que les coordonnées polaires de son image E' sont \left(\tan\left(\frac{\pi}{4}-\frac{\delta}{2}\right),\lambda\right).

Question 3)

Montrer que la projection d'un cercle \mathcal{C} passant par le pôle céleste Sud P' est une droite.

Question 4)

Soit M et N deux points de \mathcal{S}. Soit \mathcal{C} le cercle de \mathcal{S} passant par ces points. On suppose que \mathcal{C} n'est pas un grand cercle. Il existe donc un cône de sommet A tangent à \mathcal{S} en \mathcal{C}. Soit M' l'image de M par projection sur \mathcal{P}. La droite (P'A) coupe le plan \mathcal{P} en A'. Enfin, on appelle \mathcal{Q}, le plan tangent à \mathcal{S} en P'. La droite (AM), coupe le plan \mathcal{Q} en m, et la droite (P'M) coupe le plan parallèle à \mathcal{Q} passant par A en M''. La figure ci-dessous montre la construction.

projection d'un cercle
proj-cercle.png
Projection d'un cercle.
Crédit : Astrophysique sur Mesure / Marc Fouchard.

Montrer que Om=Mm. En déduire que AM=AM'' puis que A'M'=\frac{P'A'}{P'A}\cdot AM. En déduire que l'image de \mathcal{C} est un cercle.

Question 5)

On suppose maintenant que le cercle \mathcal{C} est un grand cercle. Les tangentes en tout point de \mathcal{C} sont maintenant parallèles. A n'est plus défini, mais on peut encore construire le plan \mathcal{Q} et le point m. On appelle A' le point où la parallèle à (Mm) passant par P' coupe la plan \mathcal{P} et M'', le point où la droite (Mm) coupe le plan \mathcal{P}. Voir la figure ci-dessous.

projection d'un grand cercle
proj-gd-cercle.png
Projection d'un grand cercle.
Crédit : Astrophysique sur Mesure / Marc Fouchard

Montrer que A'M'=A'P'. En déduire de nouveau que l'image de \mathcal{C} est un cercle.

Question 6)

On définit un cercle \mathcal{C} de \mathcal{S} par son centre E et son rayon z qui correspond en fait à l'angle sous lequel est vu le rayon depuis le centre O de la sphère \mathcal{S}. On suppose que z\in]0,\pi/2]. La figure ci-dessous illustre la situation.

cercle sur S
cercle-de-S.png
Cercle de \mathcal{S}.
Crédit : Astrophysique sur Mesure / Marc Fouchard

On considère le grand cercle passant par E, ayant pour coordonnées horaires (\lambda, \delta), et P', coupant \mathcal{C} en M et N. Déterminer les coordonnées horaires de M et N.

Question 7)

Montrer que les images M' et N' de M et N sont diamétralement opposées.

Ceci permet donc de construire facilement la projection du cercle, puisque connaissant M' et N' on peut déterminer le rayon et le centre du cercle projeté.

Question 8)

la construction

Ces propriétés permettent de construire facilement des cercles de latitude constante par rapport à l'équateur, comme les tropiques.

On connaît les coordonnées du Zénith (\delta=\phi et H=0). Ainsi, il est facile de tracer la projection de l'horizon puisqu'il correspond à un cercle de \mathcal{S} de centre Z et de rayon z=\pi/2. De même différents cercles de hauteur constante h par rapport à l'horizon peuvent être obtenus en changeant la valeur de z (on prend z=\pi/2-h).

Les projections des méridiens par rapport à l'équateur sont aussi faciles à tracer puisqu'ils correspondent à des demi-grands cercles passant par le pôle céleste sud P'. Les projections correspondent donc à des demi-droites. On doit juste faire attention au fait qu'en astronomie le méridien d'origine par rapport à l'équateur correspond à celui qui contient le point vernal. Or, l'angle horaire du point vernal, appelé temps sidéral et noté \tau, varie dans le temps. Ainsi les projections des méridiens équatoriaux vont tourner en même temps que \tau.

On souhaite maintenant tracer la direction des points cardinaux Sud, Sud-Ouest, Ouest, Nord-Ouest, Nord, Nord-Est, Est, Sud-Est. Ces directions sont par définition sur l'horizon céleste, et leur azimut (voir la partie sphère céleste dans le préambule) respectives sont 0, \pi/4, \pi/2, 3\pi/4, \pi, 5\pi/4, 3\pi/2, 7\pi/4. On doit donc seulement calculer les coordonnées horaires de ces points pour pouvoir déterminer leur projection. Calculer donc les coordonnées horaires d'un point M de l'horizon céleste d'azimut A en un lieu de latitude \phi.

Question 9)

On veut maintenant construire la projection des méridiens par rapport à l'horizon céleste. Soit donc le méridien d'azimut A et le méridien d'azimut A+\pi. Justifier que ces deux méridiens forment un grand cercle \mathcal{C} de \mathcal{S}, dont on déterminera le centre sur \mathcal{S} et le rayon. Ceci permet de construire facilement la projection des méridiens d'après ce qu'on a vu précédemment.

Question 10)

La trajectoire apparente du Soleil vue depuis la Terre est dans un plan appelé écliptique. L'inclinaison entre le plan de l'écliptique et le plan de l'équateur est constante et est appelée obliquité, notée \varpi. Les variations de l'obliquité sont tellement faibles qu'on peut la supposer constante ici. La droite d'intersection entre ces deux plans passe par le point vernal et le centre de la sphère céleste \mathcal{S}. En s'aidant d'un dessin, déterminer les coordonnées équatoriales, puis les coordonnées horaires du pôle de l'écliptique E ayant une déclinaison positive.

Question 11)

En déduire la méthode pour construire la projection de l'écliptique.

Question 12)

On suppose que le mouvement apparent du Soleil sur l'écliptique se fait de manière uniforme . Sachant que le 22 Mars de chaque année, le Soleil se trouve au point vernal, et que le mouvement se fait à ascension droite croissante au cours de l'année, déterminer les coordonnées équatoriales du Soleil, en fonction de la date du jours. En déduire ses coordonnées horaires.

On utilisera la relation suivante valable dans un triangle sphérique de sommet A, B, C et de côté a, b, c (aétant le côté opposé au sommet A, etc.) où C=\pi/2 (voir la figure ci-dessous) :\begin{array}{rcl}\sin b &=& \sin B \sin c \\ \tan a &=& \cos B \tan c \end{array}.

triangle sphérique
triangle-spherique.png
Crédit : Astrophysique sur Mesure / Marc Fouchard


Carte du ciel

Auteur: Marc Fouchard

Le but de cet exercice est de construire une carte illustrant la partie visible du ciel en un lieu donné en fonction de la date et de l'heure.

On peut voir dans l'animation ci-dessous le résultat final de cet exercice. L'horizon est fixe tandis que le fond d'étoiles fixes et le Soleil défilent à cause de la rotation de la Terre sur elle-même.

carte du ciel application.png

Pour bien comprendre le but de l'exercice, il faut bien assimiler les deux sphères célestes qui interviennent ici. On pourra aller voir cette page où ces deux sphères sont présentées, ainsi que les 3 principaux systèmes de coordonnées utilisés en astronomie.


Ex : carte du ciel

Auteur: Marc Fouchard

exerciceCarte du ciel

Difficulté :    Temps : 2h

Question 1)

L'exercice se fait en deux étapes, la première consiste à construire le profile de l'horizon, et la deuxième à placer sur cet horizon le fond d'étoiles fixes contenant la trajectoire apparente annuelle du Soleil.

On se place en un lieu de latitude \phi. Dessiner sur une sphère céleste avec l'équateur céleste comme plan de référence, le pôle céleste nord, l'horizon céleste, et les points cardinaux Sud, Ouest, Nord et Est. On notera aussi Sle point d'intersection du méridien passant par les pôles célestes nord et sud et par le zénith avec l'équateur céleste. On notera N le point opposé à S sur l'équateur céleste.

On mettra en évidence les angles suivants sur la figure: la déclinaison \deltad'un point M de l'horizon céleste, la latitude, la colatitude \tilde{\phi}= \pi/2-\phi, et l'angle x entre la direction ouest et le méridien équatorial passant par M,et compté positivement vers N.

Question 2)

En utilisant la formule suivante, valable dans un triangle sphérique (voir le dessin ci-dessous), déterminer la déclinaison \delta de M en fonction de xet \phi.

\sin c \cot b = \sin A \cot B + \cos A \cos c.

triangle sphérique
triangle-spherique.png
Triangle sphérique: les côtés correspondent à des arcs de grands cercles. Comme la sphère céleste est de rayon unité, la longueur d'un de ces arcs correspond à l'angle sous lequel est vu cet arc depuis le centre de la sphère.
Crédit : Astrophysique sur Mesure / Marc Fouchard.

Question 3)

Au lieu de l'angle x, on souhaite utiliser un angle associé au Soleil, appelé heure solaire vraie et noté h. Cet angle est mesuré sur l'équateur céleste, à partir de la direction N, et compté positivement vers l'Est. Ainsi, l'Est, S et l'Ouest correspondent respectivement à \pi/2, \pi et 3\pi/2. Ecrire \delta en fonction de h.

Question 4)

Etudier la fonction \delta : h \mapsto \delta(h). On déterminera en particulier la valeur de \deltaet de sa dérivée pour h=0 et \pi/2.

Question 5)

On souhaite maintenant déterminer l'équation de la trajectoire apparente annuelle du Soleil (qui est dans un plan appelé éclitpique) dans un repère où on a l'ascension droite en abscisse et la déclinaison en ordonnées.

La normale au plan de l'écliptique dirigée vers l'hémisphère nord a une direction constante par rapport à la direction du pôle céleste nord P. L'angle entre ces deux directions est appelé obliquité est vaut \varpi=23^\circ26'. Vu du pôle céleste nord, le Soleil décrit sa trajectoire dans le sens trigonométrique, c'est-à-dire que son ascension droite augmente au cours du temps.

Faire un dessin de la sphère des fixes mettant en évidence l'équateur céleste, le grand cercle de l'écliptique et les coordonnées équatoriales du Soleil.

Question 6)

En déduire \delta_\odot en fonction de \alpha_\odot et de l'obliquité \varpi.

Question 7)

Etudier la fonction \delta_\odot : \alpha_\odot \mapsto \delta_\odot(\alpha_\odot). On déterminera en particulier la valeur de \delta_\odotet de sa dérivée pour \alpha_\odot=0 et \pi/2.

Question 8)

Déterminer les coordonnées du Soleil, aux équinoxes et aux solstices.

Question 9)

Il s'agit maintenant de positionner les deux graphes l'un par rapport à l'autre. C'est le Soleil qui fait le lien entre les deux. Il faut d'abord placer le Soleil en fonction de la date du jours. Une fois celui-ci positionné, il est facile de placer le graphe de l'horizon pour une heure solaire vraie donnée, puisque l'abscisse du Soleil sur cette carte correspond à l'heure solaire vraie. Ainsi lorsque le temps s'écoule le graphe de l'horizon va glisser sur le graphe des étoiles fixes et de l'écliptique (ou l'inverse suivant comment on choisi la transparence).

Il faut faire attention au fait que pour le graphe comportant l'horizon céleste, l'axe des abcisses est orienté de la droite vers la gauche, alors que pour la carte des étoiles fixes avec l'écliptique, l'axe des abscisses est orienté de la gauche vers la droite. Les deux axes vont de 0 à 2\pi(en astronomie cependant on préfère noter les longitudes entre 0h et 24h).

Déterminer l'ascension droite du Soleil en fonction de la date du jours (ceci permet finalement de résoudre complètement l'animation présentée dans l'introduction à cet exercice).

On utilisera la relation suivante valable dans un triangle sphérique de sommet A, B, C et de côté a, b, c (aétant le côté opposé au sommet A, etc.) où C=\pi/2 (voir la figure ci-dessous) :\begin{array}{rcl}\sin b &=& \sin B \sin c \\ \tan a &=& \cos B \tan c \end{array}.

triangle sphérique
triangle-spherique.png
Crédit : Astrophysique sur Mesure / Marc Fouchard


Fonctions de plusieurs variables


Introduction

On trouvera dans ce chapitre les exercices suivants :


Dérivations partielles

Auteurs: Jérôme Thiébaut, Marc Fouchard, S. Renner

Entropie

Auteur : Jérôme Thiébaut

La connaissance du contenu de l'univers en terme de particules et d'énergie est indispensable à la compréhension de son évolution. Juste après le Big Bang, l'univers était très chaud et les premières particules présentes n'étaient ni des électrons ni des protons, mais plutôt des quarks, neutrinos... En se diluant, l'univers refroidit et les particules qui le constituent changent et évoluent ensemble puis séparément. La température de l'univers est donc une quantité extrêmement importante. La thermodynamique est la branche de la physique qui permet de traiter ce problème. Elle permet, grâce à ses lois, de relier entre elles diverses quantités fondamentales telles que la température, la pression, l'énergie, la densité de particules, de photons... L'entropie est une de ces quantités. Elle mesure en quelque sorte le degré de désordre d'un système microscopique, où autrement dit, la capacité d'un système de particules à produire ou non des phénomènes collectifs. Le but de cet exercice est d'exprimer l'entropie en fonction de la température de l'univers.


Ex: Entropie

Auteur: Jérôme Thiébaut

exerciceEntropie

Difficulté :    Temps : 10 min

On peut relier la variation d'énergie, epsilon, aux variations d'entropie, S, et de volume, V, par la relation suivante: d*epsilon=T*d*S-P*d*V, où T est la température et P la pression.

Question 1)

Exprimer la variation d'entropie en fonction des variations de volume et de densité volumique d'énergie, rho.

Question 2)

Exprimer la variation d'entropie en fonction des variations de volume et de température.

Question 3)

Montrer que (1/T)*(drond*rho/drond*T)=d((rho+P)/T)/dT.

Question 4)

Exprimer la variation d'entropie comme la variaton d'une seule quantité dépendant de V, P, T et rho puis relier l'entropie à ces variables thermodynamiques.

Question 5)

D'autres calculs thermodynamiques montrent que la densité d'entropie s=frac(2*pi^2;45)*q_(star)*((T))*T^3, où q_star*((T))est un facteur dépendant de la température et des particules présentes. L'unité de volume est V=a^3, où a est le facteur d'échelle mesurant l'expansion de l'univers. Exprimer S comme une fonction de la température T et du facteur d'échelle a.


Formulation hamiltonienne du problème de 2 corps

Auteur : Marc Fouchard

On a vu dans cet exercice comment résoudre le problème de deux corps. Nous allons voir ici, comment obtenir les équations hamiltoniennes de ce problème.

On considère donc un corps ponctuel P de masse unité mobile dans un plan et soumis à l'attraction gravitationnelle d'un corps fixe de masse m se trouvant en O. On suppose que m \gg 1.


Ex: Formulation hamiltonienne du problème de 2 corps

Auteur: Marc Fouchard

exerciceFormulation hamiltonienne du problème de 2 corps

Difficulté :    Temps : 30 mn

Question 1)

On se place dans un repère polaire, centré sur O. On précise que la force universelle de la gravitation s'appliquant au point P est:

\mathbf{F}=-\frac{m\mu}{r^3}\mathbf{OP},

\mu désigne la constante universelle de la gravitation et r=||\mathbf{OP}|| (les notations en gras dénotent des vecteurs).

Déterminer le potentiel dont dérive la force. En déduire l'énérgie potentielle E_p.

Question 2)

Calculer l'énergie cinétique. En déduire le lagrangien et le hamiltonien du système.

Question 3)

Définir les variables conjuguées p_r et p_\theta associées aux coordonnées r et \theta.

Question 4)

En déduire les équations hamiltoniennes du problème.


Les variables de Delaunay

Auteur: Marc Fouchard

On a vu dans l'exercice sur la formulation hamiltonienne du problème de 2 corps, comment écrire les équations de hamilton de ce problème. Cependant, on n'intégrait pas les équations. On va voir ici, qu'en utilisant des variables hamiltoniennes appropriées on peut intégrer le problème très facilement. Ces variables sont les variables de Delaunay.


Ex: les variables de Delaunay

Auteur: Marc Fouchard

exerciceLes variables de Delaunay

Difficulté :    Temps : 10 mn

Question 1)

Les variables de Delaunay sont les coordonnées (l,g,h) associées aux moments conjuguées (L,G,H) (voir ce cours de mécanique céleste ainsi que l' exercice précédent) avec:

\begin{array}{ccc} l=M, & g=\omega, & h=\Omega \\ L=\sqrt{\mu a}, & G=\sqrt{\mu a (1-e^2)}, & H=\sqrt{\mu a (1-e^2)}\cos i \end{array}.

M est l'anomalie moyenne, \omega est l'argument du péricentre, \Omega est la longitude du noeud ascendant, a est le demi-grand axe, e est l'excentricité et i est l'inclinaison.

Sachant que le hamiltonien du problème de deux corps (où on a supposé ici que le corps massif était de masse unité) est: \mathcal H=-\frac{\mu}{2a}, (voir cet exercice sur l'équation de Kepler) en déduire les équations de Hamilton et résoudre le système.


Théorème d'inversion de Lagrange

Auteur: S. Renner

Le théorème d'inversion de Lagrange donne le développement en série d'une fonction définie implicitement. L'application de ce théorème permet entre autres d'obtenir une solution numérique de l'équation de Kepler E = M + e \sin E, ou d'écrire des développements utiles du problème des deux corps.

Voici un énoncé de ce théorème :

Soit y fonction de 2 variables x et \alpha et d'une fonction f infiniment dérivable de la forme : y = x + \alpha f(y) avec \alpha petit.

Alors \displyastyle y = x + \Sigma_{k=1}^\infty \frac{\alpha^k}{k!} \frac{\partial^{k-1}}{\partial x^{k-1}} f^k(x).

On propose ici de le démontrer par une méthode reposant sur les dérivées partielles, révélée par Pierre-Simon Laplace.


Ex: Théorème d'inversion de Lagrange

Auteur: S. Renner

exerciceThéorème d'inversion de Lagrange

Difficulté : ☆☆   Temps : 1H

On va donc démontrer que si y = x + \alpha f(y), alors \displyastyle y = x + \Sigma_{k=1}^\infty \frac{\alpha^k}{k!} \frac{\partial^{k-1}}{\partial x^{k-1}} f^k(x) avec \alpha petit.

Question 1)

Développer y(x,\alpha) au voisinage de \alpha = 0.

Question 2)

Montrer que \frac{\partial y}{\partial \alpha} = f(y) \frac{\partial y}{\partial x}.

Question 3)

Montrer que pour tout entier n strictement positif, \displaystyle \frac{\partial^n y}{\partial \alpha^n} = \frac{\partial^{n-1}}{\partial x^{n-1}} \Big{(} f^n(y). \frac{\partial y}{\partial x}\Big{)}. On utilisera le résultat de la question précédente.

Question 4)

En déduire \displyastyle y = x + \Sigma_{k=1}^\infty \frac{\alpha^k}{k!} \frac{\partial^{k-1}}{\partial x^{k-1}} f^k(x).


Paramètre de Tisserand

Auteur: Marc Fouchard

Date de création: 9 Mai 2013

L'objectif de cet exercice est de déterminer le paramètre de Tisserand qui est une quasi-constante du mouvement pour les comètes observées. Ainsi ce paramètre permet de montrer que l'observation de deux comètes à des époques différentes correspondent en fait au même objet.


Ex: paramètre de Tisserand

Auteur: Marc Fouchard

exerciceParamètre de Tisserand

Difficulté :    Temps : 1h30

On considère un problème de trois corps où les deux premiers   P_1 et P_2, appelés primaires et de masse respective m_1 et m_2, sont sur des orbites circulaires et uniforme ; et le troisième P, de masse négligeable voit sa trajectoire affectée par les primaires alors que celui-ci n'affecte pas le mouvement des deux primaires. Ce problème est appelé le problème de trois corps restreint et circulaire.

On considère un repère tournant orthonormé, centré sur le centre de gravité O des deux primaires et dont les axes sont tels que l'axe des abscisses est dirigé vers le deuxième primaire, l'axe des ordonnées fait un angle de \pi/2 avec celui des abscisse dans le même sens que le mouvement de rotation des primaires, et l'axe des z complète un trièdre direct.

On considère aussi un repère fixe orthonormé qui coïncide avec le repère tournant à t=0.

Dans le repère tournant, les coordonnées des deux primaires sont (-d_1,0,0)^T et (d_2,0,0)^T, où d_1/d_2=m_2/m_1 et d_1+d_2=d qui est la distance (fixe) qui sépare les deux primaires. On note \omega la vitesse de rotation angulaire des deux primaires par rapport au repère fixe.

De manière générale on notera (x,y,z)^T les coordonnées dans le repère fixe et (u,v,w)^T les coordonnées dans le repère tournant. Le point au dessus d'une quantité indique la dérivée par rapport au temps de cette quantité.

Question 1)

Exprimer d_1 et d_2 en fonction de m_1, m_2 et d.

Question 2)

Déterminer les formules de passage entre (x,y,z)^T et (u,v,w)^T pour un même objet.

Question 3)

Les deux forces qui s'appliquent au troisième corps sont \overrightarrow{F}_1=\frac{G m_1}{P P_1^3} \overrightarrow{PP_1} et \overrightarrow{F}_2=\frac{G m_2}{P P_2^3} \overrightarrow{PP_2}. En appliquant le principe fondamental de la dynamique, c'est-à-dire que l'accélération est égale à la somme des forces dans le repère fixe, écire les équations différentielles vérifiées par les coordonnées (x,y,z)^T de P.

Question 4)

En différenciant les expressions de (x,y,z)^T en fonction de (u,v,w)^T, en déduire les équations du mouvement dans le repère tournant.

Question 5)

Montrer qu'il existe une fonction U(u,v,w) tel que le système d'équations précédent s'écrit :

\begin{array}{rcl}\ddot{u} -2\dot{v} \,\omega &=& \frac{\partial U}{\partial u} \\  \ddot{v}+2\,\dot{u}\,\omega &=& \frac{\partial U}{\partial v} \\ \ddot{w} &=& \frac{\partial U}{\partial w} \end{array},

Question 6)

En multipliant chaque ligne du système précédent par \dot{u}, \dot{v} et \dot{w} respectivement, puis en additionnant, montrer que le système admet une intégrale du mouvement (c'est -à-dire une quantité qui est constante au cours du temps).

Question 7)

En déduire C en fonction de (x,y,z)^T et de leur dérivées par rapport au temps.

Question 8)

On considère maintenant que les deux primaires P_1 et P_2 correspondent au Soleil et à Jupiter respectivement. On pose m_1=M_\odot, \quad m_2=m_J,\quad d=a_J, \quad \omega=\sqrt{\frac{G(M_\odot+m_J)}{a_J^3}}, \quad OP=r. Comme m_J \ll M_\odot, on en déduit que d_1\approx 0, \quad r_1\approx r, \quad M_\odot+m_J\approx M_\odot.

On peut alors considérer la trajectoire du troisième comme une orbite keplerienne autour du Soleil se trouvant à l'origine. Soit a, e, et i le demi-grand axe, l'excentricité et l'inclinaison de cette trajectoire. On a alors les relations suivantes:

Sachant que \dot{x}^2+\dot{y}^2+\dot{z}^2=2\frac{GM_\odot}{r}-\frac{GM_\odot}{a} , \dot{y}x-\dot{x}y=\sqrt{ GM_\odot a (1-e^2)}\cos i, réécrire l'équation précédente pour C en fonction de a, e, i. L'expression obtenue correspond au paramètre de Tisserand qui est une quasi constante du mouvement pour les comètes de la famille de Jupiter qui sont essentiellement soumisent à l'influence de Jupiter et du Soleil. On remarquera que les approximations faites fonctionnent pourvu que l'on ne soit pas trop proche de Jupiter.


Intégrales multiples

Auteurs: Stéphane Erard, Marc Fouchard

Photométrie des surfaces planétaires

Les surfaces planétaires réfléchissent la lumière solaire d'une façon qui dépend de leurs propriétés et de leur composition. Si les caractéristiques spectrales reflètent la composition (minéralogique) d'une surface, la distribution angulaire du rayonnement diffusé dépend surtout de ses propriétés physiques : taille de particules, porosité, rugosité à diverses échelles...

Divers modèles photométriques rendent compte de ces comportements, en décrivant la dépendance angulaire de la luminance. La luminance est la puissance émise ou diffusée dans un angle solide élémentaire par unité de surface. Il s’agit d’une caractéristique intrinsèque à la source lumineuse :

Figure 1
fig1.png
Géométrie d'observation d'une surface
Crédit : Astrophysique sur Mesure / Erard

Dans la configuration générale décrite Figure 1, la luminance est définie de la façon suivante :

L =  \frac{dW}{S\,cos\,e\,d\Omega}

où dW est la puissance recueillie par le détecteur, e est l'angle sous lequel on voit la source, d\Omega l'angle solide sous lequel la source voit le détecteur, et S la surface de la source. Dans le Système International, la luminance se mesure en W\,m^{-2}\,sr^{-1}, ou dans un intervalle de longueur d'onde élémentaire en W\,m^{-2}\,sr^{-1}\,\mu m.

Le modèle photométrique le plus commun est le modèle lambertien, que suit notamment le corps noir : la luminance est simplement isotrope (ne dépend pas de la direction e).


Ex: photométrie des surfaces planétaires

exerciceSoleil

Difficulté : ☆☆   Temps : 45 min

Question 1)

On assimile le Soleil à un corps noir. Quelle est la puissance rayonnée par un élément de surface dans une direction donnée ?

Question 2)

Calculer la luminance intégrale (intégrée spectralement), toujours dans une direction donnée. Application numérique.

Question 3)

Calculer la luminosité totale d'un élément de surface (rayonnée dans toutes les directions). Commenter.

Question 4)

Calculer la puissance totale émise par le Soleil. Application numérique (on donne pour le rayon du Soleil r_s = 695\,000\,km).

exerciceSurface lunaire

Difficulté : ☆☆   Temps : 20 min

Le cas des surfaces planétaires est différent, leur capacité à réfléchir le rayonnement solaire dépendant de leur état physique : rugosité, taille des particules en surface... En outre la position du Soleil intervient également puisqu'on observe maintenant en réflexion (voir Figure 2). Le modèle lambertien est encore adapté aux surfaces très claires, mais ne décrit pas correctement les propriétés de la Lune ou des astéroïdes qui sont relativement sombres. On utilise souvent le modèle de Lommel-Seeliger, qui donne la luminance comme : L(\mu_0, \mu) =  p F\frac{\mu_0}{\mu_0 + \mu}

où p est l'albedo de la surface (coefficient de réflexion sous incidence et émergence nulles), F est le flux solaire à la distance de la planète, \mu_0 et \mu sont les cosinus des angles d'incidence et d'émergence.

Figure 2
GeomSurface.png
Géométrie d'observation d'une surface planétaire
Crédit : Astrophysique sur Mesure / Erard
Question 1)

On utilise la réflectance hémisphérique pour étudier les propriétés thermiques des surfaces. Celle-ci est définie comme : r_{hd} = \int_{2\pi} \frac{L}{F}\,d\Omega_i

d\Omega_i est l'angle solide élémentaire dans la direction d'incidence.

Calculer cette quantité en fonction des variables \mu_0 et \mu.


Théorème de Liouville

Auteur : Marc Fouchard.

Le but de cet exercice est de montrer qu'un volume soumis à un flux, c'est-à-dire qu'en chaque point de l'espace on peut associer un vecteur vitesse donné par une équation différentielle d'ordre 1, hamiltonien reste constant.

La figure suivante illustre cette propriété dans le cas du problème de 2 corps plan. Comme on est à deux dimensions un volume correspond à une surface. Le disque est soumis à une force gravitationelle due à un corps massif se trouvant à l'origine. La surface verte est constante au cours du temps, même si la forme est fortement modifiée.

théorème de Liouville
figures-int-mult/liouville.gif
Crédit : Astrophysique sur Mesure / Fouchard

Un flux hamiltonien vérifie les équations d'hamilton. C'est-à-dire que le point de coordonnées (q_1,\dots,q_n,p_1, \dots,p_n) vérifie les équations différentielles suivantes :

\frac{{\rm d}q_i}{{\rm d}t}=\frac{\partial H}{\partial p_i}, \quad \frac{{\rm d}p_i}{{\rm d}t}=-\frac{\partial H}{\partial q_i}, avec i=1,\dots,n.

H est le hamiltonien du système, indépendant du temps.


Ex: théorème de Liouville

Auteur: Marc Fouchard

exercicethéorème de Liouville

Difficulté : ☆☆☆   Temps : 1h

Question 1)

Soit \Gamma un volume de surface \Sigma. La variation du volume au cours du temps s'écrit:

\frac{{\rm d} \Gamma}{{\rm d} t}=\int\int_{\Sigma} \vec{v} \cdot \vec{{\rm d}S},

\vec{v}=\frac{{\rm d} q}{{\rm d} t} est le vecteur vitesse et \vec{{\rm d}S} est un vecteur normal à la surface \Sigma et de norme égale à une élément de surface.

Exprimer {\rm d}\Gamma/{\rm d} t en vonction de la divergence de la vitesse {\rm div}\, \vec{v}.

Question 2)

Montrer que {\rm div}\,\vec{v}=0 et donc que {\rm d}\Gamma \,/\, {\rm d}t=0.


Séries


Introduction

On trouvera dans ce chapitre les exercices suivants :


Séries de Fourier

Auteur: Jérôme Thiébaut

Spectre de puissance

Auteur: Jérôme Thiébaut

L'un des outils les plus utilisés pour étudier le champ de densité de l'univers est le spectre de puissance P(k). Ce spectre est relié à la transformée de Fourier de la fonction d'autocorrélation du champ et est une fonction des fréquences spatiales k. En d'autres termes, la transformée de Fourier revient à effectuer un changement de base et à remplacer le champ rho(x)définie de manière locale par une somme de sinusoides, cos(kx)=cos(2*pi*n*x/L) contenues dans une boîte de taille L, définissant le champ de manière globale.

Ainsi, le spectre de puissance donne pour chaque k, le poids de cette fréquence.

Dans la jeunesse de l'univers, celui ci était invariant d'échelle, ce qui signifie qu'il n'existait aucune longueur caractéristiques ou privilégiée. Ainsi le spectre était une loi de puissance de la forme P(k) prop k^m. On voit que la connaissance d'une seule quantité, l'indice m, procure beaucoup d'information. Le spectre de puissance est donc un outil très puissant et est extrêmement utilisé en cosmologie, notamment dans les simulations numériques.

Le but de cet exercice est de relier de manière précise le spectre de puissance à la transformée de Fourier de la fonction d'autocorrélation du champ de densité.


Ex: Spectre de puissance

Auteur: Jérôme Thiébaut

exercicespectre de puissance

Difficulté : ☆☆☆   Temps : 25 min

introductionIntroduction

On définit le contraste de densité delta par delta=(rho-accent(rho;barre))/accent(rho;barre), où rho est la densité et accent(rho;barre) la densité moyenne.

On travaille dans une boîte d'univers isotrope et plat de côté L (les mesures étant faites à partir d'observations, seul l'univers proche est accessible), on peut donc exprimer le champ de contraste de densité delta(accent(x;->)) comme une série de Fourier:

delta(accent(x;->))=somme(delta_accent(k;->)*exp(-i*pscalaire(accent(k;->);accent(x;->)));accent(k;->)=0;infty), où le vecteur accent(k;->) = matrice(ligne(k_x ; k_y; k_z)) est tel que k_x=n*2*pi/L avec n= 1;2;... .De même pour k_y et k_z.

On définit la fonction d'autocorrélation du champ de densité par :

xi(accent(r;->))=<delta(accent(x;->))*delta(accent(x;->)+accent(r;->))>, où la valeur moyenne s'effectue sur le volume de la boîte.

Question 1)

Exprimer la fonction d'autocorrélation en fonction de accent(k;->).

Question 2)

Ramener la double somme obtenue à une seule.

Question 3)

Le passage de la somme en intégrale donne xi=(1/(2*pi)^3)*intégrale(abs(delta_accent(k;->))^2*exp(-i*pscalaire(accent(k;->);accent(r;->)));accent(k;->)), où le facteur 1/(2*pi)^3 a été introduit par commodité. On définit le spectre de puissance comme : P(k)=<abs(delta_k)^2>.

Calculer la partie angulaire de l'intégrale (en coordonnées sphériques, intégrer selon theta et phi) et exprimer le résultat en fonction du spectre de puissance. Si on inverse cette relation, on trouve que P(k)=intégrale(xi(r)*(sin(kr)/(k*r))*4*pi*r^2;r;0;infini).

Les relevés de galaxies tel le SDSS, permettent, grâce à des modèles de profils de masse autour des galaxies, de remonter à une estimation du champ de densité. Après avoir calculé son autocorrélation, on peut donc calculer son spectre de puissance et le comparer à la théorie.


Variables complexes


Introduction

On trouvera dans cette partie les exercices suivants :


Théorème des résidus

Auteurs: Alain Vienne, Stéphane Erard

Excentricité limite dans les développements du problème à 2 corps

Auteur: Alain Vienne

En mécanique céleste, il est quelque fois utile d'utiliser certaines formules du problème à 2 corps (ou problème keplerien) sous forme de développements. Cela permet, en théorie des perturbations, de faire des calculs analytiques.

Par exemple, l'"équation du centre", qui donne la position du corps sur son orbite en fonction du temps, est:

W=M+(2e-\frac{1}{4}e^3)\sin M + (\frac{5}{4}e^2-\frac{11}{24}e^4)\sin 2M + \frac{13}{12}e^3 \sin3M + \frac{103}{96} e^4 \sin 4M +O (e^5)

W est l'anomalie vraie, c'est à dire l'angle qui positionne le corps sur son orbite à partir de la direction du minimum de distance (péricentre ). M est le temps ou plus précisément c'est l'anomalie moyenne M=\frac{2\pi}{T} (t-t_0) avec T la période, t le temps et t_0 l'instant de passage au péricentre. e est l'excentricité.

Attention cette formule est bien une série entière en e (mais tronquée à l'ordre 4). Cela aurait été plus net si on l'avait écrit comme:

W=M+2e \sin M + e^2 \frac{5}{4}\sin 2M + e^3 (-\frac{1}{4}\sin M + \frac{13}{12} \sin3M) + e^4 (-\frac{11}{24}\sin 2M  + \frac{103}{96} \sin 4M) +\dots

Mais, en fait, on préfère l'écriture en série de Fourier, c'est-à-dire:

W=M+ f_1(e) \sin M + f_2(e)\sin 2M + f_3(e) \sin3M + f_4(e) \sin 4M + \dots

En tant que série de Fourier, la convergence ne pose pas de problème car la fonction à considérer est de classe C^1 par rapport à la variable M. Seulement, dès que les f_i sont tronqués à un certain ordre en excentricité, cela revient à considérer la série entière.

L'exercice qui est proposé utilise le théorème de Lagrange pour montrer que la série entière ci-dessus (et toutes celles du problème des 2-corps) converge si e<0,6627434\dots . Cela signifie que ces séries ne peuvent être utilisées que pour des excentricités bien en deça de cette valeur. Evidemment, la solution du problème à 2 corps elle-même existe quelque soit l'excentricité.

Théorème de Lagrange

Soit une fonction complexe \phi (z) de la variable complexe z. Soient a et \varepsilon des complexes.

Si \phi (z) est analytique à l'intérieur du contour ( \mathcal{C} ) du plan complexe entourant le point a avec ( \mathcal{C} ) tel que : | \varepsilon \phi (z) | \le |z-a|

Alors l'équation : z=a+\varepsilon \phi (z) a une raçine développable dans l'intérieur de ( \mathcal{C} ) en série entière de \varepsilon :

z= a + \sum_{n=1}^{\infty} \frac{\varepsilon ^n}{n!}\bigg[\frac{d^{n-1}}{dz^{n-1}}[\phi (z)]^n\bigg]_{z=a}

Plus généralement, pour toute fonction f analytique dans ( \mathcal{C} ), f(z) peut aussi être développée:

f(z)= f(a) + \sum_{n=1}^{\infty} \frac{\varepsilon ^n}{n!}\bigg[\frac{d^{n-1}}{dz^{n-1}}\bigg( \frac{df}{dz}[\phi (z)]^n \bigg) \bigg]_{z=a}

Un autre exercice avec ce théorème est disponible ici.


Ex : Excentricité limite dans les développements du problème à 2 corps

Auteur: Alain Vienne

exerciceExcentricité limite dans les développements du problème à 2 corps

Difficulté : ☆☆☆   Temps : 1h30

introductionIntroduction

Dans le problème à 2 corps (voir, pour plus de détails, un cours d'astronomie, par exemple celui-ci) l'anomalie vraie et l'anomalie moyenne sont liées grâce à l'anomalie excentrique E par les 2 formules suivantes:

\tan \frac{W}{2} = \sqrt{\frac{1+e}{1-e}} \tan \frac{E}{2}  , et

M=E-e\sin E   qui est appelée "équation de Képler".

Question 1)

Sachant que e est le petit paramètre, montrer que l'équation de Kepler est de la forme indiquée dans le théorème de Lagrange. Indiquer à quoi correspond chacun des paramètres de ce théorème dans notre problème.

Question 2)

E est donc complexe. On suppose M réel et on pose: E-M=\rho \exp \imath \theta (module et argument). Exprimer \sin E puis \sin ^2 E en fonction de M, \rho et \theta

Question 3)

Le contour ( \mathcal{C} ) est défini par e \le \frac{\rho}{|\sin E|}. Le cas le plus défavorable correspond à |\sin E | maximum. Donner les conditions sur \theta et M correspondantes.

Question 4)

Que devient |\sin E | pour ces conditions?

Question 5)

Par la condition e \le \frac{\rho}{|\sin E|}, on cherche donc à maximiser \frac{\rho}{cosh \rho}. Montrer que ce maximum est atteint pour \rho = 1,1996784\dots En déduire la plus grande valeur de l'exentricité e_M.

remarqueRemarque

Ainsi pour e \le e_M, on peut écrire:

E=M+\sum_{n=1}^{\infty} \frac {e^n}{n!}\frac{d^{n-1}}{dM^{n-1}}(\sin ^n M ). Pour obtenir l'équation du centre, il faut encore utiliser la formule \tan \frac{W}{2} = \sqrt{\frac{1+e}{1-e}} \tan \frac{E}{2} pour revenir à W. Mais cette formule ne pose aucun problème de convergence. La valeur de e_M est donc inchangée.


Indices de réfraction / relations de Kramers-Kronig

Auteur: Stéphane Erard

Les relations de Kramers-Kronig relient les indices de réfraction réel et imaginaire d'un même milieu matériel. Ceux-ci, bien qu'on les appelle couramment constantes optiques, varient en fonction de la longueur d'onde d'une façon caractéristique de la composition du milieu. A ce titre, ils jouent un rôle particulièrement important en Astrophysique puisque l'étude de la lumière produite, absorbée ou réfléchie par un astre distant permet de connaître sa composition.

La partie réelle de l'indice (généralement appelée indice de réfraction) intervient dans les lois de Snell-Descartes, la partie imaginaire (coefficient d'absorption) rendant compte de l'absorption au cours de la propagation dans le milieu. La mesure simultanée des deux quantités est difficile ; les relations de Kramers-Kronig qui permettent de calculer l'un en connaissant l'autre ont donc une grande importance pratique en spectroscopie de laboratoire.


Ex: relations de Kramers-Kronig

Auteur: Stéphane Erard

exerciceRelations de Kramers-Kronig

Difficulté : ☆☆   Temps : 60 min

Les équations de Maxwell dans un milieu matériel font intervenir un vecteur induction électrique défini comme :

\vec{D} = \epsilon_0 \vec{E} + \vec{P}

\vec{E} est le champ électrique appliqué, \vec{P} la polarisation électrique du milieu (qui décrit la réaction du milieu à l'application du champ électrique externe) et \epsilon_0 une constante physique appelée permittivité du vide.

Les propriétés du milieu lui-même sont décrites par un certain type de relation entre le champ électrique et la polarisation. Dans un grand nombre de cas (milieu isotrope, champ faible...) cette relation peut s'écrire :

\vec{P} = \epsilon_0 \chi_{e} \vec{E}

\chi_{e} (la susceptibilité électrique) est a priori un tenseur d'ordre 2 dépendant du temps et de la position.

La solution des équations de Maxwell dans le milieu met en évidence l'indice de réfraction complexe de ce milieu :

\eta(\omega) = \sqrt{1 + \chi(\omega)}

\chi(\omega) est la représentation en fréquence de \chi_e(t), c'est-à-dire sa transformée de Fourier.

Question 1)

On considère un milieu linéaire, tel que :

\vec{P}(t) = \int_{-\infty}^{\infty} G(t, t') \vec{E}(t') dt'

Que représente la fonction G ?

Question 2)

Déduire en utilisant le théorème de convolution une relation entre les représentations en fréquence du champ électrique et de la polarisation, puis entre G et \chi(\omega).

Question 3)

En supposant constantes les propriétés du milieu, comment peut-on simplifier la fonction G ?

Question 4)

On considère la fonction  \frac{\chi(\tilde{\omega}) }{(\tilde{\omega} - \omega) de la variable complexe \tilde{\omega}, où \omega est réel. Montrer qu'elle est analytique dans la partie supérieure du plan complexe.

Question 5)

Trouver un contour d'intégration adéquat pour calculer l'intégrale de f(\omega). Calculer l'intégrale. Commentaire ?

Question 6)

En déduire une relation entre parties réelle et imaginaire de \chi(\omega) .

Question 7)

En explicitant les symétries de \chi(\omega) , trouver une autre écriture de ces relations.

Question 8)

On écrit l'indice de réfraction en fonction de l'indice réel n et du coefficient d'absorption \alpha, avec c = vitesse de la lumière :

\eta(\omega) = \sqrt{1 + \chi(\omega)} = n(\omega) + i \alpha(\omega) \frac{c}{2 \omega}

Ecrire n en fonction de \alpha (ce sont les deux quantités directement mesurables). Quel est l'intérêt pratique de cette relation ?


Géométrie


Introduction

On trouvera dans cette partie les exercices suivants :


Théorème de Thalès

Auteur: Arnaud Beck

Distance Terre Lune

Auteur: Arnaud Beck

La première estimation de la distance Terre-Lune date de la Grèce antique. Pourtant elle est d'une précision remarquable. Elle a été effectuée par Aristarque de Samos vers 250 avant JC. Celui-ci a eu l'idée d'observer une éclipse de Lune pour comparer le rayon de la Lune avec l'ombre de la Terre projetée sur la Lune. Cette méthode est facile à mettre en oeuvre et d'une grande précision mais a l'inconvénient de donner uniquement le rapport des rayons lunaire et terrestre. Pour connaître la valeur du rayon de la Lune il faut donc connaître celui de la Terre.

Dans cet exercice, on se propose de refaire les calculs d'Aristarque de Samos en se basant sur les observations qu'il avait lui-même effectuées en son temps et de retrouver le rapport entre les rayons lunaire et terrestre.


Ex: Distance Terre Lune

Auteur: Arnaud Beck

exerciceCalcul de la distance Terre-Lune

Difficulté :   

introductionIntroduction

Eclipse de soleil

Il est connu que pendant une éclipse de Soleil, la Lune vient se placer entre la Terre et le Soleil et cache presque exactement le Soleil aux observateurs terrestres. Cela est possible car depuis la Terre, la Lune et le Soleil ont le même rayon apparent. Soit \beta le demi-angle sous lequel ces deux astres sont vus depuis la Terre (voir partie droite de la figure ci-dessous). Cet angle est connu directement par l'observation et vaut à peu près 0,25°.

Eclipse de Lune

Une éclipse de Lune se produit lorsque la Lune passe dans le cône d'ombre de la Terre éclairée par le Soleil (voir partie gauche de la figure ci-dessous). Soit \alpha l'angle d'ouverture de ce cône. Sa valeur est a priori inconnue. Aristarque de Samos avait observé que la largeur de ce cône au niveau de la Lune était de 3 diamètres lunaires.

Pour une question de lisibilité de la figure, la Lune n'a pas la même échelle sur la partie droite que sur la partie gauche. Les deux phénomène étant indépendants, cela n'a pas d'incidence sur le raisonnement.

Eclipses de soleil et de lune
Thalesok.png
La partie droite de la figure représente la configuration d'une éclipse de soleil et la partie gauche la configuration d'une éclipse de Lune. T, L_1, L_2 et S sont respectivement les positions de la Terre, de la Lune pendant une éclipse de soleil, de la Lune pendant une éclipse de Lune, et du Soleil. R_t R_s et R_l sont les rayons terrestre, solaire et lunaire.
Crédit : Arnaud Beck
Question 1)

Exprimer \tan \beta et \tan \alpha en fonction de R_t R_s et TS.

Question 2)

Que dire de \alpha et \beta si on suppose le Soleil très grand devant la Terre ?

Question 3)

Avec l'hypothèse précédente, calculer la valeur de h (défini sur la figure).

Question 4)

En déduire R_l et TL en fonction de R_t.


Théorème de Pythagore

Auteur: Arnaud Beck, Stéphane Erard, Alain Vienne

Visibilité d'un satellite

Auteur: Alain Vienne

On propose ici une application simple et directe du thèorème de Pythagore. Il faut le considérer ici comme une révision des "années collège et lycée" de l'étudiant. Notre expérience d'enseignement montre que cela n'est pas inutile.

On considère un satellite à une certaine altitude. Il s'agit de savoir sur quelle partie de la Terre il sera visible. Cet excercice peut s'appliquer directement pour savoir d'où est visible une montagne.

L'exercice proposé dans la partie "intégrale de Rieman" est plus complet et calcule notamment la surface correspondante.


Ex: Visibilité d'un satellite

Auteur: Alain Vienne

exerciceZone visible d'une montagne ou d'un satellite

Difficulté :    Temps : 20 mn

Question 1)

Soit un satellite artificiel de hauteur h, sur quelle partie de la Terre (supposée sphérique) est visible le satellite?

Question 2)

Le rayon de la Terre étant de R = 6380 km, à quelle distance maximale du point de la Terre survolé par le satellite peut-on voir le satellite d'altitude h=400 km?


Extinction atmosphérique

Auteurs: Arnaud Beck, Stéphane Erard

Quand le Soleil est au zénith, impossible de le regarder à l'oeil nu sans être ébloui voire même se brûler la rétine. Pourtant, le soir tombé, on peut admirer le Soleil couchant sans la moindre gêne.

Cela s'explique simplement par la diffusion des rayons solaires par les molécules de l'atmosphère. En effet, quand les rayons du Soleil rencontrent une molécule, une partie d'entre eux est déviée ou absorbée. Et plus le nombre de particules qu'ils rencontrent est grand, plus la proportion de rayons déviés est grande et l'énergie lumineuse reçue par l'observateur sera réduite d'autant.

Dans cet exercice, on propose de quantifier le nombre de particules rencontrées par un rayon de Soleil en fonction de sa position dans le ciel par rapport à un observateur potentiel. Le parcours atmosphérique est également calculé dans le cas général.


Ex: Extinction atmosphérique

Auteur: Arnaud Beck, Stéphane Erard

exercicePourquoi peut-on regarder le Soleil couchant ?

Difficulté :    Temps : 1h

Le nombre de particules atmosphériques rencontrées par un rayon de Soleil le long de son parcours est appelé densité de colonne, et est égal à :

N= \int_s n(s)ds

s est la coordonnée le long du trajet du rayon et n(s) est la densité atmosphérique au point de coordonnée s.

On peut approximer la densité atmosphérique à faible altitude (là où elle est la plus dense) par:

n(z)=n_0\exp{\left(-\frac{z}{z_0}\right)}

z est l'altitude (mesurée verticalement), n_0 est la densité au niveau du sol, et z_0 est l'échelle de hauteur caractéristique de l'atmosphère. Cette expression est une forme de la loi barométrique.

La figure ci-dessous représente la situation. Le centre de la Terre est au point C, l'observateur en O. Le point S représente le point de coordonnée s sur le trajet du rayon de Soleil, et d'altitude z. \theta est la hauteur du Soleil sur l'horizon (vu par l'observateur) et R est le rayon de la Terre.

Arrivée d'un rayon de Soleil sur Terre
colonne_densite.gif
Crédit : Astrophysique sur Mesure / Beck
Question 1)

Dans le cas du Soleil couchant (\theta=0), donner l'expression de l'altitude z en fonction de la coordonnée s.

Question 2)

Donner l'expression de N_0, la densité de colonne au Soleil couchant (\theta=0). On remarque que la densité de particules décroît rapidement avec l'altitude et devient petite pour z > z_0 ; on peut donc tronquer l'intégrale à une altitude maximum telle que s \ll R (l'atmosphère est fine par rapport à la taille de la planète).

Question 3)

Reprendre les questions 1) et 2) pour donner l'expression de N_{\theta}, la densité de colonne pour une position \theta quelconque du Soleil dans le ciel. En plus de l'hypothèse précédente, on évite cette fois les situations proches de l'horizon ; on a donc \frac{s}{R}\ll \sin \theta.

Question 4)

Le Soleil est au zénith quand \theta=\pi/2. Calculer le rapport \frac{N_{Zenith}}{N_0} . Pour l'application numérique on prendra R=6400 km, z_0=8 km (échelle de hauteur de l'atmosphère terrestre).

Auteur: Stéphane Erard

exerciceCalcul de la masse d'air

Difficulté :    Temps : 30 min

Le rapport \frac{s}{z} de l'exercice précédent est appelé masse d'air en Astronomie. C'est le chemin optique parcouru dans l'atmosphère par rapport à la position zénitale. Suffisamment loin de l'horizon, on a en bonne approximation \frac{s}{z} = 1/\sin\theta = 1/\cos\; i, où i est l'angle zénital (compté à partir de la verticale). Cette valeur correspond à l'approximation plan-paralléle. On cherche toujours à observer les astres sous faible masse d'air (< 2) pour limiter l'extinction atmosphérique.

On veut maintenant calculer exactement la longueur du chemin optique parcouru par les rayons lumineux dans l'atmosphère pour étudier la validité de l'approximation précédente.

Question 1)

Reprendre la question 3 de l'exercice précédent : dériver une relation entre l'altitude z et la coordonnée s pour une hauteur \theta quelconque.

On exprimera cette relation en fonction de l'angle zénital i (compté à partir de la verticale locale).

Question 2)

Résoudre en s.

Question 3)

Tracer s en fonction de l'angle zénital i et comparer avec l'approximation usuelle en sécante (1/\cos i).

Question 4)

Quel est le domaine de validité de l'approximation en sécante ? Quels autres phénomènes affectent la diffusion dans ces conditions ? Conclusion ?


Mesure de la vitesse de la lumière

Auteur: Stéphane Erard

Depuis l'antiquité jusqu'au XVIIe siècle, plusieurs conceptions de la lumière se sont succédées. Il était notamment impossible de dire si la lumière se propage instantanément ou à vitesse finie. En 1676, Ole Römer met en évidence une vitesse de propagation finie, dont il estime un ordre de grandeur correct à partir de l'observation des satellites de Jupiter. Cette méthode est reproduite ici.


ex: Mesure de la vitesse de la lumière

Auteur: Stéphane Erard

exerciceMouvement des satellites de Jupiter

Difficulté :    Temps : 60 min

En 1668, Gian Domenico Cassini a publié les premières éphémérides des satellites galiléens. L'intérêt de ces phénomènes était de fournir une horloge visible et consultable partout sur Terre : les débuts d'éclipse des satellites. Ceux-ci permettent de déterminer la longitude du lieu d'observation par comparaison avec une horloge locale.

Dans les années suivantes, Römer mit néanmoins en évidence des écarts importants avec ses propres observations de Io, le plus proche satellite de Jupiter, et le plus rapide. Ces écarts augmentaient (jusqu'à 11 minutes) puis diminuaient avec une périodicité d'un an.

Question 1)

On considère la situation de la Figure 1, lorsque Io est en émersion au point D (il sort de l'ombre de Jupiter). Durant un premier événement la Terre est au point L de son orbite, lors du suivant elle est en K.

Si la lumière se propage instantanément, quel intervalle sépare les deux événements ?

Question 2)

Même question en supposant que la lumière se déplace à la vitesse c. Remarques sur la Figure 1 ? Préciser les approximations implicites qu'on a fait.

Question 3)

Calculer en unités astronomiques la distance Terre-Jupiter à l'opposition (lorsque les deux planètes sont au plus près).

Question 4)

On effectue une première observation d'éclipse à l'opposition. A quel moment peut-on effectuer une seconde observation pour laquelle le décalage sera maximum ?

Question 5)

On observe 261 jours après l'opposition. De quels angles se sont déplacés Jupiter et la Terre sur leurs orbites depuis l'opposition ? Quel est l'angle Jupiter-Soleil-Terre à ce moment ?

Question 6)

Calculer la distance Terre-Jupiter \Delta en unités astronomiques au moment de la deuxième observation.

Question 7)

Le second événement est observé avec 13,5 min de retard par rapport à un phénomène régulier. En déduire une estimation de la vitesse de la lumière.


Applications

Auteurs: Alain Vienne, S. Renner

La loi des aires

Auteur: Alain Vienne

La loi des aires dit que, dans le problème de l'interaction gravitationnelle de deux corps, l'aire balayée par le rayon vecteur est proportionnel au temps. Cette loi est aussi appelée "deuxième loi de Kepler" (voir aussi dans ce même chapitre, le lien suivant).

figures/kep2trans.gif
La loi des aires : les aires décrites par le mobile dans des temps égaux sont égales. Ainsi, lorsque l'astre s'éloigne du Soleil, sa vitesse diminue.

En fait, la loi des aires est plus générale que la deuxième loi de Kepler puisque qu'elle s'applique pour toute force centrale. Pour la démontrer, il faut bien-sur utiliser la loi fondamentale de la dynamique:

Principe fondamental de la dynamique

L'accélération d'un mobile est proportionnelle à la force à laquelle il est soumis.

La preuve qui est proposée en exercice utilise un modèle discret. Elle est directement inspirée d'une application isssue du livre de Daniel Perrin "Nombre, mesures et géométrie" (Ed. CASSINI). Ainsi le temps est une juxtaposition d'instants t_n de durée très courte h de telle sorte que t_{n+1}-t_n=h. La discrétisation revient à supposer qu'entre les instants t_{n-1} et t_n, le mobile se déplace de M_{n-1} à M_n avec la vistesse constante v_n. En vecteur la vistesse est donc \overrightarrow{v_n}=\overrightarrow{M_{n-1}M_n}/h. Sur l'intervalle suivant [t_n,t_{n+1}], la vitesse est différente mais constante aussi pour cette durée: \overrightarrow{v_{n+1}}=\overrightarrow{M_nM_{n+1}}/h. Ainsi à l'instant t_n l'accélération est \overrightarrow{\gamma_n}=\frac{\overrightarrow{v_{n+1}}-\overrightarrow{v_n}}{h} .

aires_discret.png
Crédit : Astrophysique sur Mesure / Vienne

Le modèle continu s'obtient facilement par passage à la limite.

La loi fondamentale de la dynamique s'écrit alors: \overrightarrow{\gamma_n} \propto \overrightarrow{F_n}

Les outils mathématiques nécéssaires à cette preuve se limitent alors à deux petits lemmes que Daniel Perrin nomment lemmes de découpage et que nous admettrons:

Lemme du demi-parallélogramme :

Soit (ABCD) un parallélogramme. La diagonale [AC] partage le parallélogramme en deux triangles de même aire: \mathcal A (ABC) = \mathcal A (ACD) = \frac{1}{2} \mathcal A (ABCD). Plus généralement, pout tout point R de [CD], on a : \mathcal A (ARB) = \frac{1}{2} \mathcal A (ABCD).

Lemme de la médiane :

Soit (ABC)un triangle et A' le milieu de [BC]. La médiane [AA'] partage le triangle en deux triangles de même aire: \mathcal A (ABA') = \mathcal A (AA'C).


Ex: La loi des aires

Auteur: Alain Vienne

exerciceLa loi des aires

Difficulté : ☆☆   Temps : 1h

introductionIntroduction

Le mobile M_n est soumis à une force centrale, c'est-à-dire dirigée vers un point Ofixe (le Soleil par exemple si la masse de M_n est négligeable par rapport à celle du Soleil): la force est \overrightarrow{F_n}=k_n \overrightarrow{OM_n}.

remarqueRemarque

Il n'y a aucune hypothèse nécessaire sur le réel k_n même si on sait que pour la loi de Newton ce scalaire est négatif et inversement proportionnel au carré de la distance OM_n

aires_perrin.png
Crédit : Astrophysique sur Mesure / Vienne
Question 1)

Montrer qu'à tout instant (c'est-à-dire pour tout entier n), on a: \mathcal A (0M_{n-1}M_n) = \mathcal A (OM_nM_{n+1})

Cela signifie bien que l'aire balayé par le rayon vecteur \overrightarrow{OM_n} est proportionnel au temps parcouru.


Vitesse orbitale de la Terre

Auteur: S. Renner

Date de création: 2 mars 2009

introductionIntroduction

L'effet Doppler-Fizeau représente le décalage en fréquence d'une onde lumineuse entre les mesures à l'émission et à la réception, lorsque la distance entre un émetteur et un récepteur varie au cours du temps.

Par exemple, lors du passage d'un camion de pompier muni d'une sirène, c'est l'effet Doppler qui se manifeste dans la perception de la hauteur du son (plus aigu lorsque le véhicule se rapproche, plus grave lorsqu'il s'éloigne).

Ce phénomène est particulièrement important en astronomie car il permet de mesurer les vitesses (d'approche ou d'éloignement) des objets célestes.

On observe Arcturus, troisième étoile la plus brillante du ciel (dans la constellation du Bouvier), à deux dates t_1 et t_2 espacées de 6 mois.

La latitude par rapport au plan de l'orbite de la Terre est b_a=30.75^_\circ, et la longitude par rapport à une direction fixe \gamma est l_a=204.25^\circ. A l'instant t_1 la longitude de la Terre est l_1=114.25^\circ , et l_2=294.25^\circ à l'instant t_2. Voir la figure ci-dessous pour les conditions d'observation.

Doppler_Arcturus_small.jpg
Situation de l'observation d'Arcturus. Voir texte pour la valeur des angles.
Crédit : Astrophysique sur Mesure / Renner

On effectue aux dates t_1 et t_2 un spectre de la lumière de l'étoile. L'étude des raies d'absorption permet de remarquer qu'une raie d'absorption du fer, qui normalement se situe à \lambda_0=446.165 nm, est mesurée \lambda_1=446.123 nm sur le spectre obtenu à la date t_1, et \lambda_2=446.199 nm sur celui obtenu à la date t_2.

L'objectif est d'en déduire la vitesse orbitale de la Terre autour du Soleil, ainsi que la distance moyenne Terre-Soleil.


Ex: Vitesse orbitale de la Terre

Auteur: S. Renner

exerciceVitesse orbitale de la Terre

Difficulté :    Temps : 1h30

On fait l'hypothèse que l'orbite de la Terre est circulaire est que celle-ci est décrite avec une vitesse uniforme V.

Question 1)

On note V_A la vitesse radiale d'Arcturus par rapport au Soleil (supposée identique aux instants t_1et t_2). Ecrire en fonction de V, V_A et b_a la vitesse radiale d'Arcturus par rapport à l'observateur à l'instant t_1 (on notera cette vitesse v_{r_1}), ainsi qu'à l'instant t_2 (notée v_{r_2}).

Question 2)

En appliquant la formule de l'effet Doppler-Fizeau aux instants t_1 et t_2 pour la longueur d'onde de référence \lambda_0, écrire les expressions de v_{r_1} et v_{r_2}.

Question 3)

En déduire l'expression de V et V_A en fonction des longueurs d'onde \lambda_0, \lambda_1 et \lambda_2. Calculer leur valeur numériquement en km.s^{-1}.

Question 4)

Calculer la distance Terre-Soleil en km sachant que la période de révolution est P=365.2563 jours.


Equation de Kepler

Auteur: S. Renner

Date de création: 16 mai 2013

On reprend les résultats obtenus dans l'exercice sur la résolution du problème des 2 corps. Le but ici est d'établir l'équation de Kepler à l'aide de la géométrie essentiellement, plutôt que par le calcul. L'équation de Kepler (M=E-e \sin E) est importante car elle fait le lien entre la position de l'objet sur son orbite (voir la figure ci-dessous) et le temps, ou plus précisément l'anomalie moyenne M= \frac{2 \pi}{T} (t - \tau), avec T la période orbitale, t le temps et \tau l'instant de passage au péricentre.

Les trois anomalies
kepler.gif
Crédit : Astrophysique sur Mesure / Bessou Fouchard

Ex: Equation de Kepler

Auteur: S. Renner

exerciceEquation de Kepler

Difficulté :    Temps : 1h

Kepler_equ.jpg
Trajectoire elliptique d'un corps M (de foyer F, péricentre P, demi-grand axe a, excentricité e), cercle principal, anomalie excentrique E et vraie f
Crédit : Astrophysique sur Mesure / Renner
Question 1)

Exprimer l'aire délimitée par les points F, P, M' en fonction de a, e, E.

Question 2)

Calculer l'aire délimitée par les points F, P, M.

Question 3)

En déduire l'équation de Kepler M=E-e \sin E.


Produits scalaire, vectoriel et mixte

Auteur: Alain Vienne

Dans le problème des 2-corps

Auteur: Alain Vienne

Quand on formule le problème des 2-corps, on arrive au problème de Képler, c'est-à-dire à l'équation différentielle vectorielle suivante:

\frac{d^2 \overrightarrow{r}}{dt^2} = - \mu \frac{\overrightarrow{r}}{r^3}

\mu est une constante réelle positive et \ovserrightarrow{r} = \overrightarrow{OM} \in \mathbb{R}^3. O est un point fixe et on étudie le mouvement de M.

Les deux exercices proposés donnent la loi des aires et l'intégrale de Laplace.

En fait, le premier exercice aura 2 conséquences: la première est que le mouvement est plan et la deuxième que la loi du mouvement est la loi des aires proprement dite:

figures/kep2trans.gif
La loi des aires : les aires décrites par le mobile dans des temps égaux sont égales. Ainsi, lorsque l'astre s'éloigne du Soleil, sa vitesse diminue.

Ex : Dans le problème des 2-corps

Auteur: Alain Vienne

exerciceLa loi des aires

Difficulté :    Temps : 20mn

introductionIntroduction

La loi des aires est très facile à obtenir avec le produit vectoriel. Sans le produit vectoriel, on peut aller voir cet exercice.

Question 1)

Montrer que dans le problème képlérien, le moment cinétique:

\overrightarrow{G} = \overrightarrow{r} \wedge \frac{\overrightarrow{r}}{dt}

est invariant.

Question 2)

Montrer que le mouvement de M se fait dans un plan passant par le point O et orthogonal à \overrightarrow{G}.

Question 3)

En utilisant un élément d'aire dS parcouru par \overrightarrow{r} pendant l'élément de temps dt, montrer la loi des aires proprement dite: L'aire balayée par unité de temps est constante.

Auteur: Alain Vienne

exerciceIntégrale de Laplace

Difficulté :    Temps : 20mn

Question 1)

Montrer l'expression suivante:

\frac{d}{dt} (\frac{\overrightarrow{r}}{dt}\wedge \overrightarrow{G}) = \mu \frac{d}{dt} ( \frac{\overrightarrow{r}}{r})

Question 2)

Déduire de l'égalité précédente une expression qui est constante pendant le mouvement (intégrale de Laplace).


Les coniques

Auteurs: Marc Fouchard, Alain Vienne

Trajectoires balistiques dans le système solaire

Auteur : Alain Vienne

On considère une sonde spatiale qui se déplace dans le système solaire. On suppose qu'elle ne subit que l'attraction gravitationnelle du Soleil S. Sous cette hypothèse, le mouvement de cette sonde autour du Soleil est un mouvement képlérien c'est-à-dire que la trajectoire est une conique dont le Soleil occupe l'un des foyers.

Les coniques sont des ellipses (comme le dit la première loi de Képler) ou des hyperboles ou des paraboles.

definitionDéfinition

Une conique est l'ensemble des points dont la somme ou la différence, des distances à 2 points fixes est constante. Ces 2 points sont appelés foyers et la distance constante est appelée grand axe

On ne considère pas ici le cas des paraboles qui est le cas limite entre les ellipses et les hyperboles. Une parabole peut être vue comme une ellipse dont l'un des foyer est rejeté à l'infini, ou symétriquement, comme une hyperbole dont l'un des foyers est rejeté à l'infini.

L'exercice proposé considère 2 points T et J du système solaire avec T plus près de S que J. On peut considérer que T est la Terre et que J est Jupiter. Cela permet de fixer les idées mais il n'y a aucune obligation formelle à cela. On fait partir la sonde du point T pour qu'elle arrive au point J. S étant l'un des foyers, on note S' le second foyer de la conique (\mathcal{C}) qui définit la trajectoire.

application.png


Ex: Trajectoire balistique dans le système solaire

Auteur: Alain Vienne

exerciceTrajectoire balistique dans le système solaire

Difficulté : ☆☆   Temps : 1h30

Question 1)

Une conique est défine par \mathcal{C}_{F,F',a}=\{ M \textrm{ tel que } |MF \pm MF'|=2a \}. Préciser le cas d'une ellipse et le cas d'une hyperbole. Pour ce dernier cas, préciser aussi comment sont distinguées les deux branches de l'hyperbole.

Question 2)

Montrer que le second foyer S' se trouve sur une hyperbole (\mathcal{H}) de foyers T et J passant par S

L'hyperbole des deuxièmes foyers
figures/balistique.png
Crédit : Astrophysique sur Mesure / Vienne

Question 3)

Donner la nature de la conique (\mathcal{C}) suivant la branche de (\mathcal{H}) sur laquelle se trouve S'.

On pose S_1 le point de (\mathcal{H}) symétrique de S par rapport à l'axe focal.

Question 4)

Montrer qu'aucune trajectoire physique n'est possible quand S' se trouve entre S et S_1.

Question 5)

On fixe S' sur (\mathcal{H}), exprimer le demi-grand axe a et l'excentricité e de la conique (\mathcal{C}) en fonction des distances entre les points T, S et S'

Question 6)

Indiquer ce que devient la conique (\mathcal{C}) quand

  • S' tend vers S
  • S' tend vers S_1
  • S' tend vers l'infini sur la même branche


Les solutions du problème de deux corps

Auteur: Marc Fouchard

L'exercice de ce lien montre que la solution générale du problème de deux corps est de la forme:

r=\frac{p}{1+e\cos(\theta-\omega)}

r est la distance entre un corps C_1 se trouvant à l'origine et le second corps C_2, et \theta l'angle entre une direction de référence et le vecteur \overrightarrow{C_1C_2} et e un nombre réel supérieur ou égale à zéro et p un nombre réel strictement supérieur à zéro.

Solutions du problème de 2 corps
figures/ellipsf2.gif
Crédit : Astrophysique sur Mesure / Benoît Mosser

Le but de cet exercice et d'étudier les diférentes familles de solution de cette équation.


Ex : les solutions du problème de deux corps

Auteur: Marc Fouchard

exerciceLes solutions du problème de deux corps

Difficulté :    Temps : 1 h

Question 1)

Montrer que si e=0 la solution est un cercle dont on déterminera le rayon.

Question 2)

Montrer que dans tous les autres cas, il existe un minimum pour r que l'on déterminera et que l'on notera q. La position pour laquelle cette distance est atteinte s'appelle le péricentre. A quoi correspond \omega ? Quand est-il pour le maximum de r ? La position pour laquelle cette distance, notée Q, est atteinte s'appelle apocentre lorsqu'elle existe.

Question 3)

On se place maintenant dans un repère orthonormé direct (C_1,\hat{\mathbf x},\hat{\mathbf y}) où l'axe des abscisses \hat{\mathbf x} est dirigé vers le pericentre. Pour un point M du plan on note r la distance C_1M et \alpha l'angle entre l'axe des abscisses et le vecteur \overrightarrow{C_1M}. Ecrire l'équation de la solution générale du problème de 2 corps en utilisant les coordonnées (x,y) de M dans le repère (C_1,\hat{\mathbf x},\hat{\mathbf y}) .On remarquera que l'équation obtenue est léquation générale d'une conique.

Question 4)

Montrer que si e=1 on obtient l'équation d'une parabole dont on déterminera les coordonnées du péricentre.

Question 5)

Montrer que si 0<e<1 on obtient l'équation d'une ellipse dont on déterminera le centre, le demi-grand axe et le demi-petit axe.

Question 6)

Montrer que si e>1 on obtient l'équation d'une hyperbole dont on déterminera le péricentre et les asymptotes.


Masse du trou noir central de la galaxie

Auteur : Alain Vienne

On dispose d'un tracé de l'orbite apparente d'une étoile (appelée S2) autour d'un point (SgrA) localisé par diverses méthodes comme étant au centre de notre Galaxie. Cette orbite est une ellipse qui diffère de l'orbite réelle car elle est vue en projection sur la sphère céleste. Le plan de la figure ci-dessous est le plan perpendiculaire à la ligne de visée: le plan de projection On se propose de trouver les caractéristiques géométriques de l'ellipse réelle, qui permettent finalement de calculer la masse centrale: sa valeur n'est compatible qu'avec celle d'un trou noir. On utilise la troisième loi de képler qui relie cette masse, la période et le demi-grand axe.

Orbite apparente d'une étoile autour de SgrA
figures/eso_trou_noir.png
Crédit : European Southern Observatory / ESO

Ex: Masse du trou noir central de la galaxie

Auteur: Alain Vienne

exerciceMasse du trou noir central de la galaxie

Difficulté : ☆☆   Temps : 1h30

Question 1)

Localiser le centre C de l'ellipse projetée

Question 2)

Tracer le grand axe projeté A_1 A_2

Question 3)

Calculer l'excentricité e=CF/CA_1, puis \sqrt{1-e^2}.

Question 4)

Tracer le diamètre conjugué B_1 B_2 de A_1 A_2 (donc le projeté du petit axe).

Question 5)

Tracer point par point le projeté du cercle principal, par l'homothétie de l'ellipse projetée à B_1 B_2 et de rapport 1 / \sqrt{1-e^2}.

La définition du cercle principal est donc:

Cercle principal d'une ellipse: Cercle de rayon a (demi grand axe de l'ellipse) de centre C

Question 6)
  • Le projeté du cercle principal est une ellipse dont le grand axe est la ligne des noeuds (intersection du plan de l'orbite et du plan de projection).
  • La longueur de ce grand axe donne la valeur 2a du grand axe de l'orbite réelle.
  • Le petit axe a pour valeur 2 a \cos ii est l'inclinaison du plan de l'orbite avec le plan de projection.

Mesurer 2a sur ce grand axe, puis convertir cette valeur en UA (unité astronomique) connaissant l'échelle de la figure (longueur de la "flèche" correspond à 0,05 seconde de degré) et la distance du Soleil au centre de la Galaxie (26 000 année-lumière)

Question 7)

Estimer la période du mouvement en utilisant les dates d'observations indiquées sur le tracé.

Question 8)

En déduire la masse M présente au foyer SgrA.


Géométrie dans l'espace

Auteurs: Stéphane Erard, Alain Vienne

Angles solides

On cherche à caractériser la partie de l'espace délimitée par un cône de sommet O et de demi-ouverture \Theta. On considère la calotte sphérique de rayon R et d'aire S(R) délimitée par ce cône. La quantité

\Omega = \frac{S}{R^2}

est indépendante de R. Elle mesure l'angle solide défini par le cône.

Cette quantité sans dimension est mesurée en stéradians (sr) - voir la définition des unités physiques.

AS_fig1b.png
Crédit : Astrophysique sur Mesure / Erard

Ex: angles solides

exerciceAngles solides

Difficulté :    Temps : 30 min

Question 1)

Quel est l'angle solide sous-tendu par un demi-espace ? Par l'espace complet ?

Question 2)

Quel est l'angle solide sous-tendu par une surface quelconque, de quoi dépend-il ? Ecrire l'application à une surface plane élémentaire dS inclinée sur la ligne de visée.

Question 3)

Donner l'expression différentielle de l'angle solide élémentaire en coordonnées sphériques.

300px-Small_solid_angle.png
Crédit : Sharayanan/GNU Free Documentation License

Question 4)

On considère maintenant une couronne circulaire élémentaire de demi-ouverture \alpha. Donner l'expression de l'angle solide en fonction de cet angle. En déduire l'angle solide sous-tendu par une calotte de demi-ouverture \Theta.

AS_fig2b.png
Crédit : Astrophysique sur Mesure / Erard

Question 5)

Que devient cette valeur si l'angle \Theta est petit ? Estimer l'angle solide sous lequel on voit le Soleil et la Lune depuis la Terre.

exercicePhotométrie

Difficulté : ☆☆   Temps : 30 min

Question 1)

On observe une surface plane à l'aide d'une caméra, dans la configuration de la figure ci-dessous : le détecteur de la caméra a une surface \sigma, on observe la source de surface S sous un angle e à la distance \Delta. Les dimensions sont telles que les angles solides considérés sont petits (S \ll \Delta^2).

Ecrire l'angle solide sous lequel le détecteur voit la source. Ecrire l'angle solide sous lequel la source voit la surface collectrice. Dériver une relation entre les deux angles solides.

Fig3.png
Crédit : Astrophysique sur Mesure / Erard

Question 2)

On note W' la puissance lumineuse diffusée par la source par unité d'angle solide. L'éclairement (ou irradiance) est la puissance recueillie par unité de surface de détecteur en provenance de la source.

Ecrire l'éclairement E en fonction de la puissance totale reçue par le détecteur (dW). Quelle est l'unité de mesure de cette quantité dans le Système International ? De quoi dépend-elle en général ?

Question 3)

La luminance (ou intensité spécifique) d'une source est la puissance lumineuse émise ou diffusée dans un angle solide élémentaire par unité de surface apparente.

Ecrire la luminance en fonction de dW. Dans quelle unité SI se mesure cette quantité ? De quoi dépend-elle en général ?

Question 4)

Ecrire l'éclairement reçu par le détecteur en fonction de la luminance de la source et de la distance. Que signifie cette expression si la source est ponctuelle (c'est-à-dire si elle ne remplit pas le champ de l'instrument) et dans le cas contraire ?

exerciceIntégrales angulaires

On trouvera ici des exercices d'intégration angulaire sur les quantités photométriques.


Croissant de Lune

Auteur: Alain Vienne (et le groupe IREM de Lille1)

On se propose d'établir les conditions pour que le croissant de Lune soit vu d'un lieu de la Terre comme une gondole:

croissant_1.png
Croissant de Lune vu horizontalement (comme une "gondole").
Crédit : Astrophysique sur Mesure / Vienne

Nous allons étudier ce problème par la trigonométrie sphérique qui permet de voir facilement les choses. La notion de sphère céleste est issue du fait que, à un lieu donné et à une date donnée, l'observateur n'a pas accès à la distance entre lui et l'objet céleste. Cet observateur peut alors considérer que tous ces objets sont à une même distance (arbitraire). Cela revient à dire que l'observateur n'appréhende que les directions issues de sa position. Or l'ensemble de ces directions s'identifie à une sphère centrée sur ce point.

Aucune formule n'est nécessaire pour résoudre l'exercice suivant. Il suffit de connaitre les bases. Soit:


Ex: Croissant de Lune

Auteur: Alain Vienne

exerciceExercice

Difficulté : ☆☆   Temps : 2h

Question 1)

Montrer que la condition d'horizontalité du croissant de Lune nécessite que la Lune et le Soleil aient le même azimut.

Question 2)

La condition de même azimut est donc une condition nécessaire. Réciproquement, si cette condition est réalisée, préciser les conditions sur les hauteurs du Soleil et de la Lune pour que le croissant soit vu comme une "gondole" et non à l'envers (un "D" renversé).

La hauteur est l'angle sur le vertical (cercle de même azimut). Il est compté de -90° à 90° par rapport à l'horizon.

Question 3)

Cette figure donne, pour chaque position de la Lune sur le même vertical que le Soleil (quand la condition est réalisée), l'aspect de celle-ci.

croissant_aspects.png
Phases de la Lune sous la condition de même azimut. Figure dans le vertical de la Lune (et du Soleil).
Crédit : Astrophysique sur Mesure / Vienne

La position du zénith sur le cercle est indicatif. Elle correspond au cas de la figure donnée en solution de la question précédente. Bien-sur, si le zénith est ailleurs sur le cercle, cela change les conditions de lever/coucher du Soleil et de la Lune. Faites d'autres figures en changeant le zénith de place (cela déplace aussi l'horizon).

Question 4)

En supposant que la Lune est toujours sur l'écliptique, donner les seuls endroits de la Terre où il est possible de voir le croissant de Lune horizontal.


En savoir plus: Croissant de Lune

Auteur: Alain Vienne (et le groupe IREM de Lille1)

Il est peut-être plus facile de voir les 2 cas (coplanaire et non-coplanaire) en raisonnant sur la sphère des fixes. Précédemment, on regardait le mouvement diurne d'un point de la sphère des fixes (le pôle E de l'écliptique) sur la sphère locale (de pôle Z). Ici, nous allons faire la démarche réciproque: on regarde le mouvement diurne de Z sur la sphère des fixes. On utilise la condition suivante:

L\in\textrm{\,\, grand cercle\,\, }(ZS)\,\,\,\,\textrm{(vertical du Soleil)}

En effet, nous avons vu que c'est la condition pour voir la Lune comme une gondole (ou tout au moins, la Lune à l'horizontal).

figures_croissantlune/croissant_zenith_sphere_fixes.png
Petit cercle des zéniths sur la sphère des fixes durant le mouvement diurne. Cas d'une zone tempérée (à gauche) et d'une zone intertropicale (à droite). Cas de la Lune sur l'écliptique (en haut) et cas de la Lune de part et d'autre de l'écliptique à 5^{\circ} au plus (en bas).
Crédit : Alain Vienne / IREM de Lille

Sur une sphère des fixes où on a placé l'équateur, l'écliptique et leur pôle, et pour une latitude \phi donnée, on trace le petit cercle correspondant aux positions prises par le zénith au cours du mouvement diurne (petit cercle des Z). A chacune de ces positions de Z, il correspond un seul grand cercle passant par le Soleil: c'est le vertical du Soleil. On obtient ainsi un "faisceau" de grands cercles dont les sommets sont le Soleil et le point diamétralement opposé. Sur la figure, pour ne pas encombrer celle-ci, nous en avons tracé qu'une partie puisque qu'on les a arrétés au niveau du petit cercle desZ. En réalité, ces grands cercles sont bien complets de sorte que toute la calotte sphérique se situant au dessus du petit cercle des Z est parcouru par ces grands cercles. Ainsi la sphère est divisée en deux parties: celle contenant chaque vertical du Soleil et l'autre.

La Lune doit se trouver dans la première partie (les "faisceaux" de la figure). La frontière entre ces deux parties correspond au vertical du Soleil qui est tangent au petit cercle des Z.

Cas de la Lune sur l'écliptique: Ce cas correspond aux 2 dessins du haut de la figure. En dehors de la zone intertropicale (à gauche), l'écliptique coupe les "faisceaux" qu'en ses sommets: au Soleil et au point diamétralement opposé. Si on impose à la Lune d'être sur l'écliptique, il n'y a qu'en ces points que la condition est réalisée (éclipses). Par contre, dans la zone intertropicale, tout l'écliptique est contenu dans les "faisceaux". Ainsi la condition est réalisée deux fois par jour comme on l'a vu dans précédemment.

Cas où la Lune est de part et d'autre de l'écliptique:

L'orbite de la Lune est inclinée d'environ 5^{\circ} sur l'écliptique. Son noeud qui permettrait de positionner le grand cercle correspondant à son orbite, a un mouvement de rétrograde de -19^{\circ},34/an (période: 18,6 ans). Pour ne pas rentrer dans trop de détails superflus à la compréhension, nous allons simplement considérer que la Lune est de part et d'autre de l'écliptique sur une bande large de 10^{\circ}. Bien-sur, il ne faut pas oublier que la Lune parcourt en fait un grand cercle contenu dans cette bande: la position en longitude dans cette bande dépend de la date dans la lunaison et la position "verticale" dans cette bande dépend de la position du noeud de l'orbite lunaire.

On remarque ainsi qu'au voisinage de la pleine Lune ou au voisinage de la nouvelle Lune, la condition de "Lune horizontale" est possible partout sur la Terre. Mais on se rend bien compte que, loin des tropiques, la zone est étroite. Elle s'agrandit au fur et à mesure que le lieu considéré s'approche du tropique.

Dans le cas d'un lieu dans la zone intertropicale, la possibilité d'une telle condition est grande. La probabilité de réalisation l'est donc aussi. Cependant cette probabilité n'est pas 1, car on voit apparaitre une petite zone de la bande lunaire qui croise la partie où il n'y a pas de vertical du Soleil (en dehors des "faisceaux"). Cette zone est petite et proche du Soleil. Ainsi même dans la zone intertropicale, il peut y avoir des jours où la Lune n'est pas vue à l'horizontal. Cela se produit pour des positions particulières de l'orbite lunaire et pour des dates proches de la pleine Lune ou de la nouvelle Lune.


Algèbre


Introduction

On trouvera dans cette partie des exercices portant sur :


Anneaux des polynômes et fractions

Auteur: Alain Vienne

Une factorisation du polynôme de la méthode de Laplace

Auteur: Alain Vienne

Lors de la découverte d'un nouvel objet dans le système solaire, on souhaite rapidement connaitre sa trajectoire. Celle-ci est généralement héliocentrique et, dans un premier temps, on la suppose képlérienne. Or les observations terrestres donnent uniquement la direction de l'astre mais pas sa distance. La méthode de Laplace propose un moyen qui, à partir de 3 observations de direction faites à des dates assez rapprochées, donne les vecteurs position et la vitesse de l'astre. Le détail de la méthode peut être vu dans le cours suivant: Dynamique du système solaire. On peut y voir notamment que la méthode conduit à chercher les racines d'un polynôme de degré 8.

P(x)=x^8 + a_6 x^6 + a_3 x^3 + a_0

Il y est affirmé qu'il y a 4 racines réelles (1 négative, 3 positives) et 4 complexes non réelles. Cette affirmation est étudiée et montrée dans l'exercice Les racines du polynôme de la méthode de Laplace. Ici, on montre que x=ss est la distance Terre-Soleil, et, on utilise cette racine pour factoriser le polynôme.

Voici à titre d'exemple le graphe du polynôme dans le cas de 3 observations de Jupiter à son opposition (courbe "complète" et un agrandissement):

lagrange_typ.pnglagrange_typ_agr.png
Le polynome de la méthode de Laplace dans le cas de 3 observations de Jupiter (courbe "complète" et un agrandissement). L'axe horizontal est gradué en ua. On note que ce polynôme n'est pas très bien conditionné car la vue d'ensemble ne donne pas une idée des racines ni même du nombre de ces racines. La deuxième figure est agrandissement sur la partie utile. On note la racine x=1 ua (s) et les 2 autres racines dont celle à 5 ua.
Crédit : Astrophysique sur Mesure / Vienne

Ex: Une factorisation du polynôme de la méthode de Laplace

Auteur: Alain Vienne

exerciceFactorisation du polynôme de la méthode de Laplace

Difficulté :    Temps : 30mn

Le polynôme issu de la méthode de Laplace a la forme suivante:

P(x)=x^8 + (2t\beta-t^2-s^2) x^6 + 2\alpha(\beta-t) x^3  - \alpha^2

s est la distance Terre-Soleil et t=-\frac{\alpha}{s^3}.

\alpha et \beta sont des coefficients réels issus de la géométrie du problème.

Question 1)

Vérifier que x=s est racine de P.

Question 2)

Mettre en facteur (x-s) dans P.


Les racines des polynômes de Legendre

Auteur: Alain Vienne

En Mécanique Céleste, on est souvent conduit à utiliser les polynômes de Legendre que l'on note ici P_n .

C'est le cas, par exemple, dans le développement du potentiel terrestre. Si on suppose que la Terre est un sphéroïde, le potentiel peut s'écrire:

U(r,\varphi)=\frac{KM_T}{r} \ [1 - \sum_{m=1}^{\infty} J_{2m} (\frac{a_e}{r})^{2m}  P_{2m}(\sin\varphi) \ ]

K est la constante de gravitation de la Terre, M_T la masse totale de la Terre, a_e son rayon équatorial et J_{2m} des coefficients numériques. r et \varphi sont le rayon et la latitude du point pour lequel on évalue le potentiel U .

Un autre exemple d'utilisation est de considérer 2 corps M et M' décrivant autour d'un centre P des orbites proches d'un mouvement elliptique. Pour décrire les perturbations (gravitationnelles) entre M et M', on doit écrire l'inverse de la dsitance entre M et M', 1/\Delta, en fonction de leurs éléments d'orbite. On montre facilement que:

\frac{1}{\Delta} = \frac{1}{r'} (1-2\rho \cos S + \rho^2)^{-1/2}

Avec r=PM, r'=PM', \rho = r/r' et S l'angle entre M et M' vu de P.

Cette dernière expression est développée en puissance de \rho grâce aux polynômes de Legendre:

(1-2\rho \cos S + \rho^2)^{-1/2} =  \sum_{n=0}^{\infty} \rho^n  P_n(\cos S)

Ce développement est rapidement convergent si \rho est petit. C'est le cas si, par exemple, M est la Terre, P le Soleil et M' un satellite artificiel.

Plus de détails de ces développement peuvent être vus dans le cours de Mécanique Céleste de Luc Duriez.

Les polynômes de Legendre ont de nombreuses propriétés. Celle que nous allons utiliser dans l'exercice qui suit est la formule de Rodrigues:

P_m(x)=\frac{1}{2^m \  m!} \  \frac{d^m}{dx^m} (x^2-1)^m

Cette formule va nous permettre de montrer que l'équation P_m(x)=0 a toutes ses racines dans [-1,+1] et en a m distinctes.


Ex: Les racines des polynômes de Legendre

Auteur: Alain Vienne

exerciceLes racines des polynômes de Legendre

Difficulté : ☆☆   Temps : 1h (pour une rédaction correcte)

Les polynômes de Legendre, bien connus en Mécanique Céleste, peuvent se déterminer par la formule de Rodrigues:

P_m(x)=\frac{1}{2^m \  m!} \  \frac{d^m}{dx^m} (x^2-1)^m

Question 1)

Montrer que l'équation P_m(x)=0 a toutes ses racines dans [-1,+1] et en a m distinctes.


Systèmes linéaires

Auteurs: S. Renner, Marc Fouchard

Configurations d'équilibre de satellites co-orbitaux

Auteur: S. Renner

Date de création: 8 avril 2009

Dans le système solaire, on trouve plusieurs exemples de configurations où des petits satellites co-orbitaux sont en orbite autour d'un corps central (planète) beaucoup plus massif. Dans le système de Saturne, les satellites Hélène et Pollux sont en libration autour des points de Lagrange L_4 et L_5 de Dioné. De même, Télesto et Calypso sont respectivement au point L_4 et L_5 de Téthys. D'autre part, les satellites co-orbitaux Janus et Epiméthée ont des orbites en fer à cheval (cf. figure des points de Lagrange) autour de leur point L_3 mutuel.

Dans un autre contexte, la présence de 4 arcs de matière (des "morceaux" d'anneau) autour de Neptune pourrait s'expliquer par l'existence de satellites co-orbitaux (non découverts) qui confineraient la poussière observée de l'anneau formant les arcs.

Le but ici est de redémontrer des résultats généraux sur les configurations stationnaires (planes) de N satellites co-orbitaux, en orbite autour d'une planète beaucoup plus massive (problème à N+1 corps, plan). Ces résultats généralisent le problème des points de Lagrange et sont extraits de Renner, S. & Sicardy, B., Celestial Mechanics and Dynamical Astronomy, 88, 397-414, 2004.

Plus précisément, on va montrer que l'existence de solutions stationnaires planes pour le problème à N+1 corps dépend de la parité de N. Plus précisément, si N est impair, et pour une configuration angulaire donnée, il existe toujours un ensemble de masses (positives ou négatives) qui réalise un équilibre! Pour N pair au contraire, il n'y a à priori pas de combinaison de masses qui réalise un équilibre, pour des séparations angulaires données entre les satellites.


Ex: Configurations d'équilibre de satellites co-orbitaux

Auteur: S. Renner

exerciceEquilibres de satellites co-orbitaux

Difficulté : ☆☆   Temps : 2h30

On considère N satellites co-orbitaux en orbite autour d'un corps central supposé ponctuel de masse M. On note n_0 la vitesse angulaire moyenne et r_0 le rayon orbital moyen des satellites.

On suppose le problème plan, et on se place dans un repère centré sur M et tournant à la vitesse angulaire n_0.

Le mouvement de chaque satellite est décrit par les coordonnées (\phi_i,\xi_i), i=1,...,N, où \phi_i est la longitude du satellite i par rapport à une longitude de référence arbitraire, et \xi_i=\Delta r_i/r_0 l'excursion radiale relative du satellite par rapport au rayon moyen r_0 (voir figure ci-dessous).

intro.jpg
Notations pour la dynamique des satellites co-orbitaux (cf. texte).
Crédit : Astrophysique sur Mesure / Renner

On peut montrer que la dynamique de chaque satellite est régie par le système d'équations différentielles suivant :

\left\{ \begin{array}{lr} \displaystyle  {\dot \phi_i}= -{3 \over 2} \xi_i \\ \\ \displaystyle {\dot \xi_i}= -2  \sum_{j \neq i} m_j f'(\phi_i - \phi_j) \end{array} \right.

avec

f(\phi)= \cos \phi - {1 \over 2|\sin \phi/2|}, f'(\phi)= \sin \phi \left[ -1 + {1 \over 8|\sin \phi/2|^3} \right],  f''(\phi)= -\cos \phi - {3 + \cos \phi \over 16|\sin \phi/2|^3} \cdot

Question 1)

Ecrire les deux équations algébriques donnant les points fixes du système.

Question 2)

Que signifie la première relation?

Question 3)

La seconde relation correspond en fait à N équations linéaires des masses. Ecrire ce système sous forme matricielle. On note M_N la matrice obtenue.

Question 4)

Que peut-on dire de la matrice M_N?

Question 5)

Trouver les points d'équilibre dans le cas N=2.

Question 6)

On cherche à trouver tous les angles \phi_1,...,\phi_N tels que (m_1,...,m_N) soit solution de l'équation matricielle, avec m_1,...,m_N \geq 0. Il est évidemment impossible de résoudre cette équation analytiquement pour N quelconque. On peut néanmoins déduire des propriétés générales sur les solutions.

soit (\phi_1,...,\phi_N) \in [0,360^\circ[^N tel que \phi_i \neq \phi_j pour tout i \neq j (f' n'est pas définie en 0).

On suppose que N est impair. Déterminer le rang de la matrice M_N, puis en déduire qu'il existe une famille à k paramètres, avec k entier impair, de vecteurs (m_1,...,m_N) \in \mathbb{R}^N pour laquelle (\phi_1,...,\phi_N) est une configuration stationnaire.

Question 7)

On suppose que N est pair. Déterminer le rang de la matrice M_N, et en déduire qu'en général il n'existe pas de famille de vecteurs qui réalise un équilibre.

Question 8)

Dans le cas où N est pair, quelle propriété doit vérifier la matrice M_N pour pouvoir obtenir des solutions non-triviales (m_1,...,m_N) ?

Question 9)

Vérifier les deux questions précédentes avec le cas N=2.


En savoir plus: Configurations d'équilibre de satellites co-orbitaux

ensavoirplusEn savoir plus

Les équations du mouvement sont donc:

\left\{ \begin{array}{lr} \displaystyle  {\dot \phi_i}= -{3 \over 2} \xi_i \\ \\ \displaystyle {\dot \xi_i}= -2  \sum_{j \neq i} m_j f'(\phi_i - \phi_j) \end{array} \right.

avec

f(\phi)= \cos \phi - {1 \over 2|\sin \phi/2|}, f'(\phi)= \sin \phi \left[ -1 + {1 \over 8|\sin \phi/2|^3} \right],  f''(\phi)= -\cos \phi - {3 + \cos \phi \over 16|\sin \phi/2|^3} \cdot

Pour établir ces équations, on a fait les hypothèses suivantes:

La première équation n'est rien d'autre que la vitesse keplerienne différentielle de chaque satellite par rapport à l'orbite de référence de rayon r_0. La seconde équation contient, sous forme dérivée, tous les termes résultant des interactions gravitationnelles mutuelles entre les satellites.

La fonction f(\phi) est la somme des potentiels direct et indirect exercé par un satellite donné sur les autres co-orbitaux. C'est une fonction paire, et son graphe est tracé ci-dessous avec ses dérivées première et seconde f'(\phi) et f''(\phi).

potentiel.jpgpotderiv1.jpgpotderiv2.jpg
La fonction f(\phi), décrivant le potentiel créé par un satellite sur une particule co-orbitale, avec ses dérivées première et seconde f'(\phi) et f''(\phi).
Crédit : Astrophysique sur Mesure / Renner

Puisque  f' est impaire, il est facile de montrer d'après les équations du mouvement que \displaystyle \sum_{i} m_i \xi_i = {\rm constante}. Le rayon de référence  r_0 étant arbitraire, il peut être choisi de telle manière que \displaystyle \sum_{i} m_i \xi_i = 0, sans perte de généralité. Ainsi le système possède les intégrales premières suivantes :

\left\{ \begin{array}{lr} \displaystyle \sum_{i} m_i  \xi_i  = 0  \\ \\ \displaystyle \sum_{i} m_i \phi_i  = {\rm constante},  \end{array} \right.

qui résultent de la conservation du moment cinétique total. Cette conservation résulte elle-même de l'invariance par rotation du problème. Il existe une autre intégrale première :

J= \sum_{i} m_i \left[ - {3 \over 4} \xi_i^2 + \sum_{j \neq i} m_j  f(\phi_i - \phi_j) \right]

Elle exprime la conservation de l'énergie dans le repère tournant, et est appelée constante de Jacobi.


Exposants de Lyapunov

Auteur: Marc Fouchard

Date de création: 4 avril 2011

On considère le système dynamique suivant:

\dot{{\bf x}}={\bf f} ({\bf x}),

{\bf x} est un vecteur de dimension n, et {\bf f} une fonction vectorielle de dimension n continue et dérivable.

On appelle exposant de Lyapunov en {\bf x}_0 suivant le vecteur {\bf w}_0 la quantité:

\chi({\bf x}_0,{\bf w}_0)=\lim_{t \to +\infty}\frac{1}{t}\ln || {\bf w}(t) ||,

{\bf w}(t) est solution de l'équation différentielle:

\dot{{\bf w}}=\mathcal{J}\cdot {\bf w},

avec \mathcal J=\frac{\partial {\bf f}}{\partial {\bf x}} est le Jacobien de {\bf f}.

Cette équation, appelée équation variationnelle, est associée à l'équation différentielle décrivant l'évolution de {\bf x}. Les vecteurs {\bf x}_0 et {\bf w}_0 sont les conditions initiales de ces équations différentielles.

On appelle généralement le vecteur {\bf w} le vecteur tangent à la trajectoire. Il évolue dans un espace appelé espace tangent qui peut être identifié à \mathcal{R}^n.

Les exposants de Lyapunov permettent de savoir si la trajectoire passant par {\bf x}_0 est chaotique ou pas. Par exemple sur la figure ci-dessous on peut voir qu'un dérivé des exposants de Lyapunov (Exposant de Lyapunov Rapide) se comporte de manière différente pour une trajectoire régulière (accroisement linéaire) et pour une trajectoire chaotique (accroissement exponentiel). Dans la suite on va étudier les exposants de Lyapunov associés à la trajectoire passant par {\bf x}_0 à t=0 et démontrer quelques propriétés élémentaires de ces exposants, en particulier leur similarité avec le spectre des valeurs propres d'en endomorphisme.

Evolutions d'exposants de Lyapunov Rapides
FLI.png
Evolution de l'exposant de Lyapunov Rapide (à droite) pour trois trajectoires (à gauche). Les trajectoires rouge et verte sont régulières alors que la trajectoire noire est chaotique.
Crédit : Astrophysique sur Mesure / Marc Fouchard

Ex: exposants de Lyapunov

Auteur: Marc Fouchard

exerciceexposants de Lyapunov

Difficulté : ☆☆☆   Temps : 1h30

Question 1)

Soit une orbite périodique de condition initiale {\bf x}_0 et de période \tau. Ainsi après une période on a {\bf w}(\tau)=\mathcal{M} \cdot {\bf w}_0, où \mathcal{M} est une matrice carrée de dimension n, et pour t=k\tau \quad (k\in\mathbb{N}) on a {\bf w}(t)=\mathcal{M}^k \cdot {\bf w}_0. Montrer que si {\bf w}_0 est un vecteur propre de la matrice \mathcal M associé à la valeur propre \lambda_0 alors:

\chi({\bf x}_0,{\bf w}_0)=\frac{\ln |\lambda_0|}{\tau}

Question 2)

Ainsi on voit que pour les orbites périodiques les exposants de Lyapunov sont reliés au spectre, c'est-à-dire l'ensemble des valeurs propres, du Jacobien de \bf f. Dans le cas général la matrice \mathcal M n'existe pas. Cependant on va voir qu'on peut définir un spectre d'exposants de Lypunov associé au Jacobien de {\bf f} ayant certaines similarités avec le spectre des valeurs propres d'une matrice.

Montrer que \chi ({\bf x}_0,\lambda{\bf w}_0)=\chi({\bf x}_0,{\bf w}_0), pour \lambda \in \mathcal R.

Question 3)

Montrer que \chi({\bf x}_0,{\bf w}_0+{\bf w}_1})\le\max \left( \chi({\bf x}_0,{\bf w}_0),\chi({\bf x}_0,{\bf w}_1)\right).

Question 4)

Avec la convention \chi({\bf x_0,\vec{\bf 0})=-\infty, montrer que pour tout \alpha \in {\mathcal R}, l'ensemble \mathcal{L}(\alpha)=\left\Big\lbrace {\bf w} \in {\mathcal R}^n \quad {\rm t.q.} \quad \chi({\bf x}_0,{\bf w})\le \alpha \right\Big\rbrace, forme un sous-espace vectoriel de \mathcal{R}^n.

Question 5)

En déduire qu'il existe au plus n exposants de Lyapunov distincts pour une trajectoire donnée.

remarqueRemarque

On voit ainsi que les exposants de Lyapunov forment un spectre au même titre que les valeurs propres pour un opérateur linéaire. Si on a n exposants distincts et tels que \alpha_1>\dots\alpha_n, avec \alpha_i=\chi({\bf x}_0,{\bf w}_i), alors les vecteurs {\bf w}_i, \quad i=1,\dots,n forment une base de \mathcal{R}^n. Dans la pratique, on ne connait pas les vecteurs permettant de déterminer les exposants de Lyapunov. Mais si on prend un vecteur au hasard il apartiendra à \mathcal{L}(\alpha_1)=\mathcal{R}^n mais peu probablement à \mathcal{L}(\alpha_2). En effet pour qu'il y appartienne il faudrait que la composante de ce vecteur suivant {\bf w}_1 soit égale à zéro. Lors d'un tirage au hasard cette probabilité est nulle.

Ainsi dans la pratique on calcule en général uniquement l'exposant maximal de Lyapunov \alpha_1. Il permet de savoir si une orbite est stable ou chaotique. En effet dans le cas d'une orbite stable la norme du vecteur tangent va très probablement augmenter linéairement avec le temps, ainsi \alpha_1=0, alors que dans le cas chaotique elle va augmenter très probablement exponentiellement avec le temps, ainsi \alpha_1>0.


Applications linéaires et matrices

Auteurs: Alain Vienne, Marc Fouchard

Système de Laplace-Lagrange

Alain Vienne

Les mouvements de Jupiter et de Saturne sont très proches d'un mouvement képlérien. En effet, chacune de ces planètes est principalement attirée par le Soleil mais très peu par l'autre planète (la masse du Soleil est 1047 fois celle de Jupiter et 3498 fois celle de Saturne; ces 2 planètes étant les plus massives du système solaire). Leurs éléments d'orbite, notamment le demi-grand axe, l'excentricité (e) et la direction du péricentre (\varpi), sont quasi constants. Plus précisément, soit ils varient peu (oscillations rapides de faible amplitude), soit ils varient lentement ("variations séculaires"). On dit que l'influence de Jupiter sur le mouvement de Saturne est une perturbation (et vice versa) du mouvement képlérien. L'objet de la mécanique céleste dans le cas de systèmes perturbés, est de modéliser ces variations.

Laplace (1749-1827) avait déjà montré que les demi-grands axes des planètes n'avaient pas de variations séculaires (plus précisément: à un certain degré d'approximation, les demi-grands axes des planètes n'ont que des petites variations périodiques). Ce qui était, à l'époque, un argument fort en faveur de la stabilité du système solaire.

Il fallait quand même s'assurer que les excentricités n'atteignent pas de valeurs trop grandes. En effet, de grandes excentricités conduisent vite à des collisions! L'objet de cette application est de voir que les variations d'excentricités sont bornées.

On simplifie notablement le calcul et la compréhension en utilisant la variable complexe suivante:

z = e \exp \imath \varpi \ \ \textrm{avec} \ \ \imath = \sqrt{-1}

Par exemple, on verra dans l'exercice suivant que l'"execntricité complexe" z_J asssociée à Jupiter a le mouvement suivant:

lap-lag/laplace_lagrange.gif
"Excentricité complexe" de Jupiter (z_J) qui montre que l'exentricité de Jupiter est bornée.
Crédit : Astrophysique sur Mesure / Bessou Vienne

Ex: Système de Laplace-Lagrange

Auteur: Alain Vienne

exerciceExercice

Difficulté :    Temps : 1h30

Question 1)

La partie linéaire des équations séculaires relatives à z_J (Jupiter) et à z_S (Saturne) peut s'écrire:

\frac{d\alpha}{dt} = \imath A \alpha \  \rm{avec} \  A = \left( \begin{array}{rr}0.020031 & -0.013114\\-0.032335 & 0.049538\end{array} \right)  \rm{en\ } $''/j$

avec \alpha = \left( \begin{array}{c} z_J \\ z_S \end{array} \right)

Montrer que A est diagonalisable et donner ses valeurs propres (appelées ici, "fréquences propres").

Remarque: on notera les valeurs propres \nu_5 et \nu_6. Ces indices 5 et 6 font référence respectivement à la cinquième et à la sixième ligne de la matrice obtenue par Le Verrier lorsque celui-ci considérait les 8 planètes.

Question 2)

Intégrer le système différentiel en recherchant pour z_J et z_S une solution sous la forme de termes périodiques. On montrera que les valeurs propres de la matrice A sont les fréquences de ces termes périodiques.

Question 3)

Donner les périodes de ces termes périodiques en années.

Question 4)

Sachant qu'à t=0, on a les valeurs:

e_J = 0,04833475 et \varpi_J = 12^{\circ} 43' 15''

e_S = 0,05589231 et \varpi_S = 91^{\circ} 05' 54''

calculer les constantes d'intégration de la solution, puis les amplitudes des termes à très longues périodes des solutions de z_J et z_S (on ne demande pas les phases)

Question 5)

En déduire les valeurs extrêmes que peuvent atteindre les excentricités de Jupiter et de Saturne.

remarqueRemarque

Le fait que le système de Laplace-Lagrange conduit à des valeurs bornées de l'excentricité est illustré par la figure suivante. C'est la variable z_5 qui est représentée.

quasiperiodique_p.png
Solution de la variable en excentricité de Jupiter (z_J) issue d'un système séculaire complet (non linéarisé et avec les 8 planètes).
Crédit : Astrophysique sur Mesure / Vienne

Cette solution diffère de la notre car elle est issue d'un système séculaire complet, c'est à dire non linéarisé et avec les 8 planètes.


Pendule et matrice de Floquet

Auteur: Alain Vienne

Beaucoup de modèles dynamiques, après maintes transformations (hypothèses simplificatrices, moyennisations, ...), ressemblent au modèle du pendule (masse à une distance constante d'un point fixe sous l'effet de la pesanteur). Ici nous allons nous intéresser à un type d'équation du pendule correspondant à l'équation de Mathieu:

\ddot{\theta} + \omega_0^2(1+\varepsilon \cos \omega t ) \theta = 0

Si \varepsilon est nul, c'est l'équation d'un pendule simple pour de petites oscillations. Dans ce cas, \omega_0^2 est inversement proportionnel à la longueur du pendule. On rappelle que la période T_0 est alors \frac{2\pi}{\omega_0}.

Ici \epsilon est un petit paramètre. On dit que le modèle du pendule simple est perturbé. L'équation de Mathieu est un cas particulier de l'équation

\ddot{\theta} +  w^2(t) \theta = 0

w(t) est une fonction périodique de période T qui est utilisée en Mécanique Céleste pour l'étude du mouvement de la Lune.

De manière plus ludique, ces équations peuvent modéliser le mouvement d'une balançoire dont le passager se lève et s'assied (périodiquement) afin de s'élancer. Le fait de se lever et de s'assoir régulièrement revient à déplacer le centre de gravité du passager et donc, revient à faire varier périodiquement la longueur du pendule (ici la balançoire).

L'exercice qui suit ne résoud pas l'équation différentielle. Il cherche simplement à savoir dans quelles conditions la solution est bornée ou non (problème de stabilité). Il est insipré du théorème de Gustave Floquet (1847-1920). C'est un exercice de la théorie des équations différentielles mais il utilise beaucoup l'algèbre linéaire d'où sa présence dans cette partie.


Ex: Pendule et matrice de Floquet

Cet exercice est un classique de la théorie des équations différentielle. On le trouve donc dans la partie "Equations différentielles linéaires". Cependant il utilise beaucoup l'algèbre linéaire d'où sa présence dans cette partie.

Auteur: Alain Vienne

exerciceExercice

Difficulté : ☆☆   Temps : 2h30

Voir l'énoncé


Lever et coucher du Soleil

Marc Fouchard

L'animation ci-dessous illustre le mouvement diurne du Soleil au dessus de l'horizon en un point de latitude \varphi. Le point O correspond à l'observateur. Il observe le mouvement du Soleil au cours d'une journée. Ce mouvement correspond uniquement à un changement de direction dans laquelle le Soleil est observé. Ainsi on peut représenter ce mouvement par un point se déplaçant sur une sphère (sphère céleste) centrée sur O est de rayon qu'on prendra arbitrairement égale à 1.

Sur cette sphère, on peut représenter toutes les directions parallèles à l'horizon, ce qui défini l'horizon céleste. Les astres dont la direction se trouve en dessous de l'horizon céleste ne sont pas visibles depuis O. Sur l'horizon céleste on peut représenter les directions du Sud \mathcal{S}ud, de l'Ouest \mathcal{O}uest, du Nord \mathcal{N}ord et de l'Est \mathcal{E}st. De même, on peut représenter la direction perpendiculaire à l'horizon: le Zénith (Z) et la direction parallèle à l'axe de rotation de la Terre: le pôle céleste Nord (P). Le plan qui coupe la sphère céleste perpendiculaire à la direction (OP) et passant par O, s'appelle l'équateur céleste. Sur l'équateur céleste on note \mathcal{S}ud' la direction du Sud. On note Z' le Nadir, qui correspond à la direction opposée au Zénith, et on note P' le pôle céleste Sud qui correspond à la direction opposée au pôle céleste nord. On remarquera que les points O, \mathcal{N}ord, P et Z sont coplanaires avec \widehat{\overrightarrow{O\mathcal{N}ord},\overrightarrow{OP}}=\varphi.

Ainsi, au cours d'une journée la Terre tourne autour d'un axe parallèle à (OP). Pour l'observateur, ceci ce traduit par un déplacement des astres observés sur des cercles parallèles à l'équateur céleste.

Soit S le point de la sphère céleste indiquant la direction dans laquelle est observé le Soleil depuis O. On appelle M l'intersection de l'arc de grand cercle (PSP') avec l'équateur céleste et N l'intersection de l'arc de grand cercle (ZSZ') avec l'horizon céleste.

On note \delta l'angle \widehat{SOM}, H l'angle \widehat{\mathcal{S}ud'OM}, h l'angle \widehat{SON} et A l'angle \widehat{\mathcal{S}udON}. (A,h) sont appelées les coordonnées locales, alors que (H,\delta) sont les coordonnées horaires. Au court du mouvement diurne d'une étoile seule \delta est constant. Pour le Soleil \delta varie au cours de l'année, mais on peut le considérer constant sur une journée. L'animation permet de modifier \delta afin de voir les variations dans le mouvement diurne en fonction de \delta.

Le but de cette exercice est d'établir des relations entre les coordonnées horaires et locales par des rotations puis d'utiliser ces relations pour calculer les heures de lever et de coucher du Soleil aux solstices et aux équinoxes.

application.png


Ex: Lever et coucher du Soleil

Auteur: Marc Fouchard

exerciceExercice

Difficulté : ☆☆   Temps : 1h

Question 1)

Soit les repères orthonormés suivants :

  • \mathbf{\mathcal{R}}_{SH} le repère (\overrightarrow{OS},\overrightarrow{OU},\overrightarrow{OT}), où \overrightarrow{OT} est perpendiculaire à \overrightarrow{OS} dans le plan (POS) et tel que l'angle \widehat{\overrightarrow{OP},\overrightarrow{OT}} soit inférieur à \pi/2 en valeur absolue et \overrightarrow{OU} complète un repère orthonormé direct.
  • \mathbf{\mathcal{R}}_M le repère (\overrightarrow{OM},\overrightarrow{OU},\overrightarrow{OP}),
  • \mathbf{\mathcal{R}}_H le repère (\overrightarrow{O\mathcal{S}ud'},\overrightarrow{O\mathcal{O}uest},\overrightarrow{OP}),
  • \mathbf{\mathcal{R}}_{SL} le repère (\overrightarrow{OS},\overrightarrow{OV},\overrightarrow{OW}), où \overrightarrow{OW} est perpendiculaire à \overrightarrow{OS} dans le plan (ZOS) et tel que l'angle \widehat{\overrightarrow{OZ},\overrightarrow{OW}} soit inférieur à \pi/2 en valeur absolue et \overrightarrow{OV} complète un repère orthonormé direct.
  • \mathbf{\mathcal{R}}_N le repère (\overrightarrow{ON},\overrightarrow{OV},\overrightarrow{OZ}),
  • \mathbf{\mathcal{R}}_L le repère (\overrightarrow{O\mathcal{S}ud},\overrightarrow{O\mathcal{O}uest},\overrightarrow{OZ}).

Le but de l'exercice est d'établir relations entre les coordonnées horaires et coordonnées locales du Soleil en utilisant des matrices de rotation entre les différents repères.

Montrer que l'on passe du repère \mathcal{R}_{SH} au repère \mathcal{R}_M par une rotation d'angle -\delta et d'axe \overrightarrow{OU}. Donner la matrice de passage \mathcal{M} de la base de \mathcal{R}_{SH} à celle de \mathcal{R}_M .

Question 2)

Montrer que l'on passe du repère \mathcal{R}_M au repère \mathcal{R}_H par une rotation d'angle -H et d'axe \overrightarrow{OP}. Donner la matrice de passage \mathcal{N} de la base de \mathcal{R}_{M} à celle de \mathcal{R}_H .

Question 3)

Montrer que l'on passe du repère \mathcal{R}_{SL} au repère \mathcal{R}_N par une rotation d'angle -h et d'axe \overrightarrow{OV}. Donner la matrice de passage \mathcal{P} de la base de \mathcal{R}_{SL} à celle de \mathcal{R}_N .

Question 4)

Montrer que l'on passe du repère \mathcal{R}_N au repère \mathcal{R}_L par une rotation d'angle -A et d'axe \overrightarrow{OZ}. Donner la matrice de passage \mathcal{Q} de la base de \mathcal{R}_{N} à celle de \mathcal{R}_L.

Question 5)

Montrer que l'on passe du repère \mathcal{R}_H au repère \mathcal{R}_L par une rotation d'angle \pi/2-\varphi et d'axe \overrightarrow{O\mathcal{O}uest}. Donner la matrice de passage \mathcal{T} de la base de \mathcal{R}_{H} à celle de \mathcal{R}_L.

Question 6)

Ecrire les coordonnées de S en fonction de \delta et H dans \mathcal{R}_H.

Question 7)

Ecrire les coordonnées de S en fonction de h et A dans \mathcal{R}_L. Puis les coordonnées de S dans le repère \mathcal{R}_H en fonction de h, A et \varphi. En déduire trois relations, dépendant de \varphi, entre les coordonnées horaires et les coordonnées locales du Soleil.

Question 8)

En déduire les valeurs de l'angle horaire H au moment du lever et du coucher du Soleil en fonction de \varphi et \delta.

Question 9)

En déduire les valeurs de H (mesuré entre -12h et +12h) et de la durée du jour au moment des équinoxes (\delta =0), du solstice d'été \delta= 23,5^{\circ} et du solstice d'hiver (\delta=-23,5^{\circ}) en un point de latitude \varphi=50^{\circ} (approximativement la ville de Lille, France)


Etoiles doubles

Marc Fouchard

Les étoiles doubles correspondent à des couples d'étoiles reliées gravitationnellement l'une à l'autre. Ainsi, les deux étoiles effectuent un mouvement elliptique autour du centre de gravité du couple. La détermination des paramètres de cette ellipse, et en particulier de son demi-grand axe, est particulièrement importante parce qu'elle permet d'obtenir la masse des étoiles.

L'objet de ce petit exercice est juste d'établir le système permettant de déterminer les paramètres de l'équation algébrique d'une conique.


Ex: Etoiles doubles

Auteur: Marc Fouchard

exerciceEtoiles doubles

Difficulté :    Temps : 20 mn

Question 1)

Quelle est l'équation générale d'une conique dans le plan.

Question 2)

Pour une ellipse, on a la contrainte supplémentaire que a\ne0 (entre autre). En déduire une équation de l'ellipse contenant cinq paramètres.

Question 3)

On a donc cinq paramètres indépendants à déterminer. Combien, au moins, nous faut-il d'observation pour pouvoir déterminer les paramètres ?

Question 4)

Soit M_i \left(\begin{array}{c}x_i \\ y_i \end{array}\right) , i=1,\cdots,5, ces 5 observations. Ecrire sous forme matricielle le système à résoudre.

Question 5)

Donner une astuce pour se ramener à la résolution d'un système à trois inconnues que l'on déterminera.


Théorie Spectrale

Auteurs: Arnaud Beck, Jérôme Thiébaut

Onde de Langmuir

Auteur: Arnaud Beck

Un plasma est un gaz dont les constituants, au lieu d'être neutres, sont électriquement chargés. Cela en fait un milieu bien plus complexe qu'un fluide traditionnel.

Dans un gaz normal, toutes les perturbations se propagent de la même manière et à la même vitesse. Ainsi, si quelqu'un fait vibrer un gaz à un point A, cette vibration va se propager jusqu'au point B à la vitesse du son, indépendamment de la fréquence de la vibration. Ce sont les ondes sonores.

Dans un plasma, les interactions entre particules chargées permettent à un grand nombre d'ondes différentes d'exister. Chacune de ces ondes propage des perturbations qui peuvent être de natures différentes (charge, pression, champ électrique, champ magnétique ...) et ont des vitesses différentes qui dépendent, entre autres, de la fréquence de la perturbation.

Dans cet exercice, on propose de retrouver la relation de dispersion d'une de ces ondes de plasma appelée "Onde de Langmuir". De telles ondes sont créées lorsqu'on écarte localement le plasma de la neutralité de charge. On cherche donc à savoir comment cet écart à la neutralité va se propager dans le plasma.


Ex: Onde de Langmuir

Auteur: Arnaud Beck

exerciceRelation de dispersion

Difficulté : ☆☆   Temps : 1h

Un plasma est constitué d'ions et d'électrons. Les ions étant largement plus lourds, nous allons les supposer immobiles dans le développement qui suit. Considérons qu'ils sont répartis uniformément dans l'espace avec une densité n_0.

L'onde de Langmuir étant la propagation d'une perturbation électrostatique (écart à la neutralité mais sans création de courant électrique à grande échelle), nous pouvons, pour simplifier le problème, supposer l'absence de champ magnétique.

A l'équilibre, les électrons sont eux aussi immobiles et uniformément répartis avec une densité n_e=n_0. Mais, que se passe t-il si on perturbe cet équilibre en posant que n_e=n_0+n_1(x,t), où n_1 est un petit terme perturbatif qui dépend de la position x et du temps t ?

Dans ce cas, un champ électrique E(x,t) se crée et met les électrons en mouvement à une vitesse u(x,t).

Les équations qui gouvernent ensuite l'évolution de ces trois grandeurs (perturbation de densité, champ électrique et vitesse des électrons) sont l'équation de continuité, l'équation de conservation du moment dynamique et l'équation de Poisson:

\frac{\partial n_1}{\partial t}+n_0\frac{\partial u}{\partial x}=0

m_en_0\frac{\partial u}{\partial t}=-en_0E-3T\frac{\partial n_1}{\partial x}

\frac{\partial E}{\partial x}=-en_1/\epsilon_0

T est la température moyenne des électrons, m_e leur masse, -e leur charge, et epsilon_0 la permittivité du vide.

Les équations ont été ici écrites à une dimension, dans la direction x. On suppose que les perturbations vont se propager dans cette direction sous la forme d'onde plane et donc que l'on peut écrire:

E(x,t)=\tilde{E}exp(-i\omega t +ikx)

u(x,t)=\tilde{u}exp(-i\omega t +ikx)

n_1(x,t)=\tilde{n_1}exp(-i\omega t +ikx)

omega est la pulsation de l'onde et k l'amplitude de son vecteur d'onde selon x.

Question 1)

Écrire le système linéaire vérifié par les inconnues \tilde{E}, \tilde{u} et \tilde{n_1} et ayant omega et k comme paramètres.

Question 2)

Trouver la relation de dispersion de l'onde, c'est à dire une expression de omega en fonction de k.

Question 3)

Si on prend le mouvement des ions en compte, le système d'équation change et on trouve une nouvelle relation de dispersion qui correspond cette fois à une onde acoustique ionique.

En utilisant la même méthode que précédemment, retrouver la fonction de dispersion d'une onde acoustique ionique à partir du système d'équations ci dessous. Les indices e et i indiquent l'espèce (électron ou ion).

\frac{\partial n_{e1}}{\partial t}+n_0\frac{\partial u_e}{\partial x}=0

\frac{\partial n_{i1}}{\partial t}+n_0\frac{\partial u_i}{\partial x}=0

m_en_0\frac{\partial u_e}{\partial t}=-en_0E-\gamma_e T_e\frac{\partial n_{e1}}{\partial x}

m_in_0\frac{\partial u_i}{\partial t}=en_0E-\gamma_i T_i\frac{\partial n_{i1}}{\partial x}

\frac{\partial E}{\partial x}=e(n_{i1}-n_{e1}/)\epsilon_0

où les gamma sont des constantes (rapports des chaleurs spécifiques de chaque espèce).


Filtrage de Wiener

Auteur: Jérôme Thiébaut

En astrophysique, les photos de galaxies sont prises par des caméras CCD fixées derrière un télescope. L'instrument d'observation, ici le télescope, laisse son empreinte sur l'image. A cela s'ajoute le bruit de mesure c'est à dire un signal autre que l'image elle même qui s'ajoute à celle-ci. Ce bruit est dû à la caméra.... On se propose dans cet exercice de montrer comment retrouver l'image la plus proche de l'image initiale, c'est à dire de déconvoluer et de filtrer l'image reçue afin de s'affranchir au maximum des effets de l'instrument d'observation et du bruit.


Ex: Filtrage de Wiener

Auteur: Jérôme Thiébaut

exerciceFiltrage de Wiener

Difficulté : ☆☆   Temps : 40mn

Question 1)

L'image reçue par le CCD est une collection de pixels que l'on rassemble sous la forme d'un vecteur Y. Ce vecteur resulte de l'image initiale, X, qui a été convoluée par le télescope auquel s'ajoute un vecteur bruit noté N. La convolution se modélise par l'application d'une matrice A sur le vecteur X. Ainsi on a: Y=AX+N. Dans l'espace de Fourier, cette relation s'ecrit: Y_k=A_k*X_k +N_kk=((k_x* k_y)) représente la fréquence spatiale en deux dimensions. Dans cet espace, la matrice A_k est diagonale de valeur propre lambda_k. Le spectre de puissance de l'image suit souvent une loi de puissance, c'est à dire <X_k^2> =C_0* k^(-alpha)et le bruit est souvent un bruit blanc c'est à dire qu'il à la même intensité quelquesoit la fréquence spatiale, <N_k^2> =C, où C et C_0sont des constantes. Montrer que l'inversion simple de cette relation (qui consiste à appliquer la matrice A^(-1)sur les données Y afin de retrouver X) conduit, au delà d'une certaine fréquence, à une amplification du bruit.

Galaxie spirale M100
im1.jpg
Image convoluée bruitée
im2.jpg

Question 2)

On cherche donc maintenant à déconvoluer l'image mais aussi à filtrer le bruit. Pour cela, on va chercher le filtre R à appliquer sur les données Y qui va minimiser l'écart quadratique moyen entre la vraie image X et l'image filtrée accent(X;~)=R*Y. On cherche donc à minimiser la quantité S=<(accent(X;~)-X)^2> par rapport à R. En postulant que le bruit et le signal sont décorrélés et que le bruit est non biaisé (pas d'erreur systématique), montrer que R=(A^T*C_N^(-1)*A+C_X^(-1))^(-1)*A^T*C_N^(-1) ,où C_X=<X^T*X> et C_N=<N^T*N>sont les matrices de variance-covariance du signal et du bruit.

Question 3)

Dans l'espace de Fourier, les matrices de variance-covariance sont diagonales également et se réduisent aux spectres de puissances. Montrer que le filtre de Wiener R_k inverse les basses fréquences et coupe les plus grandes où le bruit domine.

Image déconvoluée filtrée
im4.jpg


Groupe quotient


Phénomènes mutuels

Auteur: Stéphane Erard

L'arithmétique intervient en Astronomie lorsqu'il est question de phénomènes périodiques. Historiquement, la prévision des éclipses et des fêtes religieuses a fait appel à de tels calculs. Dans la période moderne, c'est la mécanique quantique (à travers l'équation de Schrödinger) qui introduit des solutions à base de nombres entiers.


Ex: Phénomènes mutuels

Auteur: Stéphane Erard

exercicePhénomènes mutuels

Difficulté : ☆☆   Temps : 45 min

Le corps céleste A a une période synodique (par rapport à la Terre) de 105 jours et passe à l'opposition à la date J_0. Six jours plus tard on observe à l'opposition le corps B dont la période synodique est de 81 jours.

On veut déterminer la date J_1 de la prochaine opposition simultanée des deux corps.

Question 1)

Trouver une condition permettant de déterminer cette date.

Question 2)

Trouver une solution particulière de cette équation.

Question 3)

Déterminer toutes les solutions de l'équation trouvée plus haut.

Question 4)

Quelle est la date de la prochaine opposition commune ?

Question 5)

Application à Mars et Jupiter : une opposition de Mars a eu lieu le 24/12/2007, l'opposition suivante de Jupiter le 4/7/2008. Les périodes synodiques respectives sont de 780 et 399 jours. Quand se produira la prochaine opposition simultanée des deux planètes ?


Atome de Bohr

Auteur: Stéphane Erard

Les premières mesures spectroscopiques ont révélé à la fin du XIXe siècle un comportement inattendu des sources lumineuses : elles présentent fréquemment des raies intenses, soit en absorption soit en émission. Pour une source donnée, l'émission ou l'absorption ne se produisent qu'à certaines longueurs d'onde. La formule expérimentale de Balmer-Rydberg (1885-88) rend compte de la position de ces raies pour l'atome d'hydrogène, mais ne correspond à aucun phénomène connu.

Divers modèles de structure atomique ont été proposés dans les années suivantes pour intégrer les résultats expérimentaux de l'époque. Le modèle de Bohr pour l'atome d'hydrogène (1913) a fourni la première explication des résultats spectroscopiques. Il implique un comportement non-classique des systèmes microscopiques, qui sautent sans transition entre états d'énergie discrets.


Ex: Atome de Bohr

Auteur: Stéphane Erard

exerciceModèle de Rutherford

Difficulté :    Temps : 30 min

Un des premiers modèles atomiques modernes est celui de Rutherford (1911), s'appuyant sur des expériences de diffusion de particules alpha. Ce modèle suppose que l'atome est formé d'un noyau de très petites dimensions chargé positivement, autour duquel gravitent des électrons négatifs beaucoup moins massifs sur des orbites circulaires. En raison d'une analogie évidente, on l'appelle modèle planétaire.

Question 1)

On considère un atome d'hydrogène où un électron unique orbite autour d'un noyau de charge unité. L'électron est soumis à une force électrostatique d'intensité F_c = \frac{1}{4\pi\epsilon_0} \frac{e^2}{r^2}

où e est la charge de l'électron et du noyau (opposées), r leur distance et \epsilon_0 une constante physique (permittivité du vide).

Ecrire la distance électron-noyau dans ce modèle.

Question 2)

Calculer l'énergie totale (cinétique et potentielle).

Auteur: Stéphane Erard

exerciceAtome de Bohr

Difficulté :    Temps : 45 min

Une difficulté avec le modèle de Rutherford est qu'il ne rend pas compte des expériences de spectroscopie de l'époque et de l'existence de raies spectrales. Par ailleurs, l'électrodynamique classique prévoit que les électrons devraient rayonner et perdre de l'énergie, ce qui les ferait tomber sur le noyau très rapidement. Niels Borh travaillait à ce problème quand il prit connaissance de la formule de Balmer qui donne la position observée des raies spectrales de l'hydrogène dans le visible :

1/ \lambda = \nu / c = R\left(\frac{1}{2^2} - \frac {1}{n^2} \right)

\nu est la fréquence associée, n est un nombre entier > 2, R une constante et c la vitesse de la lumière.

Par ailleurs il connaissait l'hypothèse d'Einstein formulée pour l'étude de l'effet photo-électrique : la lumière peut se décomposer en "quanta" (les photons) dont l'énergie est liée à la fréquence \nu du rayonnement : E = h \nu.

En rapprochant ces faits, Bohr formula l'hypothèse que l'atome ne peut prendre que certains états d'énergie donnés dans son modèle atomique (1913).

Question 1)

Calculer les longueurs d'onde des raies visibles et dessiner le spectre de l'hydrogène à l'aide de la formule ci-dessus. On prendra les raies de Balmer n = 3 à 6 qui sont dans le domaine visible, et R = 1.1\;10^7\; m^{-1} (constante de Rydberg).

Question 2)

Ecrire les variations d'énergie de l'atome d'hydrogène liées à l'émission d'une raie de la série de Balmer.

Question 3)

En déduire les valeurs possibles du rayon de l'électron et du moment cinétique mrv.

Question 4)

Comment interpréter ce résultat ?

remarqueremarque

Les autres séries de raies de l'hydrogène correspondent à des transitions vers les couches n ≠ 2. On peut représenter les niveaux énergétiques de l'hydrogène de la façon suivante :

Diagramme énergétique de l'hydrogène
bohr.png
Crédit : Astrophysique sur Mesure / FSU

La première raie de Balmer H_{\alpha} est particulièrement importante en Astronomie car elle permet de détecter l'hydrogène atomique dans le milieu interstellaire.


Cycles astronomiques et fractions continues

Auteur: Alexandre Pousse

Introduction

Les fractions continues ont une très longue histoire car liées à celle des nombres. En effet, il existe un lien important entre celles-ci et l'algorithme d'Euclide. Plus particulièrement, elles apparaissent dans l'approximation de nombre comme π ou du nombre d'or.

Délaissées pendant un certain temps, elles sont redécouvertes en Europe en 1655 par le mathématicien anglais John Wallis, puis étudiées par la suite par Leonhard Euler qui va apporter de nombreux théorèmes.

L'interêt de l'étude des fractions continues est souvent pour l'approximation d'équations diophantiennes. Ce sont des équations algébriques pour lesquelles on cherche des solutions en entiers. Un exemple particulier qui est utile en astronomie car permettant de mettre en évidence des phénomènes de résonnances ou de prévoir le retour d'un phénomène périodique, c'est de fixer X,Y deux nombres représentant des périodes, et de trouver \lambda, \mu , deux entiers tels que \lambda X + \mu Y = 0 . La notion d'approximation introduite par les fractions continues est utilisée lorsque X, Ysont irrationnelles ou rationnelles comportant de nombreuses décimales (ce qui est fréquent de manière générale en Physique), on va alors chercher à trouver la meilleure combinaison linéaire approximant \lambda X + \mu Y. Autre application des fractions conitnues: en arithmétique, elles vont permettre l'étude et la caractérisation de nombres transcendants (par exemple, par l'étude de leur périodicité).

Définitions et propriétés

Une fraction continue est un objet s'écrivant sous la forme a_0 + \frac{b_0}{a_1 + \frac{b_1}{a_2 + \frac{b_2}{a_3 + \frac{ b_3}{a_4 + ...}}}} où les a_net les b_n sont des nombres entiers naturels ou relatifs. La fraction obtenue peut être composée d'un nombre fini ou infini de termes.

Mais ce que nous utiliserons par la suite et qui ont été étudiées plus particulièrement, ce sont les fractions continues simples, c'est-à-dire de la forme a_0 + \frac{1}{a_1 + \frac{1}{a_2 + \frac{1}{a_3 + \frac{ 1}{a_4 + ...}}}} avec a_0 \in \mathbb{Z} et (a_n)_{n>0} \subset \mathbb{N} (fini ou non). Une notation plus compacte et qui sera utilisée ici est d'écrire [a_0;a_1;a_2;...;a_n;...].

Afin de caractériser une fraction continue, on utilise la notion de réduite. Par exemple, pour n \in \mathbb{N}, on appellera réduite de la fraction continue définie par la suite (a_n)_{n\geq0} \subset \mathbb{N}, la fraction \frac{p_n}{q_n}=[a_0;a_1;a_2;a_3;...;a_n]. Pour le nombre d'or \frac{1 + \sqrt{5}}{2} , les trois premières réduites sont \frac{p_0}{q_0}= [1], \frac{p_1}{q_1}=[1;1], \frac{p_2}{q_2}=[1;1;1]. Ainsi, nous obtenons deux suites d'entiers (p_n)_{n\geq 0} et (q_n)_{n\geq 0} avec en particulier, la propriété suivante: si p_{-1}=1 , p_{0}=a_0 et \forall n \geq 1~~p_n = a_np_{n-1} + p_{n-2} , et si q_{-1}=0 , q_{0}=a_0 et \forall n \geq 1~~q_n = a_nq_{n-1} + q_{n-2}, alors \forall n \geq 0~~ \frac{p_n}{q_n} = [a_0;a_1;a_2;a_3;...a_n].

Approximation des nombres

Introduisons maintenant la fraction continue dans le cadre de l'approximation des nombres. Soit α un réel et (\alpha_n)_{n\in\mathbb{N}} une suite de réels telle que:

\alpha_0 = \alpha,

si \alpha_n \notin \mathbb{N}^* alors \alpha_n = a_n + \frac{1}{\alpha_{n+1}}~~\mbox{avec}~~ \alpha_{n+1} >1 ~~\mbox{et}~~ a_n \in \mathbb{N}^*,

sinon \alpha_n = a_n ~~\mbox{avec}~~ a_n \in \mathbb{N}^*.

Ainsi, on obtient le développement suivant \alpha = a_0 + \frac{1}{\alpha_1}= a_0 + \frac{1}{a_1 + \frac{1}{\alpha_2}}} = a_0 + \frac{1}{a_1 + \frac{1}{a_2+ \frac{1}{\alpha_3}}}}=...=[a_0;a_1;a_2;...;a_n+ \frac{1}{\alpha_{n+1}}]=...

Application à l'astronomie

L'intérêt des fractions continues dans le domaine de l'astronomie est lié à la notion de périodicité ou de résonnance et donc aux équations diophantiennes qui en résultent. En effet, si l'on considère deux phénomènes ayant chacun une période T_1 et T_2 , alors afin de caractériser le retour mutuel de ces deux phénomènes, il est commode de chercher deux entiers X et Y tels que XT_1 + YT_2 = 0. Or généralement, les périodes ne sont malheureusement pas des nombres entiers ce qui implique de grands nombres entiers pour X et Y.

L'idée est donc de chercher les "meilleurs" rationnels \frac{X}{Y} approchant \frac{T_2}{T_1} de façon à résoudre le problème au voisinage de la résonnance. C'est ce qu'on appelle l'approximation diophantienne. Nous utiliserons pour cela les fractions continues.

Les exercices qui suivent vont ainsi permettre de mettre en évidence la propriété théorique sur les réduites ainsi que des applications astronomique par la recherche de meilleure solution approximation de l'équation diophantienne via les fractions continues en caractérisant le mouvement de Saturne et de la Terre, le phénomène d'éclipse et en définissant une meilleure approximation de l'année tropique.


Ex: Cycles astronomiques et fractions continues

Auteur: Alexandre Pousse

exercicePropriété des réduites

Difficulté :    Temps : 10 min

Soient (p_n)_{n\geq 0} et (q_n)_{n\geq 0}, deux suites d'entiers. Rappelons la propriété sur les réduites donnée dans le cours:

si p_{-1}=1 , p_{0}=a_0 et \forall n \geq 1~~p_n = a_np_{n-1} + p_{n-2} ,

et si q_{-1}=0 , q_{0}=a_0 et \forall n \geq 1~~q_n = a_nq_{n-1} + q_{n-2},

alors \forall n \geq 0~~ \frac{p_n}{q_n} = [a_0;a_1;a_2;a_3;...a_n].

Question 1)

Démontrer la propriété des réduites.

Auteur: Alexandre Pousse

exerciceUne meilleure approximation de l'année tropique

Difficulté :    Temps : 30 min

Une année tropique correspond au temps s'écoulant entre deux équinoxes de printemps, c'est-à-dire 365.24219052 jours (année tropique moyenne à J2000). C'est donc l'année permettant "le retour des saisons" au mêmes dates et donc compensant le phénomène de précession des équinoxes.

En effet, avant la réforme du calendrier par Grégoire XIII au XVIe siècle, le calendrier était le calendrier Julien, établi par l'astronome Sosigène d'Alexandrie et comportant 365.25 jours (année bissextile tous les quatre ans). Cela impliquait un décalage d'un jour tous les 128 ans, d'où modification de la date de retour des saisons.

L'idée de cet exercice est de comprendre le calendrier utilisé aujourd'hui, puis de trouver par l'intermédiaire d'une fraction continue une valeur plus stable de l'année.

Question 1)

L'année grégorienne correspond à 366 jours les années multiples de quatre et non multiples de cent sauf les année multiples de quatre cents. Sinon, l'année vaut 365 jours.

Établir la valeur et la fraction représentant la partie décimale de l'année grégorienne.

Question 2)

On définira la notion de stabilité comme l'écart la durée de l'année estimée et la durée de l'année tropique moyenne. Le réel obtenu permet de déduire le décalage du retour des équinoxes.

Évaluer la stabilité du calendrier grégorien. Au bout de combien de temps le calendrier se décale d'un jour?

Question 3)

En utilisant la méthode d'approximation des nombres à l'aide d'une fraction continue, trouver une nouvelle définition de l'année beaucoup plus stable que l'année grégorienne. Proposer une méthode d'application pour remplacer le calendrier actuel.

Auteur: Alexandre Pousse

exerciceL'automate de Huygens

Difficulté :    Temps : 30 min

Christian Huygens, mathématicien et astronome du XVIIe siècle, souhaitait réaliser un automate planétaire permettant de modéliser l'évolution du système solaire au cours du temps (en approximation circulaire). À cet époque, le système solaire ne comprend que 6 planètes (Mercure, Venus, Terre, Mars, Jupiter et Saturne). Rappelons qu'un automate est un système composé d'une manivelle reliée à différents rouages, chacun associé à la période de révolution d'une planète par leur nombre de dents.

Lors de la conception de cet objet, Huygens se retrouve confronté à une difficulté: le rapport de l'année terrestre et de celle de Saturne. Combien faut-il de dents sur les deux engrenages pour décrire convenablement le mouvement de la Terre et de Saturne au cours de leur révolution?

Question 1)

Dans l'approximation d'orbites circulaires, poser l'équation diophantienne du problème de l'automate.

Question 2)

Sur son orbite, la Terre parcourt un angle \mu = 359\deg45'40''31''' en un an. De même en un an, Saturne réalise \lambda = 12\deg 13'34''18''' (Ce sont les valeurs de l'époque).

Établir la fraction rationnelle donnée par le rapport \frac{\mu}{\lambda}. Est-il raisonnable de réaliser deux engrenages associés à cette fraction?

Question 3)

Maintenant, afin de supprimer ce problème technique, introduire la notion de fraction continue pour résoudre le problème par approximation diophantienne.

Question 4)

Huygens définit la notion de stabilité comme le décalage entre l'angle parcouru par Saturne sur son automate et dans la réalité après que la Terre ait réalisé 100 révolutions.

À l'aide d'un développement en fraction continue, proposer un engrenage satisfaisant d'un point de vue technique (au delà d'un millier de dents, la réalisation est difficile) et stable au sens de Huygens.

Auteur: Alexandre Pousse

exerciceLe cycle de Saros

Difficulté :    Temps : 60 min

Un cycle de Saros correspond à 223 lunaisons. C'est une période associée au retour d'une éclipse de Soleil (resp. de Lune) après une éclipse totale. Ainsi, si une éclipse a lieu à un instant t alors il est possible de prédire qu'au temps t+223 lunaisons il s'en reproduira une autre.

L'idée de cet exercice est de comprendre et de retrouver pourquoi nous avons ce nombre de 223 lunaisons pour le retour d'une éclipse.

Question 1)

Définir géométriquement la notion d'éclipse de Lune (resp. de Soleil) vu de la Terre (avec la notion de droite ou de plan par exemple).

Question 2)

Caractériser la notion d'éclipse en terme de position de la Lune sur son orbite ainsi que de son éclairement relatif à la Terre.

Question 3)

Introduisons deux notions pour la détermination de cycle de Saros.

Le mois draconitique, c'est le temps que met la Lune à partir du noeud ascendant pour y revenir. La durée du mois draconitique est de \mu = 27.212221~\mbox{j}.

Le mois synodique ou lunaison est le temps entre deux nouvelles Lunes successives. Sa durée est d'en moyenne \lambda = 29.2953089~\mbox{j}.

Dans l'approximation d'orbites circulaires, poser l'équation diophantienne du problème du retour d'éclipse.

Question 4)

Introduire la notion de fraction continue pour résoudre le problème par approximation diophantienne.

Question 5)

Rappelons que le diamètre de la Lune et du Soleil vu de la Terre est de 30' d'arc.

Établir l'erreur de coincidence maximal pour que l'on ait une éclipse (on considère qu'une éclipse partielle est encore une éclipse).

Question 6)

Développer la fraction continue jusqu'au terme adéquat (évaluation des réduites et contrôle de l'erreur de coïncidence).

Question 7)

Conclure sur la notion de cycle de Saros.

Question 8)

Vous vous rappelez peut-être de l'éclipse totale de Soleil du 11 août 1999 (éclipse totale de la Normandie à l'Alsace en France et partielle au voisinage de cette bande). Déterminer quand cette configuration va t-elle se reproduire? Va t-elle avoir lieu aux mêmes longitudes?


Statistiques et probabilités


Introduction

Les statistiques sont d'un usage fondamental pour la mesure et le traitement du signal en Astronomie. Cette section illustre quelques notions de base :


Probabilités

Auteur: Stéphane Erard

Loi de Poisson

De nombreux processus physiques correspondent à une situation où l'on compte des événements aléatoires indépendants. C'est par exemple le cas des désintégrations radioactives ou de l'émission de lumière par une source, de façon générale toute situation où l'événement se produit avec une probabilité constante par unité de temps. On veut connaître précisément la loi du phénomène pour calculer les fluctuations associées et les minimiser.

La loi de Poisson est ainsi utilisée pour rendre compte de phénomènes aléatoires qui vérifient les deux propriétés suivantes :

Un événement se produisant en moyenne avec une fréquence \lambda, on note P(k) la densité de probabilité pour qu'il se produise k fois durant le temps t (avec k ≥ 0). La loi de Poisson de paramètre \lambda t donne :

P_{t,\lambda}(k)=  \frac{(\lambda t)^k}{k!} e^{-\lambda t}

où k! est la factorielle.

L'appliquette ci-dessous illustre la loi de Poisson : elle trace les valeurs obtenues au cours d'un certain nombre de tirages, pour une valeur de \mu = \lambda t donnée.

Loi de Poisson application.png


Ex: loi de Poisson

exerciceDérivation de la loi de Poisson

Difficulté : ☆☆   Temps : 40 min

Question 1)

Retrouver la forme générale de la loi de Poisson à partir d'un raisonnement discret.

Question 2)

Déterminer la moyenne et l'écart-type de la loi de Poisson.

Question 3)

On observe une source lumineuse faible pendant un temps t. Quel est le nombre de photons détectés en moyenne, de quoi dépend-il ? Que représente l'écart-type de cette distribution ? Quel paramètre permet de quantifier la précision de la mesure, et comment améliorer la mesure de cette source ?

remarqueRemarque

L'applet ci-dessous illustre ce dernier résultat : on améliore le rapport signal sur bruit en posant plus longtemps, mais cette amélioration est lente. Elle est spectaculaire au début (en permettant la reconnaissance de l'objet), mais ralentit de plus en plus (les détails sont de plus en plus longs à se préciser).

Loi de Poisson application.png


Loi normale

La loi de distribution gaussienne est sans doute la plus employée, en physique comme ailleurs, à tel point qu'on l'appelle généralement loi normale.

La densité de probabilité gaussienne est :

P(x) = \frac{1}{\sigma \sqrt{2 \pi}}e^{-\frac{(x-\mu)^2}{2 \sigma ^2}}

Dans cette formulation, \mu représente la moyenne, et \sigma l'écart-type. Le coefficient numérique sert à normaliser l'intégrale à 1.

On remarque la symétrie de la fonction autour du pic central, et son aspect caractéristique ("en cloche"). On note habituellement N(\mu , \sigma) la loi normale. N(0,1) est appelée loi normale centrée-réduite (moyenne nulle, variance normalisée à 1).

L'importance de la loi normale est liée au théorème de la limite centrale, qui montre que la superposition de lois de distribution différentes tend vers une loi normale. Ceci est en particulier important pour estimer les erreurs de mesure : si elles sont de provenance différentes, et de statistique mal connue, on peut généralement faire l'approximation que leur somme est distribuée de manière gaussienne. C'est le théorème de la limite centrale qui explique l'omniprésence de la loi normale.

gaussiennes
exgauss.png
Exemples de gaussiennes de moyenne et largeur à mi-hauteur variables. La surface sous la courbe est normalisée à 1.
Crédit : Astrophysique sur Mesure / Erard

Ex: Loi normale

exerciceLimite de détection d'un objet ponctuel

Difficulté :    Temps : 20 min

remarqueRemarque

Le calcul des moments de la loi normale est donné ici en exercice.

Question 1)

Calculer la largeur à mi-hauteur de la gaussienne

Question 2)

Déterminer graphiquement l'aire délimitée par une gaussienne entre ± 1, 2 et 3 écart-types de la valeur moyenne. Quel est l'intérêt de cette question ?

Question 3)

On observe un astéroïde de la ceinture principale avec un télescope de 2 m depuis le sol. Le fond de ciel produit un signal de 100 pas-codeurs à la sortie de la caméra. A quel niveau de signal peut-on penser avoir détecté l'objet ?

exerciceLimite de la loi de Poisson

remarqueRemarque

On peut montrer en utilisant la formule de Stirling que pour les grandes valeurs de l'argument, la loi de Poisson tend vers une loi normale. La loi de Poisson décrit correctement les situations où l'intervalle de valeurs possibles est borné d'un côté, donc pour les petits nombres. Dans les autres cas elle se confond pratiquement avec une loi normale (dès que n > 30 et \mu > 5).

remarqueRemarque

L'applet ci-dessous illustre ce résultat : la distribution de Poisson est comparée à la loi normale correspondante (appuyer sur le bouton "moyenne").

Loi de Poisson application.png


Chemin optique d'un photon dans le Soleil

On se propose d'estimer le temps nécessaire pour qu'un photon produit dans le Soleil soit émis à sa surface.

Le processus correspond à une suite d'émissions et absorptions successives par les atomes rencontrés en chemin. On l'assimile à une marche au hasard en trois dimensions (bien que l'identité du photon ne soit pas conservée au cours de ces événements, et que la longueur d'onde puisse changer au cours des diffusions successives).


ex: Chemin optique d'un photon dans le Soleil

Auteur: Stéphane Erard

exerciceChemin optique d'un photon dans le Soleil

Difficulté :    Temps : 45 min

Question 1)

On considère une particule se déplaçant en trois dimensions de façon aléatoire par pas de longueur d. Ecrire sa position en coordonnées sphériques après le premier pas. La distance d est appelée libre parcours moyen.

Question 2)

Ecrire les coordonnées après N pas en fonction des directions successives. Quelle est la distance au point de départ ?

Question 3)

Simplifier cette expression, sachant que les directions successives sont aléatoires et indépendantes.

Question 4)

On donne la densité \rho = 1410\; kg/m^3 et le coefficient d'absorption massique \kappa = 10\; m^2 /kg, qu'on suppose uniformes au premier ordre. Calculer le libre parcours moyen.

Question 5)

Connaissant le rayon du Soleil r_\odot \approx 7.10^8 m, combien de diffusions un photon produit au centre du Soleil subit-il en moyenne avant d'arriver en surface ? Quelle est la longueur du trajet réellement parcouru dans le Soleil ? Combien de temps faut-il au photon pour effectuer le trajet ?


Statistiques élémentaires

Auteur: Stéphane Erard

Moyennes et estimations

Mesurer une quantité physique, c'est faire une estimation de la valeur de cette quantité. Cette estimation peut être entachée de deux types d'erreurs : erreur systématique, et erreur aléatoire. La première est liée à l'instrument de mesure ou au type d'observation (si on mesure plusieurs phénomènes simultanément sans s'en rendre compte), elle peut être additive (niveau de base ou offset, par exemple le courant d'obscurité d'une caméra) ou multiplicative (réglage de gain défectueux). Les erreurs aléatoires sont liées au processus de mesure lui-même (bruit de lecture) ou à la nature du phénomène mesuré (bruit de photon d'une source lumineuse faible, lié au processus d'émission). Une bonne mesure est telle que les erreurs aléatoires sont minimisées, et que les erreurs systématiques sont beaucoup plus petites que celles-ci.

L'exercice consiste à dériver la meilleure estimation de l'éclairement d'un corps céleste, dans l'hypothèse où l'erreur systématique est faible.


Ex: moyennes et estimations

Auteur: Stéphane Erard

exerciceEstimation d'une moyenne

Difficulté :    Temps : 30 min

On mesure l'éclairement d'une étoile. La valeur "réelle" est notée \mu (celle que mesurerait un instrument parfait).

Pour obtenir une bonne estimation de cette quantité, une méthode usuelle est de pratiquer N mesures successives X_i. On s'attend à ce que celles-ci se répartissent de façon gaussienne autour de la valeur \mu (théorème de la limite centrale).

Question 1)

A partir de ces N estimations indépendantes de l'éclairement, dériver le résultat de la mesure : valeur estimée de la moyenne \mu, et incertitude sur cette estimation.

Question 2)

Plusieurs équipes ayant publié leurs résultats (maintenant notés X_i), on veut en tirer la meilleure évaluation possible. Ce problème est équivalent à celui de mesures successives entachées d'incertitudes indépendantes \sigma_i.

Application numérique : on a deux mesures indépendantes 100±5 et 94±20. Quelle est l'estimation résultante ?


Propagation d'erreurs

On s'intéresse souvent à une fonction des quantités mesurées. Le problème est alors de dériver l'incertitude associée à cette estimation.

C'est par exemple le cas en spectroscopie, où on mesure une intensité I à diverses longueurs d'onde \lambda (variable indépendante). Les quantités physiquement importantes sont liées aux variations spectrales ; elles s'expriment comme des différences ou des rapports d'intensités à différentes longueurs d'onde, ou comme des fonctions plus complexes de ces intensités. Une incertitude individuelle est associée à chaque mesure spectrale I(\lambda). En principe, les incertitudes sur les mesures individuelles sont indépendantes les unes des autres. Si ce n'est pas le cas, il y a un problème avec l'étalonnage de l'instrument et il faut ajouter un terme de covariance dans les formules ci-dessous (on parle improprement de « bruit corrélé », parce qu'un signal inconnu se superpose à celui qu'on mesure).


Ex: propagation d'erreurs

exerciceErreurs sur une fonction

Difficulté : ☆☆   Temps : 30 min

introductionIntroduction

L'exercice consiste à dériver la précision des estimations de différentes fonctions spectrales.

Question 1)

La fonction est une combinaison additive de variables : f = ax + y + b

où a et b sont des constantes, et x et y sont affectées des incertitudes \sigma_x et \sigma_y.

Quelle est l'incertitude associée \sigma_f ?

Question 2)

On utilise maintenant une fonction multiplicative :

f = cx^a . y^b où a, b et c sont des constantes.

Quelle est l'incertitude associée \sigma_f ?

Question 3)

Pour les cas plus compliqués où l'on connaît la forme analytique de la fonction, écrire la formulation générale.

Question 4)

On applique ces résultats à la situation suivante :

On mesure les intensités a, b et c aux longueurs d'onde \lambda_1, \lambda_2 et \lambda_3 situées autour d'une bande d'absorption, avec les incertitudes \sigma_1, \sigma_2 et \sigma_3.

spec.png
Exemple de spectre infrarouge et définition des paramètres mesurés
Crédit : Astrophysique sur Mesure / Erard

On estime le continuum (pente spectrale) autour de la bande d'absorption comme Ct = \frac{(a+b)}2

et la profondeur de cette bande comme Pr = \frac c{\sqrt{ab}}

Écrire les incertitudes sur ces quantités en fonction de celles des mesures.

Question 5)

On utilise maintenant la mesure d'éclairement a pour estimer la magnitude à la longueur d'onde \lambda_1.

Ecrire cette magnitude en fonction de a et d'une constante d'échelle.

Question 6)

Ecrire l'incertitude sur cette magnitude en fonction de celle sur l'éclairement.


Distribution gaussienne

Auteur: Jérôme Thiébaut

Moindres carrés

Auteur: Jérôme Thiébaut

On a très souvent accès en astrophysique à des données d (par exemple des spectres lumineux, des images CCD...) dont on veut extraire une quantité physique inconnue X (magnitude, masse, champ magnétique...). Si le problème est linéaire et si on connait les lois physiques sous jacentes, on peut modéliser le problème sous la forme accent(d;->)=A*accent(X;->), où accent(d;->) est un vecteur contenant les données, accent(X;->) est le vecteur des paramètres recherchés et A est une matrice. Connaissant accent(d;->) on veut accent(X;->) , c'est ce qu'on appelle un problème inverse. Pour résoudre ce genre de problème on utilise très souvent la méthode des moindres carrés c'est à dire qu'on cherche le vecteur accent(X_0;->) qui minimise la quantité |A*accent(X;->) - accent(d;->)*|^2.C'est ce qu'on appelle un ajustement. On se propose ici de montrer que faire des à priori gaussiens sur les distributions de probabilité de accent(X;->) et accent(d;->) revient à utiliser la méthode des moindres carrés.


Ex: Moindres carrés

Auteur: Jérôme Thiébaut

exerciceMoindres carrés

Difficulté : ☆☆   Temps : 20 min

On a le problème suivant accent(d;->)=A*accent(X;->) et on va chercher à déterminer la probabilité conditionnelle P(accent(X;->)*|*accent(d;->)).

.

Question 1)

Exprimer cette probabilité en fonction de P(accent(X;->)), P(accent(d;->)) et P(accent(d;->)*|*accent(X;->)) en utilisant le théorème de Bayes.

Question 2)

On suppose que P(accent(d;->)*|*accent(X;->)) suit une loi gaussienne multivariée de matrice de variance covariance C_d. Ecrire cette loi.

Question 3)

On suppose que le vecteur accent(X;->) suit lui aussi une loi gaussienne de matrice de variance covariance C_p et de moyenne accent(X_p;->) (a priori sur la solution, sur son spectre de puissance...) et que P(accent(d;->))=1 . Exprimer P(accent(X;->)*|*accent(d;->)).

Question 4)

Montrer que chercher les paramètres physiques les plus probables revient à résoudre la méthode des moindres carrés généralisés soit à trouver accent(X_0;->)=argmin_(accent(X;->))*(((accent(d;->)-A*accent(X;->))^T*C_d^(-1)*(accent(d;->)-A*accent(X;->))+(accent(X;->)-accent(X_p;->))^T*C_p^(-1)*(accent(X;->)-accent(X_p;->)))).

Question 5)

Dans les cas simples, on n'a pas d'a priori sur accent(X;->) et on considère que les covariances sont nulles et les variances égales entre elles donc C_d=sigma*II est la matrice identité. Que devient accent(X_0;->) ?


Réponses aux exercices

pages_approxim/exo-orb-mars.html

Exercice 'Orbite de Mars'


pages_approxim/exo-precession-peri-approxim.html

Exercice 'Précession du périastre'


pages_dim-unites/dim4.html

Exercice 'Ex: analyse dimensionnelle'


pages_develop-appli/exo-series-2corps.html

Exercice ' Séries du problème des 2-corps'


pages_develop-appli/exo-series-2corps-lagrange.html

Exercice ' Séries du problème des 2-corps (Lagrange)'


pages_develop-appli/exo-epicycle-kepler.html

Exercice 'Description épicyclique du mouvement keplerien'


pages_develop-appli/exo-sphere-influence.html

Exercice 'Sphère d'influence'


pages_suites-reelles/exo-keplerxnewton.html

Exercice 'équation de Kepler elliptique'


pages_continuite/exo-loi-de-wien-continuite.html

Exercice 'Loi de Wien'


pages_def/exo-retrogradation-mars.html

Exercice 'Rétrogradation de Mars'


pages_def/exo-perturb-orbit.html

Exercice 'Orbites perturbées du problème à 2 corps'


pages_def/exo-planck-fq.html

Exercice 'Loi de Planck en fréquence'


pages_af/exo-racines-laplace.html

Exercice 'Les racines du polynôme de la méthode de Laplace'


pages_af/exo-poly-legendre-racines.html

Exercice 'Les racines des polynômes de Legendre'


pages_af/exo-proj-mollweide-af.html

Exercice 'projection de Mollweide'


pages_extremas/exo-loi-de-wien-extremas.html

Exercice 'Loi de Wien'


pages_extremas/ex-neutron-star.html

Exercice 'Etoile à neutron'


pages_extremas/ex-sph-phot.html

Exercice 'Sphère photonique'


pages_mesure/za2.html

Exercice 'Redshift'


pages_mesure/exo-calotte.html

Exercice 'Calotte sphérique'


pages_mesure/exo-potentiel-terre.html

Exercice 'Potentiel gravitationnel de la Terre'


pages_theoremes/ex-sn1a.html

Exercice 'Supernova 1A'


pages_theoremes/exbnum.html

Exercice 'Bruit de numérisation'


pages_int-gen/exo-loi-de-stefan-int-gen.html

Exercice 'loi de Stefan'


pages_int-gen/exo-maxwell.html

Exercice 'Intégrales gaussiennes'


pages_int-gen/exo-maxwell.html

Exercice 'Distribution des vitesses'


pages_int-gen/exo-tps-vie-etoile-int-gen.html

Exercice 'Temps de vie d'une étoile'


pages_dl/lensing-ex.html

Exercice 'Lentille gravitationnelle'


pages_dl/exo-reperage-astre.html

Exercice 'Repérage des astres'


pages_dl/eq-kepler-exo.html

Exercice 'Equation de Kepler'


pages_dl/exo-limite-roche.html

Exercice 'Effet de marée et limite de Roche'


pages_dl/exo-coef-taylor.html

Exercice 'Générateur de coefficients de Taylor'


pages_dl/exo-dl-planck.html

Exercice 'Limites de la loi de Plack'


pages_deriv-part/exo-plinex.html

Exercice 'Perturbations linéaires'


pages_deriv-part/exo-potgravter.html

Exercice 'Potentiel gravitationnel terrestre'


pages_ed-01/ex-transfert-de-rayonnement.html

Exercice 'Atmosphère planétaire en visible'


pages_ed-01/ex-transfert-de-rayonnement.html

Exercice 'Face nuit de Vénus'


pages_ed-01/exo-eq-kepler.html

Exercice 'équation de Kepler'


pages_ed-02/exo-eqmathieu.html

Exercice


pages_ed-02/exo-ed-02.html

Exercice 'Détermination du potentiel dans une sphère de Debye'


pages_ed-02/exo-pb-22-corps.html

Exercice ' Le problème de 2 corps'


pages_ed-02/exo-train-gravitationnel.html

Exercice 'Train gravitationnel'


pages_ed-02/exo-pendule-foucault.html

Exercice 'Pendule de Foucault'


pages_pt-fixe/exo-loi-de-wien-pt-fixe.html

Exercice 'loi de Wien'


pages_exp/exo-magnitudes.html

Exercice 'Magnitude du Soleil vu de alpha du Centaure'


pages_exp/exo-magnitudes.html

Exercice 'Magnitudes absolues du Soleil et de Véga'


pages_exp/exo-magnitudes.html

Exercice 'Magnitude de l'amas des pleïades'


pages_exp/exo-magnitudes.html

Exercice 'Visibilités des satellites de mars'


pages_exp/exo-magnitudes.html

Exercice 'Magnitude apparente d'une planète'


pages_exp/exo-magnitudes.html

Exercice 'Magnitude instrumentale'


pages_exp/exo-mpa-exp2.html

Exercice 'Désintégration radioactive'


pages_exp/exo-mpa-exp2.html

Exercice 'Datation de météorites'


pages_hyper/courb-neg2.html

Exercice 'Univers à courbure négative'


pages_hyper/exo-eq-kepler-hyp.html

Exercice 'équation de Kepler hyperbolique'


pages_trigo/exo-astrolabe.html

Exercice 'Astrolabe'


pages_trigo/exo-carte-du-ciel.html

Exercice 'Carte du ciel'


pages_der-part/ent-exo.html

Exercice 'Entropie'


pages_der-part/exo-pb-2corps-hami.html

Exercice 'Formulation hamiltonienne du problème de 2 corps'


pages_der-part/exo-delaunay.html

Exercice 'Les variables de Delaunay'


pages_der-part/exo-th-inversion-lagrange-der-part.html

Exercice 'Théorème d'inversion de Lagrange'


pages_der-part/exo-tisserand.html

Exercice 'Paramètre de Tisserand'


pages_int-mult/exo-photometrie.html

Exercice 'Soleil'


pages_int-mult/exo-photometrie.html

Exercice 'Surface lunaire'


pages_int-mult/exo-thm-liouville.html

Exercice 'théorème de Liouville'


pages_ser-fourier/spec-ex.html

Exercice 'spectre de puissance'


pages_residus/exo-exc-limite.html

Exercice 'Excentricité limite dans les développements du problème à 2 corps'


pages_residus/exo-refraction.html

Exercice 'Relations de Kramers-Kronig'


pages_thales/exo-terre-lune.html

Exercice 'Calcul de la distance Terre-Lune'


pages_pythagore/exo-visibilite-satellite.html

Exercice 'Zone visible d'une montagne ou d'un satellite'


pages_pythagore/exo-pythagore.html

Exercice 'Pourquoi peut-on regarder le Soleil couchant ?'


pages_pythagore/exo-pythagore.html

Exercice 'Calcul de la masse d'air'


pages_pythagore/exo-vitesse-lumiere.html

Exercice 'Mouvement des satellites de Jupiter'


pages_applications/exo-loi-aires-geo.html

Exercice 'La loi des aires'


pages_applications/exo-vit-orb-terre.html

Exercice 'Vitesse orbitale de la Terre'


pages_applications/exo-equ-kepler-geo.html

Exercice 'Equation de Kepler'


pages_produits/exo-prod-2corps.html

Exercice 'La loi des aires'


pages_produits/exo-prod-2corps.html

Exercice 'Intégrale de Laplace'


pages_coniques/exo-balistique.html

Exercice 'Trajectoire balistique dans le système solaire'


pages_coniques/exo-conique-pb-22-corps.html

Exercice 'Les solutions du problème de deux corps'


pages_coniques/exo-masse-trou-noir.html

Exercice 'Masse du trou noir central de la galaxie'


pages_geospace/exo-angles-solides.html

Exercice 'Angles solides'


pages_geospace/exo-angles-solides.html

Exercice 'Photométrie'


pages_geospace/exo-croissantlune.html

Exercice


pages_poly/exo-factorisation-laplace.html

Exercice 'Factorisation du polynôme de la méthode de Laplace'


pages_poly/exo-poly-legendre-racines2.html

Exercice 'Les racines des polynômes de Legendre'


pages_syst-lin/exo-sat-coorb.html

Exercice 'Equilibres de satellites co-orbitaux'


pages_syst-lin/exo-expo-lyap.html

Exercice 'exposants de Lyapunov'


pages_appli-lin/exo-lap-lag.html

Exercice


pages_appli-lin/exo-lever-coucher.html

Exercice


pages_appli-lin/exo-ed.html

Exercice 'Etoiles doubles'


pages_spectre/exo-determinant.html

Exercice 'Relation de dispersion '


pages_spectre/exo-wiener.html

Exercice 'Filtrage de Wiener'


pages_quotient/exo-phenomenes-mutuels.html

Exercice 'Phénomènes mutuels'


pages_quotient/exo-atome-bohr.html

Exercice 'Modèle de Rutherford'


pages_quotient/exo-atome-bohr.html

Exercice 'Atome de Bohr'


pages_quotient/exo-fracont.html

Exercice 'Propriété des réduites'


pages_quotient/exo-fracont.html

Exercice 'Une meilleure approximation de l'année tropique'


pages_quotient/exo-fracont.html

Exercice 'L'automate de Huygens'


pages_quotient/exo-fracont.html

Exercice 'Le cycle de Saros'


pages_proba/poisson.html

Exercice 'Dérivation de la loi de Poisson'


pages_proba/gauss.html

Exercice 'Limite de détection d'un objet ponctuel'


pages_proba/exo-soleil.html

Exercice 'Chemin optique d'un photon dans le Soleil'


pages_stat/se.html

Exercice 'Estimation d'une moyenne'


pages_stat/err.html

Exercice 'Erreurs sur une fonction'


pages_dist-gauss/ex-m-carre.html

Exercice 'Moindres carrés'